The girl came to the sports club by chance ответы егэ

Задание №9141.
Аудирование. ЕГЭ по английскому

Вы услышите диалог. Определите, какие из приведённых утверждений А—G соответствуют содержанию текста (1 — True), какие не соответствуют (2 — False) и о чём в тексте не сказано, то есть на основании текста нельзя дать ни положительного, ни отрицательного ответа (3 — Not stated). Прослушайте запись дважды.

A. The girl came to the sports club by chance.
B. The girl is sure what sports classes she needs.
C. The girl does stretching using videos.
D. The girl has never been ill since high school.
E. The girl doesn’t find yoga interesting.
F. The club doesn’t have a big choice of dancing classes.
G. The girl is ready to make up her mind about what class to choose.

Утверждение A B C D E F G
Соответствие диалогу              

Решение:
Утверждение A (The girl came to the sports club by chance. — Девушка попала в спортклуб случайно) соответствует содержанию диалога — True: «Good morning, sir. I was just passing by and I thought I’d drop in.»

Утверждение B (The girl is sure what sports classes she needs. — Девушка уверена, какие спортивные занятия ей нужны) не соответствует содержанию диалога — False: «The thing is that I want to take up some sport but I can’t make up my mind which one to choose.»

Утверждение C (The girl does stretching using videos. — Девушка делает растяжку по видео) не отражено в диалоге — Not stated.

Утверждение D (The girl has never been ill since high school. — Девушка ни разу не болела со школы) не отражено в диалоге — Not stated.

Утверждение E (The girl doesn’t find yoga interesting. — Девушка не интересуется йогой) соответствует содержанию диалога — True: «I’m not quite sure yoga is my cup of tea, isn’t it boring?»

Утверждение F (The club doesn’t have a big choice of dancing classes. — В клубе нет большого выбора танцевальных классов) не соответствует содержанию диалога — False: «I was thinking of some dance classes. Actually we have classes of many dancing styles.»

Утверждение G (The girl is ready to make up her mind about what class to choose. — Девушка готова определиться, какой класс выбрать) не соответствует содержанию диалога — False: «Can I try the classes before making a final decision?»

Показать ответ

Источник: ФИПИ. Открытый банк тестовых заданий

Сообщить об ошибке

Тест с похожими заданиями

Задание №9141.
Аудирование. ЕГЭ по английскому

Вы услышите диалог. Определите, какие из приведённых утверждений А—G соответствуют содержанию текста (1 — True), какие не соответствуют (2 — False) и о чём в тексте не сказано, то есть на основании текста нельзя дать ни положительного, ни отрицательного ответа (3 — Not stated). Прослушайте запись дважды.

A. The girl came to the sports club by chance.
B. The girl is sure what sports classes she needs.
C. The girl does stretching using videos.
D. The girl has never been ill since high school.
E. The girl doesn’t find yoga interesting.
F. The club doesn’t have a big choice of dancing classes.
G. The girl is ready to make up her mind about what class to choose.

Утверждение A B C D E F G
Соответствие диалогу              

Решение:
Утверждение A (The girl came to the sports club by chance. — Девушка попала в спортклуб случайно) соответствует содержанию диалога — True.

Утверждение B (The girl is sure what sports classes she needs. — Девушка уверена, какие спортивные занятия ей нужны) не соответствует содержанию диалога — False.

Утверждение C (The girl does stretching using videos. — Девушка делает растяжку по видео) не отражено в диалоге — Not stated.

Утверждение D (The girl has never been ill since high school. — Девушка ни разу не болела со школы) не отражено в диалоге — Not stated.

Утверждение E (The girl doesn’t find yoga interesting. — Девушка не интересуется йогой) соответствует содержанию диалога — True.

Утверждение F (The club doesn’t have a big choice of dancing classes. — В клубе нет большого выбора танцевальных классов) не соответствует содержанию диалога — False.

Утверждение G (The girl is ready to make up her mind about what class to choose. — Девушка готова определиться, какой класс выбрать) не соответствует содержанию диалога — False.

Показать ответ

Источник: ФИПИ. Открытый банк тестовых заданий

Сообщить об ошибке

Тест с похожими заданиями

1 Dear David,
Thanks for your letter. It was great to hear from you.
I really like my school. Most of the teachers are quite relaxed and friendly. As for the sports facilities, they are so much better than at my previous school. There are clay and grass tennis courts, and best of all — an Olympic-sized swimming pool!
I haven’t joined any school clubs yet because I’m trying to work out which ones I can fit into my schedule. I have to do swimming practice most days.
Who was your first basketball match against? What position did you play? Most importantly, did your team win?
I have to go now as I’ve got a swimming lesson this evening. Hope to hear from you soon.
Best wishes,
Tom Childhood obesity is a serious problem. Therefore, some people say that schools should ban the sale of junk food. In my opinion, schools should be allowed to sell junk food because this is not the main reason that children are getting fat.
To begin with, if you stop children from eating junk food in schools, this does not mean that they will stop eating junk food altogether. They could buy it from a shop, for instance.
In addition, junk food only causes weight gain if too much of it is eaten. Therefore, it is better for schools and parents to educate children about healthy eating so that they learn to make the correct food choices early in life.
On the other hand, some say that many children are not mature enough to make sensible choices for themselves and therefore it is better if junk food is banned in schools. They believe that if children learn to appreciate a healthy diet at school, they will develop good habits for the rest of their lives. This may be true, but it is human nature to want what we are told we can’t have, so banning junk food in schools may only encourage many children to eat it out of school!
All things considered, it seems to me that banning junk food in schools is not a good idea because children could still get junk food out of school. It is better to encourage children to make healthy food choices for themselves 2 Dear Mrs Sampson,
Thank you for your letter. My journey back home was fine. My plane was on time and I chatted with a really nice girl who was sitting next to me. I had a wonderful time in
England. I felt that I learnt a lot of English and really improved my accent, too. As for what I enjoyed most, I really loved the day I went to London with you and Mr Sampson. I enjoyed all of my English classes, too!
It’s great that you’ve got a puppy! Is it a male or a female? Have you chosen a name for it yet? What kind of dog is it?
Thanks again for having me. Write back soon.
Love,
Anna The Internet is an extremely useful tool that has become an important part of our lives. In particular, it has made searching for information much easier and faster than it used to be. However, some people say that finding information in this way can make life more difficult, too.
On the one hand, using the Internet to find information has several advantages. Firstly, the fact that the Internet enables us to find information quickly saves time. Students can do research for school projects without going to the library as they did in the past, for example. Also, we can all find out travel information, read the news and do many other things via the Internet without having to leave our homes. Also, the Internet can be very educational as a huge amount of information is available on any topic.
On the other hand, finding information on the Internet can have some disadvantages. One major problem is that information found on websites is not always accurate.
Secondly, the fact that there is so much information on the Internet means that it is sometimes difficult to find what you are looking for. Also, due to the wide variety of information on the Net, it can be very easy to get distracted from what you are supposed to be doing.
In conclusion, there are both advantages and disadvantages of finding information on the Internet. On the whole, I believe that the Internet is a very useful tool for our modern lives, but it should be used wisely 3 Dear Mark,
Thanks for your letter. I can’t wait for your visit! Don’t worry about getting to my house from the airport — if my dad isn’t working he will come and pick you up and if he is, I will come and meet you and we will catch a bus to my house. As for a gift for my parents, what about something traditionally English, such as some shortbread biscuits or a nice box of English tea? Also, don’t forget to pack some warm clothes. Moscow can still be quite cold in spring!
How are your grandparents going to celebrate their wedding anniversary? Have you bought them a gift? Do you get along with your grandparents? Well, I have to go now. See you next month!
Best wishes,
Alex Nowadays, it seems that pressure to gain more qualifications to get better jobs has encouraged many people to take up distance learning, by post or, more commonly, via the Internet. In my view, however, there are still more benefits to learning in a traditional classroom environment.
On the whole, despite the increase in its popularity, I do not believe that distance learning is better than traditional classroom learning. Firstly, students have to be very dedicated and self-motivated to study by themselves and many students would find this difficult.
Also, some people need more support in their studies than a tutor can give online or by post. Others may miss the interaction with fellow students that always occurs in a traditional classroom.
On the other hand, distance learning has some positive aspects. Firstly, it can be done in the privacy of one’s own home. This allows people to fit their studying around their work schedules and the demands of home life. Secondly, it allows students to work without distractions from others and at their own pace. However, I still feel that most students would benefit more from having the input of a teacher and other students in a traditional classroom.
To sum up, while distance learning is a good solution to help many people, whom have very little time to spare, gain more qualifications, in my opinion it is not better than traditional classroom learning 4 Dear Sir/Madam,
I am writing to apply for the position of part-time waiter which was advertised in this week’s edition of The Inquirer.
I am 18 years old and I am in my first year at university. I already have experience working as a waiter, having spent the past three summers working at the Hilton Hotel as a breakfast waiter.
I consider myself to be hard-working, efficient and cheerful. I enjoy dealing with people and I think I am very good with the public.
I enclose two letters of recommendation from my previous employers. I would be very grateful if you consider my application. I am available for interview any weekday morning.
I’m looking forward to hearing from you.
Yours faithfully,
Sam Brown Most people prefer to stay in a hotel while on holiday since it provides them with good meals, comfortable furnishings, housekeeping services etc. Nevertheless, some people prefer to camp in the great outdoors.
Personally, however, I would favour staying in a hotel.
First of all, I believe that hotels are better because after a busy and tiring year people need to go on holiday to relax. At a hotel you don’t have to worry about preparing meals since they can be cooked for you there.
Moreover, there is no need to clean up after yourself since housekeeping services are provided. In addition to this, it’s great to have a comfortable bed to sleep in at night.
On the other hand, outdoor enthusiasts say that it is better to go camping. They believe it’s the only way to experience nature; the fresh air, forests and wildlife.
However, you can experience all this without having to camp outside. Hotels are often near beautiful countryside locations or national parks. Besides, it’s uncomfortable to sleep on hard ground, surrounded by mosquitoes and who knows what else outside your thin nylon tent!
To sum up, while camping allows you to experience nature, I believe that staying in a hotel room is more favourable. You can experience all that nature has to offer during the day, if you wish, and return to a comfortable hotel room at night 5 Dear Julie,
It was great to hear from you. You asked me about the
food we eat in Russia. Well, at home we eat many different kinds of meat and potato dishes. We also eat a lot of salads, such as cabbage salad, pickled vegetables and different kinds of soups.
A traditional Russian dish is Borsch, a kind of beetroot soup. In my family, we eat this with a spoonful of sour cream in it. It’s really delicious!
Although many people still eat a lot of traditional foods, I think that we are all eating more fast food and readymade food these days because we lead busier lives than in the past.
So where are you going with your friends this summer? What are you going to do there? What kind of accommodation will you stay in?
Write back soon.
Love,
Ivan Today, many households own more than one car. As a result, there are too many cars in our towns and cities, particularly in the centre. This causes several problems such as increased air pollution, which affects peoples’ health, and can cause delays due to traffic jams. Fortunately, there are a number of possible ways to deal with this problem.
One way to decrease the number of cars on the roads would be to improve public transport and encourage people to use it. If towns and cities invested in a new cheap, fast and efficient underground or tram service, for example, or provided more bus routes, people would definitely use them. This would ease the traffic problem and make it easier for people to get around, too.
Another possible solution would be to make driving less attractive by introducing expensive parking charges. This would definitely encourage people to leave their cars at home and catch public transport instead.
Finally, ‘park-and-ride’ schemes could be set up whereby drivers can leave their cars at an out-of-town car park and then catch a bus into the town or city centre. This kind of scheme would be very easy to set up and would provide a long-term solution to the problem of traffic congestion.
To sum up, there are a number of possible ways to deal with the problem of too many cars on our roads. However, I believe that in order to reduce the amount of traffic, all of these ideas need to be put into practice, not just one 6 Dear Sir/Madam,
I am writing to express my strong dissatisfaction with the service I received at your restaurant.
Last Friday, four friends and I had dinner at the restaurant to celebrate my birthday. I had heard good reports from other people about the food and I was expecting a wonderful evening. Unfortunately, I got the exact opposite.
After we were seated, we had to wait for more than half an hour until a waiter came to take our order. It then took an hour for our first course to arrive. When it came, the food was undercooked and cold.
Naturally we complained, but the staff refused to replace our meals or apologise and were generally very rude.
As you can imagine, I was very upset and my birthday was ruined. I feel I am entitled to a full refund, in addition to a written apology from the local manager.
I’m looking forward to receiving your prompt reply.
Yours faithfully,
Claire Sanders Russian is one of the most widely spoken languages in Eurasia and therefore an important language to learn.
Although it is considered to be difficult, I believe there are things students can do to help themselves learn it more effectively.
Firstly, in my opinion, as when learning any language, an excellent way for foreigners to improve their Russian is for them to find a pen-friend. By doing this, they will be able to build on and practise the language skills they have learnt in class. In addition, it would be very beneficial for foreigners to travel to Russia for a few weeks’ holiday or even to stay with a pen-friend’s family. This way, they will pick up commonly used phrases and learn how to use them in a natural way. What’s more, watching films and TV programmes, listening to music and reading magazines and leaflets in Russian would also help.
No doubt, there are certain things foreigners will find difficult about my language. For one thing, it may be hard for them to learn the complicated Russian grammar rules. Furthermore, foreigners may have trouble with the pronunciation of the Russian alphabet as some Russian vowel sounds don’t have equivalents in other languages.
To overcome these challenges, I suggest that foreigners take lessons with a good teacher in order to build a solid foundation in the language.
In conclusion, I don’t believe that learning Russian is any more difficult than learning another language. Students will need to take a course to learn the rules, grammar structures and proper pronunciation. Also, it would be a good idea for them to practise with native speakers, read and listen to Russian as much as they can and eventually travel to Russia to be immersed in the language 7 Dear Steve,
Great to hear from you! I can hardly believe it either that we are finishing school soon.
I plan to go to university immediately after my summer break. At the moment, I am preparing to take the very difficult entrance exams. I hope to study Computer Science at Moscow State University. I would like to be a software designer someday and work at a top IT (Information Technology) company. Higher education is considered very important in Russia, so most students work hard to get into one of the many excellent higher education institutions.
I’m sure your gap year will be fantastic! Why did you choose to volunteer in Nepal? What sort of things will you be doing at the orphanage? How long will you be in Nepal?
Hope to hear from you soon.
Take care,
Victor Recently, more and more young people have been choosing to eat only vegetarian food because they think it is better for their health. However, many nutritionists say that following a vegetarian diet exclusively is not such a healthy option.
On the one hand, there are several advantages to being a vegetarian.
Firstly, a vegetarian diet can provide more vitamins, minerals and fibre than a meat-based diet. In addition, vegetables contain fewer calories than meat.
This means that vegetarians are often fitter and slimmer than meat-eaters. Furthermore, vegetables are much cheaper than meat. Lastly, as meat contains a lot of fat it is known to increase the risk of heart disease.
On the other hand, being a vegetarian is not without its disadvantages. For a start, vegetables contain less protein, which is important for building muscle and giving us energy. Also, some important nutrients are found in meat and fish. Therefore, sometimes vegetarians need to replace these by taking supplements, such as vitamin pills, which can be expensive.
To sum up, although there are some drawbacks to being a vegetarian, I feel that on the whole it is not an unhealthy way to live. However, eating meat ensures that a person gets all the proteins, vitamins and minerals they need 8 Dear Sir/Madam,
I am writing to inquire about the summer English language course offered by your college. I am interested in attending the course and would like some more information about it.
Firstly, I would like to know the cost of the course and if there are any additional costs for textbooks or other materials. I would also like to know when the course begins and when it finishes. Finally, could you please tell me if accommodation is provided for students or if it is necessary for students to find their own?
I’m looking forward to receiving this information. Thank you in advance.
Yours faithfully,
Igor Petrov Technology has become part of our lives and many families have computer and video games in their homes.
While many parents are becoming increasingly concerned that these games have a harmful effect, I believe that they can often be beneficial and so children should be allowed to play them.
In my opinion, children can definitely benefit from playing video games.
Firstly, allowing children to play video games is a good way of keeping them occupied in the safety of their own home. Also, not all games are violent and some are, in fact, educational. In addition, some psychologists argue that it is better for young people to release any aggressive feelings they have by playing a video game rather than in real life.
On the other hand, some parents believe that there are several disadvantages to playing video games. Firstly, they say that if the game is violent, then the children will adopt the same behaviour and act in a similar way in real life. Secondly, they feel that as children spend so many hours playing these games, they often don’t have enough time to do their homework and their performance at school suffers. However, if parents make sure that the games are not too violent and that children don’t play the games for too long, then neither of these things should be a problem.
In conclusion, video games have both benefits and drawbacks. As long as parents keep an eye on what games are being played and for how long, I believe that they can be an enjoyable pastime for children 9 Dear Julia,
Thanks for your letter. You sound very busy.
I used to go to ballet classes three times a week, but now I have started modern dance classes instead. I only go once a week so I have more time to do other things now. In particular, I really enjoy reading novels and playing my guitar.
I don’t go out a lot in the week, but at weekends I usually meet up with my friends and we go to the cinema or just walk around the town.
How long will you be in Ireland? Have you been there before? I’ve heard that it’s a beautiful country. How many people are going on the trip?
Anyway, good luck in the competition and please send me a postcard from Ireland.
Love,
Anya More people are travelling abroad for their holidays these days. While some prefer to make their own arrangements, others choose to go on a package holiday where everything is arranged for you. In my opinion, it is more satisfying to plan your own holiday.
Firstly, you experience more freedom and independence if you arrange everything for yourself. This means that you can choose exactly where you want to stay and what you want to see. Also, you can take your time enjoying the sights as there is no schedule to restrict you. Furthermore, it is often much cheaper to make your own arrangements. You are able to choose the type of accommodation that suits your budget and how long you stay in a particular place. You can also eat quite inexpensively in local restaurants.
On the other hand, package holidays do have several benefits. For a start, they are easier and more relaxing as flights, hotels and sometimes entertainment are all organised for you. Additionally, there is usually a holiday representative to help you with any problems. However, as everything is programmed for you on a package holiday, you may not get the chance to learn as much about the local language and culture.
In conclusion, while package holidays may be more suitable for some people, I believe that arranging your own holiday is better on the whole. That way, you are able to decide on your own programme with very few restrictions, which makes the holiday a more fulfilling experience 10 Hi Harry,
Thanks for your letter. I can’t believe you have to catch two buses to get to school.
I actually live quite close to my school so I walk there with my two friends. I don’t really find the journey very tiring because it’s only a fifteen-minute walk. As for public transport in my town, well, there are plenty of buses, but most of them are quite old and dirty and there is usually a lot of traffic. There is a new metro system though that is fast and clean.
That’s a great idea to have an environment day! Did it go well? What kind of activities and events were there? What did you like best about the day?
Well, I have to go now but I hope to hear from you again soon.
Best wishes,
Dimitri Nowadays, many parents are choosing to pay for private education for their children because they believe that it is better than state education. However, do children always benefit from going to private schools?
Without a doubt, there are some advantages to sending children to private schools.
Firstly, private schools often have better facilities and a wider curriculum than state schools. For example, most have the latest computer technology and sports facilities and they are often able to offer different subjects such as dance and photography. Additionally, the teaching is usually of a high standard as private schools are able to attract good teachers. Also, there is pressure on students to do well at private schools because their parents are paying for their education. This may eventually mean that children get better qualifications.
On the other hand, there are several drawbacks to private education. First of all, it is very expensive and there may be some excellent state schools that are just as good. Secondly, some students may feel too pressured to do well in their studies at a private school.
They would possibly do better at a normal state school where they could study at their own pace in a more relaxed atmosphere.
In conclusion, private education may give some students better chances of success in life. However, parents should remember that private education can have some disadvantages and may not always be the best choice for their child 11 Dear Tom,
Thanks for your letter — it was great to hear from you!
You asked about my family. Well, there are five people in my immediate family. There’s my mum and dad, myself, my younger brother Jack who’s 13 and my younger sister Emily who’s 8. We often go to the park altogether at weekends and we fly kites or play football. In the evenings, we always eat together and then watch TV. As for my favourite relative, well, that’s easy! I love going to visit my grandma on Sunday afternoons — I like sitting in her beautiful garden chatting to her or helping her to bake a cake.
So where exactly did you go in Scotland? Did you go to Loch Ness and see the Loch Ness monster? What was the food like; did you try haggis?
Write again soon.
Love,
Diana Many people believe that it is cruel and unnecessary to keep animals in zoos as conditions in them are very different to those in the animals’ natural environments.
However, personally I do believe that there are some good reasons for keeping zoos open.
Firstly, many modern zoos spend a lot of time, money and energy trying to protect animal species. In fact, zoos’ breeding programmes are often the only hope for endangered species such as gorillas. In addition, zoos these days usually look after their animals very well and try to recreate the conditions in the animals’ natural environment. As animals are often in danger in the wild from loss of habitats and hunters, a zoo can be a safer environment than a natural habitat. Lastly, people who visit zoos usually leave with a better understanding of and respect for animals.
On the other hand, opponents of zoos argue that it is never good to keep an animal in an unnatural environment. They say that there are still some zoos in which animals are not well looked after. They may be kept in small, dirty cages and not given the correct food and medical treatment. However, I feel that these kinds of zoos are not very common.
To sum up, while it would be great if animals could all live in their natural environments, these days they face many dangers in the wild. Therefore, I believe that there are several very good reasons to keep animals in zoos 12 Hi Jack,
Thanks for your letter. At my school, we take exams twice a year — at Christmas and again in the spring.
Some of my friends like taking exams because, they say, it gives them a chance to show what they can do, but personally I don’t like them at all. I find they make me feel a bit stressed out! I try to play basketball, however, a few times a week during exam time. I find this really relaxes me a lot and helps me to cope with the pressure.
I also try to get plenty of sleep. So, it’s your best friend’s birthday this weekend! How is he going to celebrate it? How old will he be? Are you going to buy him a nice gift?
Good luck with your exams. Write back soon.
Best wishes,
Gary Most people who enjoy extreme sports are young people who enjoy the excitement of these thrilling activities such as white-water rafting, bungee jumping and sky surfing. Some people claim, however, that these sports are too dangerous. In my opinion, people should be allowed to take part in these activities if they are properly trained before and if they follow safety instructions.
Firstly, I believe that extreme sports are usually safe because these activities are well organised by properly trained instructors. Participants have to wear correct clothing, check their equipment and be trained before they are allowed to participate in an extreme sport like sky surfing. In addition, extreme sports can be beneficial to people’s health. Taking part in them can help people to relax and let off steam. They could even help people to overcome their fears of taking risks.
On the other hand, opponents of extreme sports say that there is always the risk of something going wrong while participating in an extreme sport. If a rope snapped while bungee jumping, for example, a participant would be seriously injured or even die. As mentioned before, however, I feel that the risk is not very high, because organisers of extreme sports take safety very seriously.
To sum up, while extreme sports carry a low risk of injury, I believe that they are safe to take part in if the participants carefully follow all the instructions. They can even benefit the participants mentally 13 Hi David,
Thanks for your letter. You asked me about pocket money. Well, I do get some pocket money, but like your parents, mine don’t have a lot of extra money so they can’t give me very much.
I think it’s good for parents to give their children some money if they can afford it, because it teaches them about managing money. Some teens here in Russia have part-time jobs in the evenings or at weekends, but it’s often quite difficult to find work.
Personally, I don’t want to work because I’m too busy with my schoolwork, but it would be nice to have some extra money!
So, what musical instrument are you learning to play? Are you taking lessons? Can you read music?
Well, I have to go now. Write again soon!
Best wishes,
Alexandra Since the first commercial flight over fifty years ago, aeroplanes have helped billions of people around the world to travel to their destinations. But is travelling by plane always the best option?
On the one hand, there are several clear advantages of air travel. The main advantage of travelling by plane is that it is quicker than any other means of transport. One can travel from one continent to another in just a few hours. Also, it is safe. There are fewer accidents in the air than on land or at sea. Moreover, a lot of passengers tend to enjoy the whole flying experience; the reclinable seats, airline feature films and being served food and drink.
However, travelling by plane does have its disadvantages.
Firstly, the tickets are usually quite expensive and so not everyone can afford them. Secondly, travelling by plane is not always very comfortable. Passengers have to sit in small seats for long periods of time. In addition, planes can’t fly travellers to their exact location and airports are usually far from city centres. Finally, there are the frequent problems of missing luggage, cancelled flights and long check-in queues which can make passengers’ journeys take much longer than they should.
To conclude, despite the disadvantages, I do believe that air travel is often the best way to get to a particular destination. In fact, for destinations that are very far away, it is the only realistic way to get there 14 Hi Sally,
Thanks for your letter and I hope you’re well! Your environment group sounds great.
We have a big problem in my neighbourhood; there are no green spaces at all. Also, it’s quite a busy area so there’s a lot of pollution from cars and factories. I think that I help the environment in a few ways; I always turn off lights when I’m not in a room and I try not to waste water. I also recycle newspapers and plastic. I think we can save the planet but we have to educate everybody and we all have to work together.
The surprise party sounds like good fun! Where are you going to have the party? Are you going to invite lots of people? Do you think Anna suspects anything?
Write back soon.
Lots of love,
Laura Starting your own business often seems like an attractive alternative to being an employee. In my opinion, however, it brings a lot of risk and responsibility and is sometimes not worth the time and effort.
Firstly, you usually have many more responsibilities when you run your own business. You may have to work long hours because you are ultimately responsible for getting the work finished on time. It can also be extremely risky to start your own business. For one thing, many new businesses fail in the first one or two years. Also, the costs of starting up your business can be very high. Installing computers, printers and other office equipment is very expensive and you have to pay your employees each month.
On the other hand, it is true that owning your own business can be very satisfying. You are in charge, after all, and have the freedom to make all your own decisions.
You can decide on the hours that you work, and how much you pay your staff. Moreover, if all goes well, you will have the chance of making a large profit, too.
In conclusion, I believe that while starting a business can be the right decision for many people, it is quite risky and brings a lot of responsibility. Personally, I would prefer to be an employee 15 Dear Alex,
Thanks for your letter. It was great to hear from you!
I love listening to rock music, too! Sometimes I listen to techno with my friends, as well. One of my friends is a big fan!
As for TV, I have to say that our house is the same as yours! The TV is on a lot, especially after school. We generally watch TV as a family in the evenings for about two hours. We usually like to watch soap operas or comedy programmes.
Were there a lot of people at the festival? What type of music was played? Did any famous bands play at the festival?
Write soon.
Best wishes,
Dina Many people say that they would love to become famous — perhaps a singer, actor or TV presenter. Of course, in many ways the life of a famous person is fantastic, but there are definitely some negative aspects of the celebrity lifestyle, too.
There are some obvious advantages to being a famous celebrity.
Firstly, a celebrity’s lifestyle usually brings many great opportunities. A famous singer, for example, travels all over the world, stays in luxury hotels and meets interesting people. In addition, famous people earn a lot more money than most people and can afford things such as large houses, designer clothes and luxury holidays. Finally, famous people are living their dreams, which must be very satisfying.
On the other hand, there are some clear disadvantages to being famous. First of all, you are recognised everywhere you go. You can no longer go for a walk in your neighbourhood or pop to the shops without being followed and photographed by the media or even by fans. Even worse, the media often print unpleasant or untrue stories about you in newspapers and magazines.
Lastly, some famous people find that fame and money do not bring them happiness. Eventually, their stressful celebrity lifestyles cause them to have problems such as depression or even worse.
To sum up, while a celebrity lifestyle can be easier and more fun than a normal lifestyle in many ways, it can be very stressful. Personally, I would not really like to be famous! 16 Dear Sir,
I recently stayed at your bed and breakfast and had a very nice time.
Unfortunately, however, when I got home I realised that I didn’t have my diary and that I must have left it in my room.
I think that I left the diary in the top drawer of the bedside table on the right-hand side of my bed. I was staying in room 12. The diary is quite large and it is pink with a yellow flowery pattern on it.
I would be extremely grateful if you could send my diary back to me at the above address if you find it. I would be happy to send you a cheque to cover the postage costs.
Thank you in advance.
Yours faithfully,
Mary Waters Everyone has a special place they have fond memories of, somewhere that remains dear to them. For me, that place is Long Sault in Ontario, Canada. It’s my favourite spot in the world because that’s where I spent many happy summers as a child.
Long Sault is a beautiful area made up of eleven islands connected by bridges. These islands were formed after the flooding of the St Lawrence River in the 1950s.
Around these picturesque islands with their tall, green grass and the occasional wooded area, are pretty sandy beaches and areas of rock from where travellers can fish or swim. There is so much to do there; hiking for outdoor enthusiasts, bird watching, camping, kayaking, golf and even goose feeding.
I have so many fond memories of Long Sault. My cousins and I spent many mornings there looking under colourful rocks for crayfish and crabs. That’s the place where I hooked my first worm, caught my first fish and climbed my first tree. In the evenings, my uncle would always make a campfire and we’d barbecue whatever we had managed to catch during the day.
Even though I’m currently living nowhere near Long Sault, I will never forget the place which gave me so many unforgettable memories. Sometimes when I close my eyes, I feel like I am experiencing those moments again 17 Dear Becky,
Thanks for your letter. How are you?
It was interesting to hear about homes in Great Britain.
We’ve just moved to a bigger flat, so now my sister and I have our own bedrooms. It’s great — I have enough space for all my belongings now. Most people in Russian towns and cities live in apartment blocks and unfortunately, often the apartments can be a little small for a family. I think I would rather live in the countryside than the town. It’s a lot quieter, there isn’t a lot of traffic or pollution and people are usually friendlier than in towns and cities.
Did you have a nice time staying with your uncle in Wales? Did you get up early to help milk the cows? Does your uncle have any other animals?
Write back soon.
Love,
Maxim Everyone agrees that teenagers should do some form of physical activity to stay healthy, but are sports the answer? In my opinion the answers is yes; all teenagers should take part in sports.
In the first place, doing sports has many health benefits.
For example, sports help teenagers stay physically fit and control their weight. Furthermore, sports help teens to increase their overall strength and coordination. In addition to the physical benefits of sports, there are also many psychological benefits. For instance, team sports teach social skills, teamwork skills and respect for authority, not to mention the fact that sports help increase one’s confidence. This often results in improved academic performance.
On the other hand, some argue that sports are not for everyone and that other activities have similar benefits.
Simply walking to school instead of catching a bus would keep a teen quite fit, for example, and taking part in a reading club would help improve social skills and confidence. Moreover, sports can create stress for some teens by putting pressure on them to be as good as their teammates. Not all sports are competitive team sports, however, and teenagers should be able to find one that suits them.
To conclude, I believe that sports should have a place in every teenager’s life. Besides the obvious physical benefits, teenagers develop important skills and build self-esteem through sports. Young people should all learn how to take care of their bodies and minds so that they continue doing this throughout their lives 18 Dear Monica,
It was great to hear from you! I’m sure you’ll have lots of fun at the wedding.
What about giving a traditional gift made of silver that the couple can keep forever? Or some money, which is what Russians usually give? A traditional Russian wedding is a huge celebration that can last several days. It starts with the groom picking up the bride at her home. The couple usually has a church ceremony followed by a civil one. There is usually lots of dancing, food and drink at the reception afterwards.
Weddings are great but my favourite Russian celebration is New Year’s Eve because of all the gifts and parties.
Your creative writing course sounds very interesting! What made you decide to take the course? Is it expensive? What do you write about?
Write soon.
Love,
Irene Mark Twain once said that ‘clothes make the man’, but is this really true? Nowadays, appearance and dress are very important to many school pupils, but many of them have to wear a uniform when they are at school. In my view, school uniforms are a good idea because they help avoid some potential problems.
To begin with, school uniforms help to reduce the amount of peer pressure that young people experience at school. If everyone has to wear the same clothes, noone can be teased or bullied because they are not wearing the latest fashions. Also, pupils can concentrate more on learning because they are no longer so worried about how they look. Furthermore, it is usually cheaper for parents if their children have to wear a uniform because they don’t have to buy lots of expensive fashionable outfits. Finally, wearing a uniform helps give pupils a sense of identity.
Opponents of this view argue that school uniforms prevent pupils from expressing their individuality through their clothes. However, there are many other ways in which students can express themselves — through taking part in school clubs and activities such as sports teams or the school newspaper, for example.
To sum up, in my opinion clothes do not make the man. Therefore, I believe that it is a good idea for school pupils to wear a school uniform because it promotes equality and a sense of belonging 19 Dear Jamie,
Thanks for your letter. Enjoy your new computer!
I don’t have my own computer, but I often go to an Internet café near my house. I usually do research on the Internet there for my school work. I also send a lot of emails to friends and family. Also, when I have free time, I like to play some of the online games with my friends at the café. We have a great time! I would love to have an MP3 player, so I could download my favourite music from the Internet to listen to whenever I wanted.
I’m sorry to hear about your arm. Get well soon. How did it happen? Are you wearing a cast? How long will it take to get better?
Hope to hear from you soon.
Best wishes,
Sergey Which is a better place to live, the city or the country?
Well, in my opinion, there are pros and cons of both, but personally I would much rather live in the city.
First of all, there is a wider range of jobs and careers available in the city than in the country. Very often, cities also have better education and health services. In addition, a variety of different kinds of people and cultures are usually represented in a city, which creates a very interesting environment. Moreover, big cities have better entertainment facilities such as shops, cinemas, theatres, galleries and sports centres. Lastly, public transport is better in the city than in the country, which makes it easier to get around.
On the other hand, many argue that city life can be very stressful. Also, they say that people in the country are friendlier and take better care of each other. This may be true, but it is usually not too hard to make good friends in a city. Also, there are many ways to deal with stress in a city, such as enjoying a relaxing walk in a park or going to a concert, a leisure centre or an outdoor festival.
All in all, I would prefer to live in a city because I believe that cities offer more opportunities and a far more interesting lifestyle than living in the country. It is often said that ‘variety is the spice of life’ and variety can definitely be found in cities 20 Dear Mr Burns,
Our school is holding its Careers Day on 21st May this year and we would very much like you to come to the school and give a talk.
The Careers Day is an opportunity for students to learn about different career opportunities. As you are a successful businessman, we would appreciate hearing about what makes a business successful and what careers and types of businesses you think will be most important in the future. The talk would be about half an hour long and would take place in the school gym at about 11 am.
I would be grateful if you could let me know whether you are able to accept our invitation as soon as possible. You can contact me at the above number.
I’m looking forward to hearing from you.
Yours sincerely,
Ivan Nikitin It is often said that children are the key to the future and this is certainly true with regards to the environment.
Fortunately, there are many ways to teach young people how to care for the environment.
To begin with, schools need to get more involved in environmental education. They could organise environment days, for example. These encourage students to think about different environmental issues and how they can get involved. Another way schools can educate students is by starting different programmes such as recycling and composting projects. As a result, students will grow up understanding how good habits make a difference in solving environmental problems.
Communities and families can also get involved in educating children about the environment. Local councils could organise events such as tree-planting or littercollecting days, for example. Consequently, young people would not only learn about these issues, but see the results of their actions, too. Finally, parents need to set good examples by being environmentally-friendly at home. They should recycle and save water and electricity, for instance. In this way, children will probably continue with the same habits when they have houses of their own.
All things considered, communities, families and schools can all successfully encourage children to care for the environment. By doing so, they can help to make sure that future generations will protect the planet

Тест ЕГЭ-2011 по английскому языку.

Демонстрационный вариант (II).

(Ответы в конце теста)

1. ЧТЕНИЕ (30 минут). Задания В2, В3, А15-А21 

B2. Прочитайте тексты.

1. Her Majesty’s Government, in spite of its name, derives its authority and power from its party representation in Parliament. Parliament is housed in the Palace of Westminster, once a home of the monarchy. Like the monarchy, Parliament is an ancient institution, dating from the middle of the thirteenth century. Parliament is the seat of British democracy, but it is perhaps valuable to remember that while the House of Lords was created in order to provide a council of the nobility for the king, the Commons were summoned originally in order to provide the king with money.

2. The reigning monarch is not only head of state but symbol of the unity of the nation. The monarchy is Britain’s oldest secular institution, its continuity for over a thousand years broken only once by a republic that lasted a mere eleven years (1649-60). The monarchy is hereditary, the succession passing automatically to the oldest male child, or in the absence of males to the oldest female offspring of the monarch. In law the monarch is head of the executive and of the judiciary, head of the Church of England, and commander-in-chief of the armed forces.

3. The dynamic power of Parliament lies in its lower chamber. Of its 650 members, 523 represent constituencies in England, 38 in Wales, 72 in Scotland and 17 in Northern Ireland. There are only seats in the Commons debating chamber for 370 members, but except on matters of great interest, it is unusual for all members to be present at any one time. Many MPs find themselves in other rooms of the Commons, participating in a variety of committees and meetings necessary for an effective parliamentary process/

4. Britain is a democracy, yet its people are not, as one might expect in a democracy, constitutionally in control of the state. The constitutional situation is an apparently contradictory one. As a result of a historical process the people of Britain are subjects of the Crown, accepting the Queen as the head of the state. Yet even the Queen is not sovereign in any substantial sense since she receives her authority from Parliament, and is subject to its direction in almost all matters. This curious situation came about as a result of a long struggle for power between the Crown and Parliament during the sixteenth and the seventeenth centuries.

5. Her Majesty’s Government governs in the name of the Queen, and its hub, Downing Street, lies in Whitehall, a short walk from Parliament. Following a general election, the Queen invites the leader of the majority party represented in the Commons, to form a government on her behalf. Government ministers are invariably members of the House of Commons, but infrequently members of the House of Lords are appointed. All government members continue to represent “constituencies” which elected them/

6. Each parliamentary session begins with the “State Opening of Parliament”, a ceremonial occasion in which the Queen proceeds from Buckingham Palace to the Palace of Westminster where she delivers the Queen’s Speech from her throne in the House of Lords. Her speech is drafted by her government, and describes what the government intends to implement during the forthcoming session. Leading members of the Commons may hear the speech from the far end of the chamber, but are not allowed to enter the House of Lords.

7.  The upper chamber of Parliament is not democratic in any sense at all. It consists of four categories of peer. The majority are hereditary peers, a total of almost 800, but of whom only about half take an active interest in the affairs of the state. A smaller number, between 350 and 400, are “life” peers – an idea introduced in 1958 to elevate to the peerage certain people who rendered political or public service to the nation. The purpose was not only to honour but also to enhance the quality of business done in the Lords.

Установите соответствие между заголовками A – Н и текстами 1 – 7. Используйте каждую букву только один раз. В задании один заголовок лишний. 

A) The House of Commons

B) Parliamentary Procedure

C) The House of Lords

D) Westminster

E) The System of Government

F) Parliamentary Committees

G) Whitehall

H) The Crown

B3. Прочитайте текст.

‘Second Stonehenge’ discovered near original

Archaeologists have discovered evidence of what they believe was a second Stonehenge located a little more than a mile away from the world-famous prehistoric monument.
The new find on the west bank of the river Avon has been called «Bluestonehenge», after the colour of the 25 Welsh stones of 1___________________.

Excavations at the site have suggested there was once a stone circle 10 metres in diameter and surrounded by a henge – a ditch with an external bank, according to the project director, Professor Mike Parker Pearson, of the University of Sheffield.

The stones at the site were removed thousands of years ago but the sizes of the holes in 2  _________________ indicate that this was a circle of bluestones, brought from the Preseli mountains of Wales, 150 miles away.

The standing stones marked the end of the avenue 3  __________________, a 1¾-mile long processional route constructed at the end of the Stone Age. The outer henge around the stones was built about 2400BC but arrowheads found in the stone circle indicate the stones were put up as much as 500 years earlier.

Parker Pearson said his team was waiting for results of radiocarbon dating
4 __________________  whether stones currently in the inner circle of Stonehenge were originally located at the other riverside construction.

Pearson said: «The big, big question is when these stones were erected and when they were removed – and when we get the dating evidence we can answer both those questions.».

He added: «We speculated in the past 5 ____________________ at the end of the avenue near the river. But we were completely unprepared to discover that there was an entire stone circle. Another team member, Professor Julian Thomas, said the discovery indicated 6  _________________ was central to the religious lives of the people who built Stonehenge. «Old theories about Stonehenge that do not explain the evident significance of the river will have to be rethought,» he said. Dr Josh Pollard, project co-director from the University of Bristol, described the discovery as «incredible».

Заполните пропуски 1 – 6 частями предложeний, обозначенными буквами A – G. Одна из частей в списке А – G лишняя.

A) which could reveal

B) which they stood

C) which it was once made up

D) that this stretch of the river Avon

E) that there might have been something

F) that it should be considered as integral part

G) that leads from the river Avon to Stonehenge

Прочитайте текст и выполните задания А15 – А21. В каждом задании выберите вариант ответа: 1, 2, 3 или 4.

Getting What He Deserved?

There were seven or eight of us in the line, waiting to pay the cashier for our lunches. We were all in a hurry because that’s the way of the American business-day lunch. At the front of the line there was a pretty woman with a small boy of about eight. He was a cute little fellow wearing black jeans, white sneakers and a blue pullover sweater. A shock of dark hair fell over his eyes. He looked very much like his mother. The boy had a charming face with chiseled features but he was depressed.

As the woman fumbled in her purse, looking for money to pay her check, the kid noticed a display of candy bars beside the cash register and immediately wanted one.

“You can’t have any candy”, said his mother. “You had pie with your lunch”. She took out her handkerchief, then put it back and went on fumbling in her purse.

“But I want some candy”, said the kid. His tone was surprisingly insistent. Almost aggressive.

The mother continued her search for money in her purse, and the kid continued to whine about the candy. Then he began to stamp his feet and shout.

           The rest of us in line were beginning to get fidgety. We bunched a little closer together and several folks began mumbling under their breath. “Ought to snatch him bald”, said one man quietly.

The kid by now was reaching for the candy display in open opposition to his mother. She grabbed his arm and pulled it away, but not before he clutched a Snickers bar in his hand.

“Put it back”, she said.

“No!” shouted the child. It was an arrogant “No!”.

The line bunched even more closely together, and the man who had suggested snatching the kid bald appeared ready to do so himself. So much for the kid’s shock of dark hair, I thought.

But the mother moved suddenly and with purpose. She paid the cashier, took back her change and dropped it into her purse. Then with one quick motion, she grabbed hold of the child’s pullover sweater and lifted him off the floor. The moment his sneakers came back to earth, she turned his back toward her and began flailing him. A look of disbelief came across the kid’s face. His eyes filled with tears. He tried to break away but that made his mother flail him again.

When she had finished administering the punishment, she turned the child around and pointed a finger squarely in his sobbing face. With a voice strong and certain, she said, “The next time I tell you do something, young man, will you do it?”

The child looked at the floor. Meekly and sincerely, he replied, “Yes, ma’am.”

The mother turned to go. The child returned the Snickers bar without further hesitation and marched dutifully out behind her.

The people in a line broke into spontaneous applause.

“Did the kid deserve the punishment he had? What would I do if I were his mother? She may have been absolutely right for all I know. I have no children. I have no right to argue with the mother” I thought. “There is nothing I can do but wait. Perhaps the best way to get an idea of normal behavior of children is to get married and raise a few”.

А15. The people in the cafe were all in a hurry because

1) they had to buy their lunch far away from the office.

2) they wanted to be the first in line to pay for the food.

3) it was the way they normally behaved at lunch time.

4) they had to buy their lunch before the cashier left for her lunch.

А16. The woman was fumbling in her purse because she wanted to

1) pay for her food.

2) buy a candy bar.

3) find her phone.

4) pay with a check.

А17. The mother would not let the child buy a candy bar because she

1) didn’t have enough money to pay for it.

2) was unreasonably strict with the little kid.

3) thought that he had enough candy already.

4) didn’t trust the quality of the candy from the display.

А18. The boy was persistent in getting what he wanted and the people in the line

1) supported him.

2) started to show irritation.

3) started to shout at the kid.

4) remained indifferent to the incident.

А19. Judging by the child’s reaction to the punishment we can say that

1) it did not teach him anything.

2) he wasn’t used to being spanked.

3) he was indifferent to being spanked.

4) he wasn’t ready to change his behavior.

А20. The people in a line broke into spontaneous applause because the child

1) was forced to obey.

2) managed to get his way.

3) Had already eaten the candy bar.

4) manipulated his mother skillfully.

А21. Reflecting on the incident the narrator thought that

1) The mother had overreacted.

2) The mother was right in her reaction.

3) he/she wanted his/her own children badly.

4) One should be a parent to have a right to judge.

2. ГРАММАТИКА И ЛЕКСИКА (40 минут)

Прочитайте приведённый ниже текст. Преобразуйте слова, напечатанные заглавными буквами в конце предложений, так чтобы они грамматически соответствовали содержанию текста. Заполните пропуски полученными словами. Каждый пропуск соответствует отдельному заданию из группы B4 – B10.

Where did basketball come from?

B4.

Basketball is now a major sport in the USA. Basketball __________________ by Dr. James Naismith at Springfield, Massachusetts, in 1891. Basketball is a game played between two teams of five players, in which each team tries to win points by throwing a ball through a net.

INVENT

В5.

Basketball is a very spectator-oriented sport. It is the __________________ most popular game in the USA. People enjoy both watching and playing it.

THREE

В6.

There__________________ many basketball centers in cities and towns of every state. They do not often produce sportsmen or sportswomen who are successful in world basketball championships but they help young people to keep fit and look athletic.

BE

В7.

Basketball __________________ by men and women at all levels, from the professional level to high schools and clubs.

PLAY

В8.

One of the __________________  and the most famous basketball teams is the Harlem Globetrotters, formed in 1926. It has a lot of fans in the USA, Canada, Great Britain, Australia, and New Zealand.

OLD

В9.

This team __________________ crowds of fans especially for final or championship matches. One can see photos of its members in newspapers and magazines all over the USA.

ATTRACT

В10.

The name of the team __________________   a symbol of a fast and furious play.

BECOME

Прочитайте приведенный ниже текст. Преобразуйте, если необходимо, слова, напечатанные заглавными буквами в конце предложений, так, чтобы они грамматически и лексически соответствовали содержанию текста. Заполните пропуски полученными словами. Каждый пропуск соответствует отдельному заданию из группы В11 – В16.

The Opening Night

В11.

The Phantom of the Opera, Cats, Mama Mia… They are all stage musicals. The opening night of a new musical is always a  ________________ event.

MARVEL

В12.

After years of __________________ and weeks of rehearsals the production has to be ready for the public and the critics.

PREPARE

В13.

First nights usually start earlier than the regular  __________________ so that the critics can write their reviews in time to include them in the next morning’s newspapers.

PERFORM

В14.

The critics are the most __________________ people on the first night because their opinions will either help make the show a hit or force it to close.

FRIGHT

В15.

The rest of the audience on the first night is usually made up of friends of the cast and famous celebrities. The celebrities attract newspaper __________________ and help give the musical maximum publicity.

PHOTOGRAPH

В16.

There will also be some angels there who will __________________ be more nervous than the performers. And then after the curtain has come down and the show is over, there’s the opening night party.

PROBABLE

Прочитайте текст с пропусками, обозначенными номерами А22–А28. Эти номера соответствуют заданиям A22–A28, в которых представлены возможные варианты ответов. Укажите номер выбранного вами варианта ответа.

The Fruitcake Special 

I was a chemist at the Amos Cosmetics factory in New Jersey, USA, trying to design a new perfume when it happened. I never thought I would discover something quite so amazing by  A22______. Not me. I was only 23 and it was my second year at the factory. I liked my job very much but I was not a lucky person. I had come straight from the university then, but now I was a chemist in one of the biggest factories in New Jersey. It was an important position to have and meant lots of work 

I was A23______ out all the usual mixes of flowers and things- just as I always did — when I decided to throw in a A24______ of the fruitcake Momma had packed for my lunch. I don’t know why I did it –just did it. I put it into the mix with all the other things.

I thought it A25______ nice, but there was nothing special about it, so I put the bottle into my handbag. I couldn’t give something like that to my boss. After all, I was a chemist and my job was to make perfumes in the proper A26______.

If I told him how I made this one he would tell me not to be a silly girl.  Later, he would probably A27______ a joke about it to his friends at the golf club.

“Anna!”

It was my boss, David Amos. He happened to be walking past where I worked. I couldn’t A28______ being nervous. He was fairly sure of his good looks and never spoke to ordinary-looking girls like me. I was thrilled.

В каждом задании выберите один ответ.

А22.

1) incident

2) accident

3) event

4) occasion

А23.

1) producing

2) making

3) trying

4) doing

А24.

1) piece

2) lump

3) bunch

4) pinch

А25.

1) heard

2) touched

3) tasted

4) smelled

А26.

1) way

2) road

3) path

4) method

А27.

1) Do

2) produce

3) make

4) have

А28.

1) hold

2) feel

3) keep

4) help

Ответы:

Раздел 1

Раздел 2

B2

1D,2H,3A,4E,5G,6B,7C

B4

was invented

B3

1C,2B,3G,4A,5E,6D

B5

third

A15

3

B6

are

A16

1

B7

is played

A17

3

B8

oldest

A18

2

B9

attracts

A19

2

B10

has become

A20

1

B11

marvellous

A21

4

B12

preparation

B13

performance

B14

frightful

B15

photographers

B16

probably

А22

2

А23

3

А24

1

А25

4

А26

1

А27

3

А28

4

Скачать весь материал можно здесь.

Структурно-смысловые связи

TASK   1.Про­чи­тай­те текст и за­пол­ни­те про­пус­ки A–F ча­стя­ми пред­ло­же­ний, обо­зна­чен­ны­ми циф­ра­ми 1–7. Одна из ча­стей в спис­ке 1–7 — лиш­няя. За­не­си­те цифры, обо­зна­ча­ю­щие со­от­вет­ству­ю­щие части пред­ло­же­ний, в таб­ли­цу.

Fire Crews Hunt Escaped Hamster

 Eight firefighters have been called in to help find an escaped hamster. Two crews used a chocolate-covered camera and a vacuum cleaner A ____ , called Fudgie, at the home of a six-year-old girl in Dunbar, Scotland.

The girl’s mother said: ‘We came down for breakfast and discovered Fudgie had opened the top lid of her cage and had made her way into the kitchen and we think she has gone В ____ .’

The fire crews spent five hours trying to recover the pet after it ran down a hole in the kitchen floor. But, the hamster still refused С ____ .

In the search for Fudgie, the firefighters took the family cooker and gas pipes apart. They also dropped a mini-camera coated with chocolate under the floorboards. They then hoped to take out the hamster using a vacuum cleaner. Despite all their efforts, they failed to find Fudgie.

In the end, the firefighters put another camera down the hole D ____ , connected to the screen of the family home computer, to see if Fudgie appeared. Besides, the girl and her parents regularly dropped food E ____ .

At last, after eight days the hamster returned to her cage safe and sound. She crawled from the hole in the kitchen floor early in the morning. It was the girl’s father who first found Fudgie F ____ .

The girl said that day it was like Christmas morning for her. Her parents added that they too felt extremely happy when Fudgie had finally returned.

 1. through a small hole in the floor

2. through the hole for the hamster

3. and locked the runaway hamster

4. to come out of the hole

5. to look after the pet

6. to try and locate the missing hamster

7. and left it under the floorboards

Про­пуск

A

B

C

D

E

F

Часть пред­ло­же­ния

           

TASK2.Про­чи­тай­те текст и за­пол­ни­те про­пус­ки A–F ча­стя­ми пред­ло­же­ний, обо­зна­чен­ны­ми циф­ра­ми 1–7. Одна из ча­стей в спис­ке 1–7 — лиш­няя. За­не­си­те цифры, обо­зна­ча­ю­щие со­от­вет­ству­ю­щие части пред­ло­же­ний, в таб­ли­цу.

                                Hi-Tech Brings Families Together

 Technology is helping families stay in touch like never before, says a report carried out in the US.

Instead of driving people apart, mobile phones and the Internet are A ______ . The research looked at the differences in technology use between families with children and single adults. It found that traditional families have more hi-tech gadgets in their home В ______ . Several mobile phones were found in 89% of families and 66% had a high-speed Internet connection. The research also found that 58% of families have more С ______ .

Many people use their mobile phone to keep in touch and communicate with parents and children. Seventy percent of couples, D ______ , use it every day to chat or say hello. In addition, it was found that 42% of parents contact their children via their mobile every day.

The growing use of mobile phones, computers and the Internet means that families no longer gather round the TV to spend time together. 25% of those who took part in the report said they now spend less time E ______ . Only 58% of 18—29 year olds said they watched TV every day. Instead the research found that 52% of Internet users who live with their families go online F ______ several times a week and 51% of parents browse the web with their children.

Some analysts have worried that new technologies hurt families, but we see that technology allows for new kinds of connectedness built around cell phones and the Internet/ said the report.

1. than any other group

2. watching television

3. in the company of someone else

4. than two computers in the home

5. communicated with their families

6. helping them communicate

7. owning a mobile

Про­пуск

A

B

C

D

E

F

Часть пред­ло­же­ния

           
 

TASK3.

Про­чи­тай­те текст и за­пол­ни­те про­пус­ки A–F ча­стя­ми пред­ло­же­ний, обо­зна­чен­ны­ми циф­ра­ми 1–7. Одна из ча­стей в спис­ке 1–7 — лиш­няя. За­не­си­те цифры, обо­зна­ча­ю­щие со­от­вет­ству­ю­щие части пред­ло­же­ний, в таб­ли­цу.

                                    The Power of ‘Hello’

I work at a company where there are hundreds of employees. I know most of them and almost all of them know me. It is all based on one simple principle: I believe every single person deserves to be acknowledged, A ______ .

When I was about 10 years old, I was walking down the street with my mother. She stopped to speak to Mr. Lee. I knew I could see Mr. Lee any time around the neighborhood, В ______ .

After we passed Mr. Lee, my mother said something that has stuck with me from that day until now. She said, ‘You let that be the last time you ever walk by somebody and not open up your mouth to speak, because even a dog can wag its tail С ______ . That phrase sounds simple, but it has been a guidepost for me and the foundation of who I am. I started to see that when I spoke to someone, they spoke back. And that felt good. It is not just something I believe in — D ______ . I believe that every person deserves to feel someone acknowledges their presence, no matter how unimportant they may be.

At work, I always used to say ‘hello’ to the founder of the company and ask him how our business was doing. But I was also speaking to the people in the cafe, and asked how their children were doing. I remembered after a few years of passing by the founder, I had the courage to ask him for a meeting. We had a great talk.

At a certain point, I asked him E ______ . He said, ‘If you want to, you can get all the way to this seat.’ I have become vice president, but that has not changed the way I approach people. I speak to everyone I see, no matter where I am. I have learned that speaking to people creates a pathway into their world, F ______ .

 1. it has become a way of life.

2. when it passes you on the street.

3. when you see him and talk to him.

4. and it lets them come into mine, too.

5. so I did not pay any attention to him.

6. however small or simple the greeting is.

7. how far he thought I could go in his company.

Про­пуск

A

B

C

D

E

F

Часть пред­ло­же­ния

           

TASK4. Про­чи­тай­те текст и за­пол­ни­те про­пус­ки A–F ча­стя­ми пред­ло­же­ний, обо­зна­чен­ны­ми циф­ра­ми 1–7. Одна из ча­стей в спис­ке 1–7 — лиш­няя. За­не­си­те цифры, обо­зна­ча­ю­щие со­от­вет­ству­ю­щие части пред­ло­же­ний, в таб­ли­цу.

                                                 Friendship and Love

 A strong friendship takes a significant amount of time to develop. It will not just magically mature overnight. A friendship involves committing oneself to help another person A ______ . I believe that, nothing can replace a true friend, not material objects, or money, and definitely not a boy.

I met this guy a couple summers ago who I ended up spending almost all of my free time with. His parents did not approve of our dating because of our age difference, В ______ . He had told me the day we met that he had joined the air force and would leave for overseas that coming October. After three months had past, the time came when he had to leave. This left me feeling completely alone.

I turned to my friends for support, but to my surprise, С ______ . I had spent so much time with this guy and so little time with them, that they did not feel sorry for me when he left. For so long they had become the only constant in my life, and I had taken them for granted over something D ______ .

When my boyfriend came back, our relationship changed. I tried to fix all the aspects in my life that had gone so wrong in the previous six months.

This experience taught me that true friendships will only survive if one puts forth effort to make them last. Keeping friends close will guarantee that E ______ . When a relationship falls apart, a friend will always do everything in their power to make everything less painful. As for me, I try to keep my friends as close as I can. I know they will always support me in whatever I do, and to them, I F ______ .

 1. but we did anyway.

2. whenever a need arises.

3. they did not really care.

4. whenever they need your help.

5. could not guarantee would even last.

6. am eternally grateful’for a second chance.

7. someone will always have a shoulder to cry on.

Про­пуск

A

B

C

D

E

F

Часть пред­ло­же­ния

           

TASK 5.Про­чи­тай­те текст и за­пол­ни­те про­пус­ки A–F ча­стя­ми пред­ло­же­ний, обо­зна­чен­ны­ми циф­ра­ми 1–7. Одна из ча­стей в спис­ке 1–7 — лиш­няя. За­не­си­те цифры, обо­зна­ча­ю­щие со­от­вет­ству­ю­щие части пред­ло­же­ний, в таб­ли­цу.

                                                                 Culture and customs

 In less than twenty years, the mobile telephone has gone from being rare, expensive equipment of the business elite to a pervasive, low-cost personal item. In many countries, mobile telephones A ______ ; in the U.S., 50 per cent of children have mobile telephones. In many young adults’ households it has supplanted the land-line telephone. The mobile phone is В ______ , such as North Korea.

Paul Levinson in his 2004 book Cellphone argues that by looking back through history we can find many precursors to the idea of people simultaneously walking and talking on a mobile phone. Mobile phones are the next extension in portable media, that now can be С ______ into one device. Levinson highlights that as the only mammal to use only two out of our four limbs to walk, we are left two hands free D ______ — like talking on a mobile phone.

Levinson writes that “Intelligence and inventiveness, applied to our need to communicate regardless of where we may be, led logically and eventually to telephones that we E ______ .”

Given the high levels of societal mobile telephone service penetration, it is a key means for people F ______ . The SMS feature spawned the «texting» sub-culture. In December 1993, the first person-to-person SMS text message was transmitted in Finland. Currently, texting is the most widely-used data service; 1.8 billion users generated $80 billion of revenue in 2006.

1. to perform other actions

2. outnumber traditional telephones

3. to communicate with each other 

4. combined with the Internet

5. to serve basic needs

6. banned in some countries

7. carry in our pockets

Про­пуск

A

B

C

D

E

F

Часть пред­ло­же­ния

           

ANSWERS

По­яс­не­ние 1.

А−6: missing hamster(по смыс­лу и грам­ма­ти­че­ски to try and locate).

B−1: through a small hole in the floor (по смыс­лу).

C−4: refused to come out (смысл).

D−7: and left it (по смыс­лу.

E−2: dropped food… for the humster.

F−3: found and locked…

По­яс­не­ние 2.

A−6: по­мо­га­ют им об­щать­ся. are helping (Present Cont.) + по смыс­лу речь идет о сред­ствах связи.

B−1: срав­не­ние, чем…(than any other group).

C−4: more than two computers. (опять срав­не­ние — более чем 2 ком­пью­те­ра…)

D−7: вла­де­ю­щих мо­биль­ным те­ле­фо­ном, стоит за­пя­тая, под­ска­зы­вая Ving + по смыс­лу, (чтобы ска­зать Hello и по­бол­тать) — owning a mobile.

E−2: По смыс­лу, речь в этом па­ра­гра­фе идет о те­ле­ви­зо­ре.

F−3: вы­хо­дят в ин­тер­нет с кем-либо еще. (по смыс­лу).

По­яс­не­ние 3.

A−6: по смыс­лу.

B−5: Mr Lee… didn’t pay attention to him.

C−2: the dog… it passes you.

D−1: it… it has become.

E−7: по смыс­лу.

F−4: a pathway… it lets them…

По­яс­не­ние 4.

A−2: по пе­ре­во­ду (когда не­об­хо­ди­мость…).

B−1: его ро­ди­те­ли не одоб­ря­ют, но мы… (parents didn’t approve… but we did…).

C−3: по пе­ре­во­ду… но к моему удив­ле­нию им без­раз­лич­но.

D−5: по пе­ре­во­ду.

E−7: по пе­ре­во­ду.

F−6: за­вер­ша­ю­щая фраза.

По­яс­не­ние 5.

A−2: по пе­ре­во­ду.

B−6: по пе­ре­во­ду и грам­ма­ти­че­ски (is banned) за­пре­щен.

C−4: по пе­ре­во­ду (может быть ском­би­ни­ро­ва­но с Ин­тер­не­том в одном при­бо­ре).

D−1: по струк­ту­ре и по смыс­лу — to be free to do smth (оста­ют­ся 2 сво­бод­ных руки, чтобы про­из­во­дить дру­гие дей­ствия).

E−7: ко­то­рые мы можем но­сить в своем кар­ма­не (we can carry…).

F−3: по пе­ре­во­ду.

«Личное письмо»                 ЕГЭ.

TASK 1.       

You have received a letter from your English-speaking pen friend Bill who writes

       …I don’t think it will be a problem for me to choose a good job in the future as I’m really interested in foreign languages, cultures and countries and I hope I’ll work as a translator or teacher of foreign languages some day. Have you already decided on your career? What job are you going to choose? Why?

I’ve lived in the USA my whole life but I’d really love to travel to other countries…

      Write a letter to Bill. In your letter answer his questions, ask 3 questions about his plans for travelling. Write 100−140 words. Remember the rules of letter writing.

TASK 2.       

You have received a letter from your English-speaking pen-friend Mary who writes:

…Summer is coming and I want to look my best: healthy, energetic and phyically fit. So I’m trying to eat plain, simply cooked natural food, have enough sleep at night and I have recently joined our local fitness club. Do you do anything special to stay healthy? What makes people healthy and strong? What ao uou think about a healthy lifestule?

By the way, I m going to spend a month at the seaside this summer…

 Write a letter to Mary. In your letter answer her questions, ask 3 questions about her coming summer holidays. Write 100—140 words. Remember the rules of letter writing. You have 20 minutes to do this task.

TASK 3.       

You have received a letter from your New Zealand pen-friend Sheila who writes:

    …You know, my idea of a perfect weekend is to do absolutely nothing, just read a book. A walk in the park is not bad if the weather is nice. I don’t understand people who spend Sunday in a gym or a fitness centre.

What is your idea of a perfect weekend? What do you like to do in your free time after the lessons? What is your hobby?.

Soon I’m going on vacation and I can’t decide what books to take with me…

    Write a letter to Sheila. In your letter ask his questions, ask 3 questions about her favourite books. Write 100—140 words. Remember the rules of letter writing. You have 20 minutes to do this task.

TASK 4.     

  You have received a letter from your New Zealand pen-friend Tom who writes:

   …Last month our class went to Washington to visit the National Museum of American History. It was my first visit there and it was fun! How often do you go to museums with your class, if at all? Which museum is your favourite or what museum would you like to visit? Why do you think people should go there?

This summer we plan to go hiking with my parents…

 Write a letter to Tom. In your letter answer his questions, ask 3 questions about his summer plans. Write 100—140 words. Remember the rules of letter writing. You have 20 minutes to do this task.

TASK 5.     

You have received a letter from your English-speaking pen-friend Jane who writes:

… Yesterday my Mum asked me to help her about the house. We were very busy with cleaning up after the birthday party the whole morning. I got quite tired and even missed my fitness class. What are your family duties, if any? Is there anything you especially like or dislike about house work? Do you find helping your parents necessary, why or why not?

Oh, I have some great news! I got a lovely kitten for my birthday…

   Write a letter to Jane. In your letter answer her questions, ask 3 questions about her kitten. Write 100—140 words. Remember the rules of letter writing. You have 20 minutes to do this task.

TASK 6.     

You have received a letter from your English-speaking pen-friend Alice who writes:

 … I have to think about my pocket money all the time. My parents try to make me earn it by doing the housework. They try to make me save up and then buy something useful. But I need pocket money for the cinema and ice-cream and other treats like that. Do you have to ‘earn’ your pocket money in any way or do your parents just give it to you? What do you spend your pocket money on? What do your parents think of the way you spend it?

Last weekend my friends and I went on a picnic to the countryside…

   Write a letter to Alice. In your letter answer her questions, ask 3 questions about her picnic. Write 100—140 words. Remember the rules of letter writing. You have 20 minutes to do this task.

TASK 7.     

You have received a letter from your English-speaking pen-friend Jane who writes:

… So you see that I enjoy films based on true historic facts. What kinds of films do you like watching? Do you prefer watching films in the cinema or at home? Why? Do you agree that it is better to read a book before watching the film based on it? Why or why not?

I’ve just come back from a trip to Wales…

Write a letter to Jane. In your letter answer her questions, ask 3 questions about her trip to Wales. Write 100—140 words. Remember the rules of letter writing. You have 20 minutes to do this task.

TASK 8.     

You have received a letter from your English-speaking pen-friend Nick who writes:

…I’m, going to do a project on reading in different countries. Could you help me? Do young people read as much as old people in your country? Do you prefer to read E-books or traditional books? Why? How much time do you and your friends spend reading daily? As for the latest news, I have just joined a sport club…

   Write a letter to Nick. In your letter answer his questions, ask 3 questions about sports. Write 100–140 words. Remember the rules of letter writing.

Example (task 1):

                                                                                                                Moscow, Russia

                                                                                                                         October, 21

Dear Bill,

Thank you for your letter. It’s great that you have already chosen your future job!

Talking about me — I’m not that sure as you are. I also like foreign languages, different cultures and I enjoy travelling, but I don’t think that translation or being a professor is something for me. I’d like to choose a job which would give me a possibility of traveling, meeting new people, communicate with them on a foreign language, because that’s what I’m interested in and what, I guess I could do successfully.

Anyway, you’ve mentioned travelling. Have you already decided when you are going to have your trip? What countries would you like to visit? Do you want to take your parents with you?

I’ve got to go as my friend is asking me for help.

Write back soon.

Best wishes,

Kate.

Автор: Кожемяка А.В.

Скачать весь материал.

Установите соответствие тем 1 — 8 текстам A — G. Занесите свои ответы в соответствующее поле справа. Используйте каждую цифру только один раз. В задании одна тема лишняя.

1. Comparing insects

2. Some special features

3. Some dragonflies migrate

4. Excellent vision

5. Keeping them warm or cool

6. Masters of flight

7. Ancient insects

8. Risks of extinction

A. Long before the dinosaurs walked the Earth, dragonflies took to the air. If we could transport ourselves back 250 million years, we would immediately recognize the familiar sight of dragonflies flying in pursuit of prey. Ancient dragonflies may have been considerably larger than those we see today. A fossilized impression of a dragonfly wing, found in a coal mine in England, is the oldest known dragonfly specimen. This dragonfly lived 320 million years ago and had a wingspan of 8 inches.

B. Relative to other insects, dragonfly vision is extraordinarily good. The head consists almost entirely of two huge compound eyes, which gives the dragonfly nearly 360° vision. Each compound eye contains as many as 30,000 lenses, or ommatidia. A dragonfly uses about 80% of its brain to process all this visual information. They can see a wider spectrum of colours than humans. This remarkable vision helps them to detect the movement of other insects and avoid collisions in flight.

C. Dragonflies can move each of their four wings independently. In addition to flapping each wing up and down, they can rotate their wings forward and back on”an axis. This flexibility enables them to put on an aerial show like no other insect. Dragonflies can move straight up or down, fly backwards, stop and hover, and make hairpin turns, at full speed or in slow motion. A dragonfly can fly forward at a speed of 100 body lengths per second, or up to 30 miles per hour.

D. A number of dragonfly species are known to migrate. As with other organisms that migrate, dragonflies relocate to follow or find needed resources, or in response to environmental changes like cold weather. The globe skimmer is one of several species known to develop in temporary freshwater pools. Forced to follow the rains that replenish their breeding sites, the globe skimmer set a new insect world record when a biologist documented its 11,000 mile trip between India and Africa.

E. Fifty years ago there were twice as many ponds in Britain as there are today. The draining of agricultural land, «filling in and pollution have all contributed to the disappearance of most countryside ponds. Canals have also suffered from pollution, especially by chemicals used on farmland draining into water. The loss of suitable fresh water habitats has affected dragonflies enormously and they are becoming increasingly rare. Some of dragonflies which can be found living only in the Norfolk Broads, is on the list of British endangered species of insects.

F. A dragonfly has two large compound eyes which take up most of its head. Dragonflies have long, delicate, membranous wings which are transparent and some have light yellow colouring near the tips. Their bodies are long and slender and they have short antennae. Dragonflies are very colourful. Some are red like the Comet Darner and yellow like the Emerald Darner. They can beat each pairs of wings together or separately and their rear wings can be out of phase with the front wings. Their wing beat is around 50-90 beats per second.

G. Dragonflies and butterflies possess two pair of wings. The butterfly’s wings are made up of two large pairs of wings each possessing a forewing and a hindwing. Butterflies can’t fly if the temperature of their body falls below 8″5 degrees and therefore need to sun themselves in order to warm up. Butterflies also have a pair of antennae with small receptors attached for smelling. Dragonflies have two pairs of wings that are transparent, rigid, straight, and have few veins. Unlike butterflies, dragonflies are adept at flying.

Английский язык (Вариант 4)

  • 1
  • 2
  • 3
  • 4

Вы услышите рассказ. В заданиях 3—9 впишите цифру 1, 2 или 3, соответствующую вы­бранному вами варианту ответа в поле ответа ниже. Вы услышите запись дважды.

3.The girl’s grandmother goes by the name of Elaine because

1) it is only natural to go by one’s first name for a girl.

2) she didn’t like her real first name and changed it when she grew up.

3) she wanted to avoid confusion with other female members in the family.

4.Elaine liked primary school because

1) she could be alone at least on the way to school.

2) her teacher always made her feel special.

3) she could learn more, listening to older kids.

5.Among Elaine’s chores was

1) waxing the floors.

2) tidying the closets.

3) washing the dishes.

6.When in high school Elaine

1) proved to be a natural dancer.

2) couldn’t learn to play basketball.

3) could take athletics with boys.

7.She got a chance to go to college because

1) her employer helped her to continue her education.

2) she earned enough money working at the creamery and at the cafeteria.

3) she got an academic scholarship.

8.She majored in

1) economics.

2) home economics education.

3) education.

9.Elaine had a big family, and remembering her kids growing up

1) she feels sorry she had so many children.

2) she wishes she had more.

3) she believes it was a reward for all her previous problems.

Заметили ошибку в тексте?
Выделите её и нажмите Ctrl + Enter

Task 1 Прочитайте текст и заполните пропуски A–F частями предложений, обозначенными цифрами 1–7. Одна из частей в списке 1–7 лишняя. Занесите цифру, обозначающую соответствующую часть предложения, в таблицу.

Fire Crews Hunt Escaped Hamster

Eight firefighters have been called in to help find an escaped hamster. Two crews used a chocolate-covered camera and a vacuum cleaner A ____ , called Fudgie, at the home of a six-year-old girl in Dunbar, Scotland.

The girl’s mother said: ‘We came down for breakfast and discovered Fudgie had opened the top lid of her cage and had made her way into the kitchen and we think she has gone В ____ .’

The fire crews spent five hours trying to recover the pet after it ran down a hole in the kitchen floor. But, the hamster still refused С ____ .

In the search for Fudgie, the firefighters took the family cooker and gas pipes apart. They also dropped a mini-camera coated with chocolate under the floorboards. They then hoped to take out the hamster using a vacuum cleaner. Despite all their efforts, they failed to find Fudgie.

In the end, the firefighters put another camera down the hole D ____ , connected to the screen of the family home computer, to see if Fudgie appeared. Besides, the girl and her parents regularly dropped food E ____ .

At last, after eight days the hamster returned to her cage safe and sound. She crawled from the hole in the kitchen floor early in the morning. It was the girl’s father who first found Fudgie F ____ .

The girl said that day it was like Christmas morning for her. Her parents added that they too felt extremely happy when Fudgie had finally returned.

  1. through a small hole in the floor

  2. through the hole for the hamster

  3. and locked the runaway hamster

  4. to come out of the hole

  5. to look after the pet

  6. to try and locate the missing hamster

  7. and left it under the floorboards

Ответ

A

B

C

D

E

F

6

1

4

7

2

3

Task 2

Speed of eating is ‘key to obesity’

If you eat very quickly, it may be enough to increase your risk of being overweight, research suggests.

Osaka University scientists looked at the eating habits of 3,000 people. Just about half of them told researchers that they A ______ . Compared with those who did not eat quickly, fast-eating men were 84% more likely to be overweight, and women were 100% more likely to В ______ .

Japanese scientists said that there were a number of reasons why eating fast С ______ . They said it could prevent the work of a signalling system which tells your brain to stop eating because your stomach is full. They said: ‘If you eat quickly you basically fill your stomach before the system has a chance to react, so you D _____ .

The researchers also explained that a mechanism that helps make us fat today, developed with evolution and helped people get more food in the periods when they were short of it. The scientists added that the habit of eating fast could be received from one’s parents genes or E ______ .

They said that, if possible, children should be taught to F ______ , and allowed to stop when they felt full up at mealtimes. ‘The advice of our grandmothers about chewing everything 20 times might be true — if you take a bit more time eating, it could have a positive influence on your weight.

  1. just overfill your stomach

  2. could be bad for your weight

  3. have a habit of eating quickly

  4. linked to obesity

  5. eat as slowly as possible

  6. put on weight

  7. learned at a very early age

Ответ

A

B

C

D

E

F

3

6

2

1

7

5

Task 3

Hi-Tech Brings Families Together 

Technology is helping families stay in touch like never before, says a report carried out in the US.

Instead of driving people apart, mobile phones and the Internet are A ____ . The research looked at the differences in technology use between families with children and single adults. It found that traditional families have more hi-tech gadgets in their home В ____ . Several mobile phones were found in 89% of families and 66% had a high-speed Internet connection. The research also found that 58% of families have more С ____ .

Many people use their mobile phone to keep in touch and communicate with parents and children. Seventy percent of couples, D ____ , use it every day to chat or say hello. In addition, it was found that 42% of parents contact their children via their mobile every day.

The growing use of mobile phones, computers and the Internet means that families no longer gather round the TV to spend time together. 25% of those who took part in the report said they now spend less time E ____ . Only 58% of 18—29 year olds said they watched TV every day. Instead the research found that 52% of Internet users who live with their families go online F ____ several times a week and 51% of parents browse the web with their children.

Some analysts have worried that new technologies hurt families, but we see that technology allows for new kinds of connectedness built around cell phones and the Internet/ said the report.

  1. than any other group

  2. watching television

  3. in the company of someone else

  4. than two computers in the home

  5. communicated with their families

  6. helping them communicate

  7. owning a mobile

Ответ

A

B

C

D

E

F

6

1

4

7

2

3

Task 4

The Power of ‘Hello’ 

I work at a company where there are hundreds of employees. I know most of them and almost all of them know me. It is all based on one simple principle: I believe every single person deserves to be acknowledged, A ______ .

When I was about 10 years old, I was walking down the street with my mother. She stopped to speak to Mr. Lee. I knew I could see Mr. Lee any time around the neighborhood, В ______ .

After we passed Mr. Lee, my mother said something that has stuck with me from that day until now. She said, ‘You let that be the last time you ever walk by somebody and not open up your mouth to speak, because even a dog can wag its tail С______ . That phrase sounds simple, but it has been a guidepost for me and the foundation of who I am. I started to see that when I spoke to someone, they spoke back. And that felt good. It is not just something I believe in — D ______ . I believe that every person deserves to feel someone acknowledges their presence, no matter how unimportant they may be.

At work, I always used to say ‘hello’ to the founder of the company and ask him how our business was doing. But I was also speaking to the people in the cafe, and asked how their children were doing. I remembered after a few years of passing by the founder, I had the courage to ask him for a meeting. We had a great talk.

At a certain point, I asked him E ______ . He said, ‘If you want to, you can get all the way to this seat.’ I have become vice president, but that has not changed the way I approach people. I speak to everyone I see, no matter where I am. I have learned that speaking to people creates a pathway into their world, F ______ .

  1. it has become a way of life.

  2. when it passes you on the street.

  3. when you see him and talk to him.

  4. and it lets them come into mine, too.

  5. so I did not pay any attention to him.

  6. however small or simple the greeting is.

  7. how far he thought I could go in his company.

Ответ

A

B

C

D

E

F

6

5

2

1

7

4

Task 5

Friendship and Love

 A strong friendship takes a significant amount of time to develop. It will not just magically mature overnight. A friendship involves committing oneself to help another person A ______ . I believe that, nothing can replace a true friend, not material objects, or money, and definitely not a boy.

I met this guy a couple summers ago who I ended up spending almost all of my free time with. His parents did not approve of our dating because of our age difference, В ______ . He had told me the day we met that he had joined the air force and would leave for overseas that coming October. After three months had past, the time came when he had to leave. This left me feeling completely alone.

I turned to my friends for support, but to my surprise, С ______ . I had spent so much time with this guy and so little time with them, that they did not feel sorry for me when he left. For so long they had become the only constant in my life, and I had taken them for granted over something D ______ .

When my boyfriend came back, our relationship changed. I tried to fix all the aspects in my life that had gone so wrong in the previous six months.

This experience taught me that true friendships will only survive if one puts forth effort to make them last. Keeping friends close will guarantee that E ______ . When a relationship falls apart, a friend will always do everything in their power to make everything less painful. As for me, I try to keep my friends as close as I can. I know they will always support me in whatever I do, and to them, I F ______ .

  1. but we did anyway.

  2. whenever a need arises.

  3. they did not really care.

  4. whenever they need your help.

  5. could not guarantee would even last.

  6. am eternally grateful for a second chance.

  7. someone will always have a shoulder to cry on.

Ответ

A

B

C

D

E

F

2

1

3

5

7

6

Task 6

Mobile phones

 On New Year’s Day, 1985, Michael Harrison phoned his father, Sir Ernest, to wish him a happy new year. Sir Ernest was chairman of Racal Electronics, the owner of Vodafone, A ______ .

At the time, mobile phones weighed almost a kilogram, cost several thousand pounds and provided only 20 minutes talktime. The networks themselves were small; Vodafone had just a dozen masts covering London. Nobody had any idea of the huge potential of wireless communication and the dramatic impact В ______ .

Hardly anyone believed there would come a day when mobile phones were so popular С ______ .But in 1999 one mobile phone was sold in the UK every four seconds, and by 2004 there were more mobile phones in the UK than people. The boom was a result of increased competition which pushed prices lower and created innovations in the way that mobiles were sold.

When the government introduced more competition, companies started cutting prices to attract more customers. Cellnet, for example, changed its prices, D ______ . It also introduced local call tariffs.

The way that handsets themselves were marketed was also changing and it was Finland’s Nokia who made E ______ . In the late 1990s Nokia realized that the mobile phone was a fashion item: so it offered interchangeable covers which allowed you to customize and personalize your handset.

The mobile phone industry has spent the later part of the past decade reducing its monthly charge F ______ , which has culminated in the fight between the iPhone and a succession of touch screen rivals.

  1. trying to persuade people to do more with their phones than just call and text

  2. that there would be more phones in the UK than there are people

  3. and relying instead on actual call charges

  4. that mobile phones would have over the next quarter century

  5. the leap from phones as technology to phones as fashion items

  6. and his son was making the first-ever mobile phone call in the UK

  7. the move to digital technology, connecting machines to wireless networks

Ответ

A

B

C

D

E

F

6

4

2

1

5

3

Task 7

London Zoo

 London Zoo is one of the most important zoos in the world. There are over 12,000 animals at London Zoo and A ______ ! Its main concern is to breed threatened animals in captivity. This means we might be able to restock the wild, should disaster ever befall the wild population.

Partula Snail, Red Crowned Crane, Arabian Oryx, Golden Lion Tamarin, Persian Leopard, Asiatic Lion and Sumatran Tiger are just some of the species London Zoo is helping to save.

That is why it is so important that we fight to preserve the habitats that these animals live in, as well as eliminate other dangers В ______ . But we aim to make your day at London Zoo a fun and memorable time, С ______ .

In the Ambika Paul Children’s Zoo, for instance, youngsters can learn a new love and appreciation for animals D ______ . They can also learn how to care for favourite pets in the Pet Care Centre.

Then there are numerous special Highlight events E ______ unforgettable pony rides to feeding times and spectacular animal displays. You will get to meet keepers and ask them what you are interested in about the animals they care for, F ______ .

Whatever you decide, you will have a great day. We have left no stone unturned to make sure you do!

  1. such as hunting exotic animals and selling furs

  2. as well as the ins and outs of being a keeper at London Zoo

  3. which take place every day, from

  4. because they see and touch them close up

  5. despite the serious side to our work

  6. which demand much time and effort

  7. that is not counting every ant in the colony

Ответ

A

B

C

D

E

F

7

1

5

4

3

2

Task 8

‘Second Stonehenge’ discovered near original

 Archaeologists have discovered evidence of what they believe was a second Stonehenge located a little more than a mile away from the world-famous prehistoric monument.

The new find on the west bank of the river Avon has been called «Bluestonehenge», after the colour of the 25 Welsh stones of A______.

Excavations at the site have suggested there was once a stone circle 10 metres in diameter and surrounded by a henge — a ditch with an external bank, according to the project director, Professor Mike Parker Pearson, of the University of Sheffield.

The stones at the site were removed thousands of years ago but the sizes of the holes in B ______ indicate that this was a circle of bluestones, brought from the Preseli mountains of Wales, 150 miles away.

The standing stones marked the end of the avenue C _____, a 1¾-mile long processional route constructed at the end of the Stone Age. The outer henge around the stones was built about 2400BC but arrowheads found in the stone circle indicate the stones were put up as much as 500 years earlier.

Parker Pearson said his team was waiting for results of radiocarbon dating D _____ whether stones currently in the inner circle of Stonehenge were originally located at the other riverside construction.

Pearson said: «The big, big question is when these stones were erected and when they were removed — and when we get the dating evidence we can answer both those questions.»

He added: «We speculated in the past E ______ at the end of the avenue near the river. But we were completely unprepared to discover that there was an entire stone circle. Another team member, Professor Julian Thomas, said the discovery indicated F______was central to the religious lives of the people who built Stonehenge. «Old theories about Stonehenge that do not explain the evident significance of the river will have to be rethought,» he said. Dr Josh Pollard, project co-director from the University of Bristol, described the discovery as «incredible».

  1. which could reveal

  2. which they stood

  3. which it was once made up

  4. that this stretch of the river Avon

  5. that there might have been something

  6. that it should be considered as integral part

  7. that leads from the river Avon to Stonehenge

Ответ

A

B

C

D

E

F

3

2

7

1

5

4

Task 9

Australia

 Australia was the last great landmass to be discovered by the Europeans. The continent they eventually discovered had already been inhabited for tens of thousands of years.

Australia is an island continent A _____ is the result of gradual changes wrought over millions of years.

B ____, Australia is one of the most stable land masses, and for about 100 million years has been free of the forces that have given rise to huge mountain ranges elsewhere.

From the east coast a narrow, fertile strip merges into the greatly eroded Great Dividing Range, C ____.

The mountains are merely reminders of the mighty range, D ____. Only in the section straddling the New South Wales border with Victoria and in Tasmania, are they high enough to have winter snow.

West of the range of the country becomes increasingly flat and dry. The endless flatness is broken only by salt lakes, occasional mysterious protuberances and some mountains E ____. In places the scant vegetation is sufficient to allow some grazing. However, much of the Australian outback is a barren land of harsh stone deserts and dry lakes.

The extreme north of Australia, the Top End, is a tropical area within the monsoon belt. F ____, it comes in more or less one short, sharp burst. This has prevented the Top End from becoming seriously productive area.

  1. that once stood here

  2. that is almost continent long

  3. whose property is situated to the north of Tasmania

  4. whose landscape — much of bleak and inhospitable

  5. whose beauty reminds of the MacDonald Ranges

  6. Although its annual rainfall looks adequate on paper

  7. Although there is still seismic activity in the eastern highland area

Ответ

A

B

C

D

E

F

4

7

2

1

5

6

Task 10

Scotland Yard

 Scotland Yard is the headquarters of the Metropolitan Police in London. To most people, its name immediately brings to mind the picture of a detective — cool, efficient, ready to track down any criminal, or a helmeted police constable — A____ and trusty helper of every traveller from overseas.

Scotland Yard is situated on the Thames Embankment close to the Houses of Parliament and the familiar clock tower of Big Ben, and its jurisdiction extends over 740 square miles with the exception of the ancient City of London, B____.

One of the most successful developments in Scotland Yard’s crime detection and emergency service has been the “999 system”. On receipt of a call the 999 Room operator ascertains by electronic device the position of the nearest available police car, C ____. Almost instantly a message is also sent by teleprinter to the police station concerned so that within seconds of a call for assistance being received, a police car is on its way to the scene. An old-established section of the Metropolitan police is the Mounted Branch, with its strength of about 200 horses stabled at strategic points. These horses are particularly suited to ceremonial occasions, D ____.

An interesting branch of Scotland Yard is the branch of Police Dogs, first used as an experiment in 1939. Now these dogs are an important part of the Force. One dog, for example, can search a warehouse in ten minutes, E ____.

There is also the River Police, or Thames Division, which deals with all crimes occurring within its river boundaries.

There are two other departments of Scotland Yard – the Witness Room (known as the Rogues’ Gallery) where a photographic record of known and suspected criminals is kept, and the Museum, F ____.

  1. which is contacted by radio

  2. that familiar figure of the London scene

  3. for they are accustomed to military bands

  4. which possesses its own separate police force

  5. which contains murder relics and forgery exhibits

  6. that this policeman will bring the criminal to justice

  7. whereas the same search would take six men an hour

Ответ

A

B

C

D

E

F

2

4

1

3

7

5

Task 11

Harry Potter course for university students 

Students of Durham University are being given the chance to sign up to what is thought to be the UK’s first course focusing on the world of Harry Potter. Although every English-speaking person in the world knows about Harry Potter books and films, few have thought of using them as a guide to … modern life.

The Durham University module uses the works of JK Rowling A ______ modern society. “Harry Potter and the Age of Illusion” will be available for study next year. So far about 80 undergraduates have signed В ______ a BA degree in Education Studies. Future educationalists will analyse JK Rowling’s fanfiction from various points of view.

A university spokesman said: “This module places the Harry Potter novels in a wider social and cultural context.” He added that a number of themes would be explored, С ______ the classroom, bullying, friendship and solidarity and the ideals of and good citizenship.

The module was created by the head of the Department of Education at Durham University. He said the idea for the new module had appeared in response D ______ body: “It seeks to place the series in its wider social and cultural context and will explore some fundamental issues E ______ . You just need to read the academic writing which started F ______ that Harry Potter is worthy of serious study.”

  1. up for the optional module, part of

  2. to emerge four or five years ago to see

  3. to examine prejudice, citizenship and bullying in

  4. such as the response of the writer

  5. including the world of rituals, prejudice and intolerance in

  6. to growing demand from the student

  7. such as the moral universe of the school

Ответ

A

B

C

D

E

F

3

1

5

6

7

2

Task 12

Laughing and evolution

 The first hoots of laughter from an ancient ancestor of humans could be heard at least 10 million years ago, according to the results of a new study. Researchers used recordings of apes and babies being tickled A ______ to the last common ancestor that humans shared with the modern great apes, which include chimpanzees, gorillas and orangutans.

The finding challenges the opinion В ______ , suggesting instead that it emerged long before humans split from the evolutionary path that led to our primate cousins, between 10m and 16m years ago.

“In humans, laughing can be the strongest way of expressing how much we are enjoying ourselves, but it can also be used in other contexts, like making fun of someone,” said Marina Davila Ross, a psychologist at Portsmouth University. “I was interested in С ______ .”

Davila Ross travelled to seven zoos around Europe and visited a wildlife reserve in Sabah, Borneo, to record baby and juvenile apes D ______ . Great apes are known to make noises that are similar to laughter when they are excited and while they are playing with each other.

Davila Ross collected recordings of laughter from 21 chimps, gorillas, orangutans and bonobos and added recordings of three babies that were tickled to make them laugh.

To analyze the recordings, the team put them into a computer program. “Our evolutionary tree based on these acoustic recordings alone showed E ______ , but furthest from orangutans, with gorillas somewhere in the middle.” said Davila Ross. “What this shows is strong evidence to suggest F ______ .”

  1. whether laughing emerged earlier on than humans did

  2. to create the evolutionary tree linking humans and apes

  3. that laughter is a uniquely human trait

  4. that humans were closest to chimps and bonobos

  5. that laughing comes from a common primate ancestor

  6. while their caretakers tickled them

  7. to trace the origin of laughter back

Ответ

A

B

C

D

E

F

7

3

1

6

4

5

Task 13

Nenets culture affected by global warming

 For 1,000 years the indigenous Nenets people have migrated along the 450-mile- long Yamal peninsula in northern Russia. In summer they wander northwards, taking their reindeer with them. In winter they return southwards.

But this remote region of north-west Siberia is now being affected by global warming. Traditionally the Nenets travel across the frozen River Ob in November A ___ around Nadym. These days, though, this annual winter migration is delayed. Last year the Nenets, together with many thousands of reindeer, had to wait until late December В ____ .

“Our reindeer were hungry. There wasn’t enough food,” Jakov Japtik, a Nenets reindeer herder, said. “The snow is melting sooner, quicker and faster than before. In spring it’s difficult for the reindeer to pull the sledges. They get tired,” Japtik said.

Herders say that the peninsula’s weather is increasingly unpredictable — with unseasonal snowstorms  С ___, and milder longer autumns. In winter, temperatures used to go down to -50°C. Now they are normally around -30°C, according to Japtik. “Obviously we prefer -30°C. But the changes aren’t good for the reindeer D ___,” he said, setting off on his sledge to round up his reindeer herd.

Even here, in one of the most remote parts of the planet, E __ . Last year the Nenets arrived at a regular summer camping spot and discovered that half of their lake had disappeared. The water had drained away after a landslide. The Nenets report other curious changes — there are fewer mosquitoes and a strange increase in flies. Scientists say there is unmistakable evidence F ___ .

  1. when the ice was finally thick enough to cross

  2. that the impact on Russia would be disastrous

  3. the environment is under pressure

  4. and in the end what is good for the reindeer is good for us

  5. and set up their camps in the southern forests

  6. that Yamal’s ancient permafrost is melting

  7. when the reindeer give birth in May

Ответ

A

B

C

D

E

F

5

1

7

4

3

6

Task 14

Duration of life and its social implications

The world’s population is about to reach a landmark of huge social and economic importance, when the proportion of the global population over 65 outnumbers children under 5 for the first time. A new report by the US census bureau shows A____ , with enormous consequences for both rich and poor nations.

The rate of growth will shoot up in the next couple of years. The В ___ a combination of the high birth rates after the Second World War and more recent improvements in health that are bringing down death rates at older ages. Separate UN forecasts predict that the global population will be more than nine billion by 2050.

The US census bureau was the first to sound the С ___ . Its latest forecasts warn governments and international bodies that this change in population structure will bring widespread challenges at every level of human organization, starting with the structure of the family, which will be transformed as people live longer. This will in turn place new burdens on careers and social services providers, D ___ for health services and pensions systems.

“People are living longer and, in some parts of the world, healthier lives,” the authors conclude. “This represents one of the greatest achievements of the last century but also a significant challenge E ___ population.”

Ageing will put pressure on societies at all levels. One way of measuring that is to look at the older dependency ratio, F ___ that must be supported by them. The ODR is the number of people aged 65 and over for every 100 people aged 20 to 64. It varies widely, from just six in Kenya to 33 in Italy and Japan. The UK has an ODR of 26, and the US has 21.

  1. which recently replaced Italy as the world’s oldest major country

  2. alarm about these changes

  3. a huge shift towards an ageing population

  4. change is due to

  5. while patterns of work and retirement will have huge implications

  6. which shows the balance between working-age people and the older

  7. as proportions of older people increase in most countries

Ответ

A

B

C

D

E

F

3

4

2

5

7

6

Task 15

Elephants sense ‘danger’ clothes

 St Andrews University researchers discovered that elephants could recognise the degree of danger posed by various groups of individuals. The study found that African elephants always reacted with fear A ______ previously worn by men of the Maasai tribe. They are known to demonstrate their courage by В ______ .

The elephants also responded aggressively to red clothing, which defines traditional Maasai dress.

However, the elephants showed a much milder reaction to clothing previously worn by the Kamba people, С ______ and pose little threat.

The researchers first presented elephants with clean, red clothing and with red clothing that had been worn for five days by D ______ .

They revealed that Maasai-smelt clothing motivated elephants to travel significantly faster in the first minute after they moved away.

They then investigated whether elephants could also use the colour of clothing as a cue to classify a potential threat and found the elephants reacted with aggression E ______ . This suggested that they associated the colour red with the Maasai.

The researchers believe the distinction in the elephants’ emotional reaction to smell and colour might be explained by F ______ . They might be able to distinguish among different human groups according to the level of risk they posed.

«We regard this experiment as just a start to investigating precisely how elephants ‘see the world’, and it may be that their abilities will turn out to equal or exceed those of our closer relatives, the monkeys and apes,» researchers added.

  1. either a Maasai or a Kamba man

  2. who do not hunt elephants

  3. when they detected the smell of clothes

  4. who carried out the research

  5. the amount of risk they sense

  6. spearing elephants

  7. when they spotted red but not white cloth

Ответ

A

B

C

D

E

F

3

6

2

1

7

5

Task 16

Culture and customs

 In less than twenty years, the mobile telephone has gone from being rare, expensive equipment of the business elite to a pervasive, low-cost personal item. In many countries, mobile telephones A ___ ; in the U.S., 50 per cent of children have mobile telephones. In many young adults’ households it has supplanted the land-line telephone. The mobile phone is В ___ , such as North Korea.

Paul Levinson in his 2004 book Cellphone argues that by looking back through history we can find many precursors to the idea of people simultaneously walking and talking on a mobile phone. Mobile phones are the next extension in portable media, that now can be С ___ into one device. Levinson highlights that as the only mammal to use only two out of our four limbs to walk, we are left two hands free D ___ — like talking on a mobile phone.

Levinson writes that “Intelligence and inventiveness, applied to our need to communicate regardless of where we may be, led logically and eventually to telephones that we E ___ .”

Given the high levels of societal mobile telephone service penetration, it is a key means for people F ___ . The SMS feature spawned the «texting» sub-culture. In December 1993, the first person-to-person SMS text message was transmitted in Finland. Currently, texting is the most widely-used data service; 1.8 billion users generated $80 billion of revenue in 2006.

  1. to perform other actions

  2. outnumber traditional telephones

  3. to communicate with each other

  4. combined with the Internet

  5. to serve basic needs

  6. banned in some countries

  7. carry in our pockets

Ответ

A

B

C

D

E

F

2

6

4

1

7

5

Task 17

My Stage

 My family moved to Rockaway, New Jersey in the summer of 1978. It was there that my dreams of stardom began.

I was nine years old. Heather Lambrix lived next door, and she and I became best friends. I thought she was so lucky A ___ . She took tap and jazz and got to wear cool costumes with bright sequences and makeup and perform on stage. I went to all of her recitals and В ___ .

My living room and sometimes the garage were my stage. I belonged to a cast of four, which consisted of Heather, my two younger sisters, Lisa and Faith, and I. Since I was the oldest and the bossiest, I was the director. Heather came with her own costumes С ___ . We choreographed most of our dance numbers as we went along. Poor Faith … we would throw her around D ___ . She was only about four or five … and so agile. We danced around in our bathing suits to audiocassettes and records from all the Broadway musicals. We’d put a small piece of plywood on the living room carpet, E ___ . And I would imitate her in my sneakers on the linoleum in the hall. I was a dancer in the making.

My dad eventually converted a part of our basement into a small theater. He hung two “spotlights” and a sheet for a curtain. We performed dance numbers to tunes like “One” and “The Music and the Mirror” from A Chorus Line. I sang all the songs from Annie. I loved to sing, F ___. I just loved to sing. So I belted out songs like “Tomorrow”, “Maybe” and “What I Did For Love.” I knew then, this is what I wanted to do with my life.

  1. like she was a rag doll

  2. whether I was good at it or not

  3. wished I, too, could be on stage

  4. and I designed the rest

  5. and I was star struck

  6. so Heather could do her tap routine

  7. because she got to go to dance lessons

Ответ

A

B

C

D

E

F

7

3

4

1

6

2

Task 18

Cat’s punctuality

 Sergeant Podge, a Norwegian Forest Cat, disappears from his owner’s home in a small town in Kent, every night. But what baffles his owner, Liz Bullard, mostly is the fact that the next morning, the 12-year-old cat always pops up in exactly the same place, A ___ . And every morning Ms. Bullard takes her son to school before collecting Sergeant Podge.

She said that the routine had set in earlier this year, when Sergeant Podge disappeared one day. Ms. Bullard spent hours telephoning her neighbours В ___ .

An elderly woman living about one and a half miles away called back to inform Ms. Bullard that she had found a cat matching Sergeant Podge’s description. Ms. Bullard picked him up but within days he vanished from sight again. She rang the elderly woman С ___ .

She said a routine has now become established, where each morning she takes her son to school before driving to collect Sergeant Podge D ___ .

It is thought Sergeant Podge walks across a golf course every night to reach his destination.

Ms. Bullard said: “If it’s raining he may be in the bush but he comes running if I clap my hands.” All she has to do is open the car passenger door from the inside for Sergeant Podge to jump in.

Ms. Bullard also makes the trip at weekends and during school holidays — E ___ .

She does not know why, after 12 years, Sergeant Podge has begun the routine but explained that another woman who lived nearby used to feed him sardines, and that he may be F ___ .

His owner doesn’t mind his wandering off at night as long as she knows where to collect him.

  1. on the look-out for more treats

  2. from the pavement between 0800 and 0815 GMT

  3. to discover Sergeant Podge was back outside her home

  4. on a pavement about one and a half miles (2.4km) away

  5. to identify if anyone had bumped into him

  6. when her son is having a lie-in

  7. collected by car every morning

Ответ

A

B

C

D

E

F

4

5

3

2

6

1

Task 19

Do you speak English?

When I arrived in England I thought I knew English. After I’d been here an hour I realized that I did not understand one word. In the first week I picked up a tolerable working knowledge of the language and the next seven years convinced me gradually but thoroughly that I A ______ , let alone perfectly. This is sad. My only consolation being that nobody speaks English perfectly.

Remember that those five hundred words an average Englishman uses are B ______ . You may learn another five hundred and another five thousand and yet another fifty thousand and still you may come across a further fifty thousand C ______ .

If you live here long enough you will find out to your greatest amazement that the adjective nice is not the only adjective the language possesses, in spite of the fact that D ______ . You can say that the weather is nice, a restaurant is nice, Mr. Soandso is nice, Mrs. Soandso’s clothes are nice, you had a nice time, E ______ .

Then you have to decide on your accent. The easiest way to give the impression of having a good accent or no foreign accent at all is to hold an unlit pipe in your mouth, to mutter between your teeth and finish all your sentences with the question: “isn’t it?” People will not understand much, but they are accustomed to that and they will get a F ______ .

  1. whatever it costs

  2. most excellent impression

  3. you have never heard of before, and nobody else either

  4. in the first three years you do not need to learn or use any other adjectives

  5. would never know it really well

  6. far from being the whole vocabulary of the language

  7. and all this

Ответ

A

B

C

D

E

F

5

6

3

4

7

2

Task 20

Before the Hubble Space Telescope was launched, scientists thought they knew the universe. They were wrong.

The Hubble Space Telescope has changed many scientists’ view of the universe. The telescope is named after American astronomer Edwin Hubble, A ______ .

He established that many galaxies exist and developed the first system for their classifications.

In many ways, Hubble is like any other telescope. It simply gathers light. It is roughly the size of a large school bus. What makes Hubble special is not what it is, B ______ .

Hubble was launched in 1990 from the “Discovery” space shuttle and it is about 350 miles above our planet, C ______ .

It is far from the glare of city lights, it doesn’t have to look through the air, D ______ .

And what a view it is! Hubble is so powerful it could spot a fly on the moon. Yet in an average orbit, it uses the same amount of energy as 28100-watt light bulbs. Hubble pictures require no film. The telescope takes digital images E ______ .

Hubble has snapped photos of storms on Saturn and exploding stars. Hubble doesn’t just focus on our solar system. It also peers into our galaxy and beyond. Many Hubble photos show the stars that make up the Milky Way galaxy. A galaxy is a city of stars.

Hubble cannot take pictures of the sun or other very bright objects, because doing so could “fry” the telescope’s instruments, but it can detect infrared and ultra violet light F ______ .

Some of the sights of our solar system that Hubble has glimpsed may even change the number of planets in it.

  1. which is above Earth’s atmosphere.

  2. which are transmitted to scientists on Earth.

  3. which is invisible to the human eye.

  4. who calculated the speed at which galaxies move.

  5. so it has a clear view of space.

  6. because many stars are in clouds of gas.

  7. but where it is.

Ответ

A

B

C

D

E

F

4

7

1

5

2

3

Task 21

The science of sound, or acoustics, as it is often called, has been made over radically within a comparatively short space of time. Not so long ago the lectures on sound in colleges and high schools dealt chiefly with the vibrations of such things as the air columns in organ pipes. Nowadays, however, thanks chiefly to a number of electronic instruments engineers can study sounds as effectively A ____ . The result has been a new approach to research in sound. Scientists have been able to make far-reaching discoveries in many fields of acoustics B _____ .

Foremost among the instruments that have revolutionized the study of acoustics are electronic sound-level meters also known as sound meters and sound-intensity meters. These are effective devices that first convert sound waves into weak electric signals, then amplify the signals through electronic means C ______ . The intensity of a sound is measured in units called decibels. “Zero” sound is the faintest sound D ______ . The decibel measures the ratio of the intensity of a given sound to the standard “zero” sound. The decibel scale ranges from 0 to 130. An intensity of 130 decibels is perceived not only as a sound, but also E ______ . The normal range of painlessly audible sounds for the average human ear is about 120 decibels. For forms of life other than ourselves, the range can be quite different.

The ordinary sound meter measures the intensity of a given sound, rather than its actual loudness. Under most conditions, however, it is a quite good indicator of loudness. Probably the loudest known noise ever heard by human ears was that of the explosive eruption in August, 1883, of the volcano of Krakatoa in the East Indies. No electronic sound meters, of course, were in existence then, but physicists estimate that the sound at its source must have had an intensity of 190 decibels, F ______ .

  1. and finally measure them.

  2. since it was heard 3,000 miles away.

  3. and they have been able to put many of these discoveries to practical use.

  4. that loud sound is of high intensity.

  5. as they study mechanical forces.

  6. as a painful sensation in the ear.

  7. that the unaided human ear can detect.

Ответ

A

B

C

D

E

F

5

3

1

7

6

2

Task 22

Chocolate 

Chocolate is made from a number of raw and processed foods produced from the seeds of tropical cacao trees. Cacao has been cultivated in A ______ at least 3000 years. For most of this time it was made into a drink called, in translation — “bitter water”. This is because В ______ to be fermented to develop a palatable flavour. After fermentation the beans are dried and roasted and the shell is removed to produce cacao nibs. These are then ground and liquefied into chocolate liquor. The liquor is then processed into cocoa solids or cocoa butter. Pure chocolate contains primarily cocoa solids and butter in different proportions. Much of С ______ with added sugar. Milk chocolate is sweetened chocolate that additionally contains either milk powder or condensed milk. White chocolate on the other D ______ is therefore not a true chocolate. Chocolate contains theobromine and phenethylamine which have physiological effects on the body. It is similar to serotonin levels in the brain. Scientists claim E ______ , can lower blood pressure. Recently, dark chocolate has also been promoted for its health benefits. But pet owners should remember that the presence of theobromine makes it toxic to cats and dogs. Chocolate is now one F______ , although 16 of the top 20 chocolate consuming countries are in Europe. Also interesting is that 66% of world chocolate is consumed between meals.

  1. the chocolate consumed today is made

  2. that chocolate, eaten in moderation

  3. central and southern America for

  4. of the world’s most popular flavours

  5. hand contains no cocoa solids and

  6. cacao seeds are intensely bitter and have

  7. many countries worldwide at

Ответ

A

B

C

D

E

F

3

6

1

5

2

4

Task 23

Reality TV

 Reality TV seems to dominate broadcasting these days. But what is it, how did it emerge and why on earth is it so popular? The first question is easily answered. Reality TV A ______ presents unscripted, dramatic or humorous situations or events. It can involve celebrities В ______ of the public. Reality TV has been gradually growing in importance for over 60 years. “Candid Camera” — the show that filmed ordinary people reacting to set ups and pranks — started in 1948. Some people, however, believe it was the Japanese with their awful shows in the 1980s and 90s that brought reality TV to centre stage. Others believe С ______ that is called “Big Brother” was the show that spawned the reality TV age. But why are the shows so popular? Different theories come to life. Some believe that it is D ______ we like to watch horrible behaviour: the same instinct that once inspired the ancient Romans to go and watch gladiators destroy each other at the Coliseum. Others suggest a kind of voyeurism is involved there — an unhealthy curiosity to spy on other people’s lives.

Whatever the real reason — the trend seems to have already peaked. A lot of such shows E ______ or are expected to go in the near future. And the replacement seems to be talents shows — watching competitions in dance, singing and general entertainment. Does it mean that people are changing? It is too early to say. Most agree that these F ______ .

  1. due to basic human instinct that

  2. is still early to judge

  3. are simply the cycles of fashion

  4. but more usually the stars are members

  5. that the television phenomenon

  6. is a type of programme that

  7. seem to have disappeared

Ответ

A

B

C

D

E

F

6

4

5

1

7

3

Task 24

Mikhail Lomonosov and Moscow State University

 Mikhail Lomonosov was one of the intellectual titans of XVIII century. His interests ranged from history, rhetoric, art and poetry A ______ . Alexander Pushkin described him as В ______ , whose lifelong passion was learning.

Lomonosov’s activity is a manifestation of the enormous potential of the Russian scientific community. Peter I reformed Russia, which allowed the country to reach the standard of С ______ many spheres. Great importance was placed on education. St. Petersburg Academy of Sciences, founded by Peter I, established a university and a grammar school to educate intellectuals and researchers the country needed; however, these educational establishments could not fulfill the task they took on. It was Michail Lomonosov D ______ of establishing a university in Moscow. An influential courtier and the E ______ Count Shuvalov supported Lomonosov’s plans for a new university and presented them to the Empress.

In 1755, on 25 January-St. Tatiana’s Day according to the Russian Orthodox Church calendar — Elizaveta signed the decree that a university should be founded in Moscow. The opening ceremony took place on 26 April, when Elizaveta’s coronation day was celebrated. Since 1755 25 January and 26 April F ______ Moscow University; the annual conference where students present the results of their research work is traditionally held in April.

  1. who suggested in his letter to Count Shuvalov the idea

  2. to mechanics, chemistry and mineralogy

  3. a person of formidable willpower and keen scientific mind

  4. favourite of Empress Elizaveta Petrovna, the patron of arts and science

  5. the contemporary European powers in

  6. are marked by special events and festivities at

  7. famous among all educated people

Ответ

A

B

C

D

E

F

2

3

5

1

4

6

Task 25

Window Shopping

 The day would be spent with my best friends Kath and Kate. We are actually three Catherines (by birth spelt with a C), A______ we are all K’s: Kat (that’s me), Kath and Kate — the 3K Window Shopping gang!

Window shopping is simply wonderful. You can look at any outfit. You can try on В ______ not a single item on sale for which the price is a problem. You will try something on, ponder, pout, twirl, think hard, check yourself in the mirror one last time and finally reflect С ______ right for you! The highlight of this regular adventure however, is generally the 3K chocolate and ice cream break in the Shopping Centre’s top floor cafii Of course we do not believe that we are wasting anyone’s time. We do D ______ as well, but a reliable equation for us is — 3Ks + shopping mall = a good time.

But E ______ out to be especially memorable. One of the stores had a questionnaire lottery with the first prize being a voucher worth £200. We filled in the question forms while in the cafiiand returned to the store by their 2.00pm deadline. Kate won the first prize but we had decided in advance that if any of us won something, we would share equally: All for one К and one for all! At this point our morning of window shopping paid off. We completed F ______ slightly less than 10 minutes: three skirts, three hats and three belts and three very OK, K’s.

  1. not like to spend our time

  2. that it’s probably not quite

  3. that particular day turned

  4. our real shopping in

  5. sometimes go shopping for real

  6. anything you want and there is

  7. but when we are together

Ответ

A

B

C

D

E

F

7

6

2

5

3

4

Task 26

The Hotel

 “Have you stayed with us before sir?” asked the receptionist. His accent sounded middle-European; Czech possibly or Polish. Actually I hadn’t stayed at this particular hotel before A ______ to many others from the same chain that I had stayed at. “No — first time” I replied with unnecessary brevity. The thing is I always feel В ______ rather than treated as an individual. Every word that I was about to hear, I had heard before — delivered no doubt from the depths of a tourism and hospitality course. “Welcome to Newcastle sir. Is this your first visit to our city? Can I trouble you to complete this form? Actually the first two lines and the signature at the bottom will do. Would you like С ______ , Sir? This will automatically unlock room facilities like mini-bar and telephone and any other extras you may require. Can I see your passport sir?” The questions and information D ______ responses were actually required and I handed over my passport, credit card and partly filled out form. I was tempted to write under name and address “Donald Duck, Duck Towers, Disney Street” — E ______ ever read the form again. But being a creature of habit I wrote my real name and address. While my card was being processed I looked across the reception area through the wall height windows to the beautiful River Tyne. A wave of nostalgia came over me. It was good to be back. I found myself thinking about her again and wondering F ______ a voice broke in: “It’s a plastic key card sir. You also need it to activate the lift and when you get to your room, plug it into the switch on the left as you open the door. It will automatically supply electricity to the room. Any help with your baggage? No? Then enjoy your stay”. The accomplished young Pole smiled as he delivered the final command and duly processed, I proceeded to the card activated lift.

  1. me to take a print of your credit card

  2. points poured out smoothly, no verbal

  3. if I would even see her when

  4. although it seemed virtually identical

  5. so sure was I that nobody would

  6. me to help you with your luggage

  7. as if I am being processed like a product

Ответ

A

B

C

D

E

F

4

7

1

2

5

3

5

Task 27

Lindsay Wildlife Museum

 Lindsay Wildlife Museum is a unique natural history and environmental education centre where visitors can listen to the cry of a red-tailed hawk, go eye-to-eye with a grey fox and watch a bald eagle eat lunch. More than fifty species of native California animals are on exhibit here.

 Thousands of school children learn about the natural environment in their classrooms A ____ of the museum. Nature- and science- oriented classes and trips are offered for adults and children. More than 600 volunteers help to feed and care for wild animals, В _____. Volunteers are active in the museum’s work, contributing С ____.

 The museum was founded by a local businessman, Alexander Lindsay. Sandy, as friends knew him, started teaching neighborhood children about nature in the early 1950s. Initially housed in an elementary school, the museum began offering school-aged children summer classes, D ____.

 After nearly a decade of the museum operation, it became apparent E ____. With a new 5,000 square-foot home, the museum could now develop and display a permanent collection of live, native wildlife and natural history objects. People came to the museum for help with wild animals F ___ urban growth. In response, a formal wildlife rehabilitation programme — the first of its kind in the United States of America — began in 1970.

  1. that a permanent, year-round site was necessary

  2. as well as field trips focused on the natural world

  3. many hours of service to wildlife care and fundraising

  4. that had been injured or orphaned because of intense

  5. that needed public attention and a new building

  6. as well as teach children and adults about nature

  7. through education programmes and on-site tours

Ответ

A

B

C

D

E

F

7

6

3

2

1

4

Task 28

America’s fun place on America’s main street

 If any city were considered a part of every citizen in the United States, it would be Washington, DC. To many, the Old Post Office Pavilion serves A ____. If you are in the area, be a part of it all by visiting us — or В ____. Doing so will keep you aware of the latest musical events, great happenings and international dining, to say the least.

Originally built in 1899, the Old Post Office Pavilion embodied the modern spirit С ____. Today, our architecture and spirit of innovation continues to evolve and thrive. And, thanks to forward-thinking people, you can now stroll through the Old Post Office Pavilion and experience both D ____ with international food, eclectic shopping and musical events. All designed to entertain lunch, mid-day and after work audiences all week long.

A highlight of the Old Post Office Pavilion is its 315-foot Clock Tower. Offering a breath-taking view of the city, National Park Service Rangers give free Clock Tower tours every day! Individuals and large tour groups are all welcome. The Old Post Office Clock Tower also proudly houses the official United States Bells of Congress, a gift from England E ____. The Washington Ringing Society sounds the Bells of Congress every Thursday evening and on special occasions.

Visit the Old Post Office Pavilion, right on Pennsylvania Avenue between the White House and the Capitol. It is a great opportunity F ____, this is a landmark not to be missed no matter your age.

  1. that are offered to the visitors

  2. its glamorous past and fun-filled present

  3. as a landmark reminder of wonderful experiences

  4. by joining our e-community

  5. that was sweeping the country

  6. celebrating the end of the Revolutionary War

  7. to learn more about American history

Ответ

A

B

C

D

E

F

3

4

5

2

6

7

Task 29

Number of teenagers with Saturday job drops

 The number of teenagers with Saturday jobs has dropped. Young people do not acquire any experience for their CVs — a crucial step towards getting full-time work. The proportion of teenagers combining part-time jobs with school or college has slumped from 40% in the 1990s to around 20% now, according to the UK Commission for Employment and Skills (UKCES), a government agency. Latest figures show that only A ____ in 1997.

The trend is not just recession-related, but the result of an increasing expectation В ____ well as a falling number of Saturday jobs, according to the report. Many of the jobs that young people do, such as bar work, are in long-term decline, and are forecast to decline further over the next decade.

«Recruiters place significant emphasis on experience С ____,» the report says. Word of mouth is the most common way to get a job, D _____ young people are unable to build up informal contacts, it adds.

Ms. Todd, a commissioner at the UKCES, said: «There’s more emphasis on doing well at school, young people are finding less time to do what they would have done a few years ago.» «I think it’s also the changing structure of the labour market. Retail is still a big employer, E ____. As a consequence, we need to think about how we get young people the work experience they need.»

A new initiative to send employees into state schools to talk about their careers was also launched recently. The scheme, Inspiring the Future, is meant to give state schoolchildren access to the kind of careers advice that private schools offer. The deputy prime minister said: «The power of making connections F ____ and can be life-changing.»

  1. that it was researching the system of funding education after 16

  2. 260,000 teenagers have a Saturday job compared with 435,000

  3. but young people are leaving education increasingly less experienced

  4. that inspire young people is immeasurable

  5. but an increasing shortage of work experience means

  6. that young people should stay on at school, as

  7. but a lot more of it is being done online

Ответ

A

B

C

D

E

F

2

6

3

5

7

4

Task 30

Lots of fun in Cardiff

 As you would expect of a capital city, Cardiff offers a huge choice of exciting sport and entertainment throughout the year.

Every March the city celebrates St. David, Wales’ patron saint, with parades and music. August sees the International Festival of Street Entertainment, with the heart of the city A ____. Family fun days in the parks and at the waterfront are part of this sensational summer scene. Brass and military bands are often to be seen on Cardiff s streets. Between May and October the world’s only seagoing paddle steamer cruises from Cardiff’s seaside resort.

In autumn the fun continues with Cardiff s Festival of the Arts В _____. Music is at the centre of the festival, with international stars С ____. Christmas in Cardiff is full of colour and festivities. The truly spectacular Christmas illuminations have earned Cardiff the title of «Christmas City». And there is entertainment for all the family, D ____.

There is always something happening in Cardiff. The BBC National Orchestra of Wales and Welsh National Opera can both be heard here. Cardiff previews many London «West End» shows E _____.

The city’s range of accommodation facilities is truly impressive, F ____. And with a city as compact as Cardiff there are places to stay in all price brackets.

  1. from international names to family-run guest houses

  2. joining some of Wales’ most talented musicians

  3. having their summer holidays in Cardiff

  4. that usually attract hundreds of theatre lovers

  5. which features music, film, literature and graphics

  6. from pantomimes to Christmas tree celebrations

  7. beating with dance and theatrical performances

Ответ

A

B

C

D

E

F

7

5

2

6

4

1

Task 31

Changing image

 For more than 200 years Madame Tussaud’s has been attracting tourists from all over the world and it remains just as popular as it ever was. There are many reasons for this enduring success, but at the heart of it all is good, old-fashioned curiosity.

Madame Tussaud’s original concept has entered a brand new era of interactive entertainment A _____. Today’s visitors are sent on a breathtaking journey in black cabs through hundreds of years of the past. They have a unique chance to see the great legends of history, В _____ of politics.

Much of the figure construction technique follows the traditional pattern, beginning whenever possible with the subject С _____ and personal characteristics. The surprising likeliness of the wax portraits also owes much to many stars D _____, either by providing their stage clothes, or simply giving useful advice.

The museum continues constantly to add figures E ____ popularity. The attraction also continues to expand globally with established international branches in New York, Hong Kong, Amsterdam and many other cities. And they all have the same rich mix of interaction, authenticity and local appeal.

The museum provides a stimulating and educational environment for schoolchildren. Its specialists are working together with practicing teachers and educational advisors to create different programmes of activities, F ____.

  1. as well as resources on art, technology and drama

  2. as well as the idols of popular music and the icons

  3. who is sitting to determine exact measurements

  4. ranging from special effects to fully animated figures

  5. ranging from all kinds of souvenirs to sports equipment

  6. that reflect contemporary public opinion and celebrity

  7. who are eager to help in any possible way they can

Ответ

A

B

C

D

E

F

4

2

3

7

6

1

Task 32

Saturday jobs: memories of weekend working

 Research has shown a sharp fall in the number of teenagers who do Saturday jobs. It seems such a shame — my Saturday job as a kitchen porter was something of a rite of passage. I’ll never forget long hours A _____, scouring grease off huge saucepans and griddles. Working atmosphere there helped me grow a thicker skin, develop quicker banter and, most importantly, taught me the value of hard work. It also resulted in a steady supply of cash, В ____. I’m not the only one who has strong memories of weekend work. DJ Trevor Nelson said everyone should be able to have a Saturday job: «It taught me a lot, С ____.»

The link between the type of Saturday job a celebrity performed and their later career is sometimes obvious. Dragon’s Den star and businessman Peter Jones, for example, showed early promise by starting his own business. «I passed my Lawn Tennis Association coaching exam, D ____,» he explains. «At the start I was coaching other kids, E ____, for which I could charge £25-30 an hour. While my friends on milk rounds were getting £35 a week, I was doing five hours on a Saturday and earning four times as much.»

Skier Chemmy Alcott got a job working for the Good Ski Guide, on the advertising side. «It became clear to me what my personal value to companies could be. It led directly to me finding my head sponsor … and it offered me an eight-year contract. That gave me the financial backing F ____.»

As part of its response to the Saturday job statistics, the UK Commission for Employment and Skills said a lack of early work opportunities makes it harder for young people to acquire experience for their СVs.

  1. and things would be different if everyone was given the chance

  2. which let me know he approved of me

  3. and I persuaded my local club to let me use a court on Saturdays

  4. which I needed to become a professional skier

  5. which I would happily spend as I liked

  6. that I spent in the kitchen of a busy country pub in East Sussex

  7. but soon I got adults wanting to book lessons

Ответ

A

B

C

D

E

F

6

5

1

3

7

4

Task 33

Orient Express

 In the early 1860s, trains were the preferred way to travel. They weren’t particularly comfortable, however, until American engineer George Mortimer Pullman decided to make trains more luxurious.

By the late 1860s, trains furnished not only sleeping cars, but kitchen and dining facilities, where A _____. This was innovative for the time, and was aimed to encourage people В _____. The first of these Pullman trains in England ran from London to Brighton and used electricity for illumination.

In 1881, another railway entrepreneur, George Nagelmacker, introduced the use of a restaurant car onboard, and the first Orient Express train service was begun. Running from Paris to Romania the route included Strasbourg, Vienna, Budapest and Bucharest.

Thanks to the 12 mile Simplon Tunnel, С _____, the Orient Express expanded, including a route to Istanbul, and the legendary romance of the Orient Express was in full swing.

Everyone in the social register, including royalty, chose to travel on the wheels of that luxury hotel D _____ in wealthy surroundings. Legends, stories, and intrigue surrounded those trips to exotic places, and those famous people E _____.

Unfortunately, during World War II this luxury travel was closed for the most part, and later, after the war, F ____ to start it again. Within the next few years airplane travel became popular, and train passenger service declined.

  1. elegant meals were served to passengers

  2. to use trains for long distance travel and vacations

  3. who rode the train

  4. who wrote about it

  5. which connected Switzerland and Italy

  6. that served dishes and wines

  7. there was no money

Ответ

A

B

C

D

E

F

1

2

5

6

3

7

Task 34

Arizona’s world class cruise

 Spectacular Canyon Lake is situated in the heart of the Superstition Mountains in Arizona, giving home to the Dolly Steamboat. The Dolly Steamboat, A ____, now cruises the secluded inner waterways of this beautiful lake. It is worth exploring this favourite destination of President Theodore Roosevelt who declared, «The Apache Trail and surrounding area combines the grandeur of the Alps, the glory of the Rockies, the magnificence of the Grand Canyon and then adds something В ____.» You will marvel as you travel up to the national forest, which provides the most inspiring and beautiful panorama С ____. Every trip brings new discoveries of rock formations, geological history, and the flora and fauna distinct to the deserts of Arizona.

Once aboard the Dolly Steamboat, you may view the majestic desert big horn sheep, bald eagles and a host bird of other wildlife, water fowl, D ____. Experience the unique sound harmony that is created by the waters of Canyon Lake. Stretch out and relax at one of the tables or stand next to the railings on the deck. There is plenty of leg room on the Dolly. You will get a unique chance to listen to the captain E ____.

All the passengers are treated with outstanding service and personal attention to every need. Feel free to ask questions, move about and mingle with the crew. So enjoy an unforgettable vacation cruise and see F ____ ,like a ride on Arizona’s Dolly Steamboat.

  1. that nature has ever created in the wild

  2. that none of the others have

  3. hovering over the magnificent lake

  4. who retells the legends of the mysterious past

  5. for yourself why there is nothing quite

  6. who pays much attention to children’s safety

  7. continuing a tradition of cruising since 1925

Ответ

A

B

C

D

E

F

7

2

1

3

4

5

Task 35

US Congress

 The Congress of the United States of America is an important part of the US federal government.

It is an assembly of elected representatives A ____ but not to select the chief executive of the nation; that individual is elected by the people.

Congress is not a single organization; it is a vast and complex collection of organizations B ____ and through which members of Congress form alliances.

C ____, in which political parties are the only important kind of organization, parties are only one of many important units in Congress.

In fact other organizations have grown in number D ____.

The Democrats and Republicans in the House and the Senate are organized by party leaders, E ____ within the House and Senate. The party structure is essentially the same in the House as in the Senate, though the titles of various posts are different.

But leadership carries more power in the House than in the Senate because of the House rules. F _____, the House must restrict debate and schedule its business with great care; thus leaders who do the scheduling and who determine how the rules shall be applied usually have substantial influence.

  1. as party influence has declined

  2. against the spirit of the Constitution

  3. being so large (435 members)

  4. empowered to make laws

  5. unlike the British Parliament

  6. by which the business of Congress is carried on

  7. who in turn are elected by the full party membership

Ответ

A

B

C

D

E

F

4

6

5

1

7

3

Task 36

The Trailblazers

 In the early 1800s, the area that would become the western United States was completely undeveloped.

Explorers, hunters, traders, and settlers had to blaze their own trails. A____ to move possessions and supplies became common place.

Manifest Destiny was the belief that Americans had a God-given right to take over the continent. As they moved west, settlers used this policy B_____ to new people and territories.

Trails increased trade opportunities between western and eastern regions, and the U.S. economy prospered C_____ on each other for goods.

To achieve Manifest Destiny, the United States purchased land from other countries or conquered territory D_____ until its borders stretched from coast to coast.

More than one-half million people chose to travel West on trails between 1800 and 1870, E_____.

As new technology spread across the West, however, the use of trails came to an end. The railroads built thousands of miles of tracks, and, F ____, a cheap, relatively safe, and quick way to transport people and supplies to western areas existed.

  1. to spread U.S. ideas and government

  2. for the first time in history

  3. thus replacing them forever

  4. as territories became interdependent

  5. the use of covered wagons

  6. by taking land from Native peoples

  7. forming the largest mass migration in history

Ответ

A

B

C

D

E

F

5

1

4

6

7

2

Task 37

A Young Mayor

 This is a very unusual case, but as you will see, unusual doesn’t mean impossible.

An 18-year-old school girl has become the youngest mayor of a British town in history. Amanda Bracebridge, A_____, won leadership of Clun village council in a dramatic election last night. The tiny village only has 122 voters and Amanda won the election by just two votes from the only other candidate, 69-year-old Fred Gardner of the Conservative party. Amanda, B _____, was an independent candidate. She was surprised by her success, C _____. “My election promise was to make sure D _____,” she told us. She was referring to the plans from a large company to buy up farmland and build flats there. “We live in one of the most beautiful villages in Shropshire and I want to make sure it stays that way.”

Amanda, who is in her last year at nearby Bishop’s Castle High School, E _____ and her exams which she takes in two months. “It’s going to be a pretty busy few months,” she said. “But when the exams are over I will be able to concentrate completely on helping my village”.

Amanda had plans to go to university but is now going to start a year later F _____. “I’ve talked to Leeds University and they say my place will wait for me”. And what is she going to study? Politics? “No, actually, I am going to do sociology and economics”.

  1. who is not a member of any political party

  2. that our village would be protected from outside interests

  3. but it was not a total shock to her

  4. being a politics student at the university

  5. so she can do her job as mayor properly

  6. who is only just old enough to vote herself

  7. will have to find time for her work as mayor

Ответ

A

B

C

D

E

F

6

1

3

2

7

5

Task 38

Is there enough to say?

 They only appeared about ten years ago but already they are everywhere, everyone’s got one. They are the wonder of the modern age — mobile phones, or cell phones, A ____. Apparently, mobile phones are now used by about 2.5 billion people worldwide, and about one billion new mobile phones are sold every year worldwide. Go back to 1997, and only 100 million were sold. As we can see, the mobile phone business B_____.

And the developments keep on coming. Once we could only make phone calls; now mobile phones C_____ and do many other useful things. Once we had to hold our mobile phones in our hand; now we can use throat microphones. What next? We are told that soon, tiny microphones will be implanted into our lips. We’ll be able to dial numbers just by saying them.

But surely we need to ask ourselves: What’s good about this? OK, we can talk to other people almost all the time now — but is that so great? Watch and listen to people when a plane has landed. Anxious D _____, dial a number, and then: “It’s me, I’m here. I’ll be there in twenty minutes.” Is this communication? Is this what all these years of technology have brought us to?

In the early days of communication there were letters. When they arrived at your house, you knew they had been delivered by a man E _____.

In those days, people would think very hard before they wrote a letter. You had to have a good reason to write — communication was serious. Now it’s not — people phone each other F ____. Once the phone was a way for people far away from each other to talk — now it’s just an excuse to talk.

  1. has been developed very quickly

  2. not understand why they are doing it for

  3. as Americans call them

  4. riding halfway across the country on a horse

  5. just because they can

  6. can also be used to take and send photos

  7. fingers immediately switch on the mobile phone

Ответ

A

B

C

D

E

F

3

1

6

7

4

5

Task 39

Promoting language learning

 The European Union (EU) is committed to supporting the rights of its citizens to personal and professional mobility, and their ability to communicate with each other. It does so by A_____ to promote the teaching and learning of European languages. These programmes have at least one thing in common: they cover cross-border projects involving partners from two, and often three or more, EU countries.

The EU programmes are designed to complement the national education policies of member countries. Each government is responsible for its own national education policy, B_____. What the EU programmes do is to create links between countries and regions via joint projects, C____.

Since 2007 the main programmes have been put under the overall umbrella of the EU’s lifelong learning programme. All languages are eligible for support under this programme: official languages, regional, minority and migrant languages, D____. There are national information centres in each country, E_____.

The cultural programmes of the EU also promote linguistic and cultural diversity in a number of ways. The “Media” programme funds the dubbing and subtitling of European films for F ____. The “Culture” programme builds cross-cultural bridges by supporting the translation of modern authors into other EU languages.

  1. and the languages of the EU’s major trading partners

  2. which includes language teaching and learning

  3. cinemas and television in other EU countries

  4. which enhance the impact of language teaching and learning

  5. funding a number of educational programmes

  6. and encouraging people to learn new languages

  7. where details about the application procedures are given

Ответ

A

B

C

D

E

F

5

2

4

1

7

3

Task 40

Starting your own business

What are the reasons for starting your own business? One of them is because you believe you are the best in that line or because you have a product or service that has never been offered to the market before. Another is that you are a person in a real hurry and cannot suffer the A_____ to reach your goals. Sometimes it is because you have an inheritance B_____ soon after you set up a business or that there already is a cash purse with loose strings and you want to make the best of this bonanza.

If your reasons are any or all of the above, abandon the thought right now and save yourself the disillusionment C____ into the world of commerce.

Start your own business just for the sake of doing a trade, or for D____. Do not burden yourself with lofty notions of superiority when compared to your peers. When setting out to start your own business, be emotional about it, but not impractical; don’t be led by your heart, but be dictated by your mind.

Having covered those parts that are not taught in a business school, let us look at E____ your own business. You should start with a SWOT analysis – strengths, weaknesses, opportunities and threats – analyze these for yourself, for partners in your business, if any, and for the business itself.

If the result of the analysis is encouraging, then prepare a business plan. It is like a road map for actions in the near foreseeable future to achieve your business goals. Finally, execute the business plan with precision; tweak it as you go along, only so that it helps to meet the end goal of successfully F_____ the business.

  1. the essentials of starting

  2. that awaits when you step

  3. trials and tribulations of employment

  4. establishing and conducting

  5. preparing a business plan

  6. waiting to be acquired

  7. undertaking the commercial activity

Ответ

A

B

C

D

E

F

3

6

2

7

1

4

Task 41

Archaeology done underwater

 Nautical archaeology is the science of finding, collecting, preserving, and studying human objects that have become lost or buried under water. It is a fairly modern field of study since it depends on having the technology to be able to remain underwater for some time to do real work. Whether it is conducted in freshwater or in the sea, A____, nautical archaeology is another way of learning more about the human past.

Although some use the words nautical archaeology to mean a specialized branch of underwater archaeology, B____, most consider the term to mean the same as the words underwater archaeology or marine

archaeology. All of these interchangeable terms mean simply C_____.

Once real trade began, it is safe to say D_____ was probably transported over water at some point in time. By studying submerged objects, we can learn more about past human cultures. In fact, studying ancient artifacts is the only way to learn anything about human societies E_____. Being able to examine the actual objects made and used by ancient people not only adds to the written records they left behind, but allows us to get much closer to the reality of what life was like when they lived. Also, if we pay close attention to how the objects were made and used, we begin to get a more realistic picture of F_____.

  1. that existed long before the invention of writing

  2. that nearly every object made by humans

  3. what those people were really like

  4. which is concerned only with ships and the history of seafaring

  5. that it is the study of archaeology done underwater

  6. and whether it finds sunken ships or old cities

  7. and what was discovered underwater

Ответ

A

B

C

D

E

F

6

4

5

2

1

3

Task 42

Visiting the Royal Parks

 London has a well-deserved reputation as one of the greenest cities in Europe, with a huge number of open spaces across the center of the city. Tourists A_____ can always relax in a lovely, quiet London park.

The Royal Parks, such as St James’s, Green Park, the Regent’s Park, Hyde Park, Richmond, Greenwich, Bushy Park and Kensington Gardens, are beautifully maintained and popular with locals and visitors alike. Many are former hunting estates of English monarchs, preserved as open space B______. They are ideal places to relax and sunbathe in summer, enjoy gorgeous flower beds in spring C_____.

The Royal Parks provide fantastic green routes in London D______ and through some of the most attractive areas of the capital. Picnics in the parks are also a popular activity especially during the busy summer months.

Dogs are welcome in all the Royal Parks, although there are some places E_____. These are clearly indicated within each park and are usually ecologically sensitive sites, children’s play areas, restaurants, cafes and some sports areas. Ground nesting birds are particularly sensitive to disturbance by dogs and people. So it is necessary to observe the warning signs F____. In Bushy Park and Richmond Park dogs should be kept away from the deer.

The Royal Parks are for everyone to enjoy.

  1. that are displayed during the nesting season

  2. while the city has grown up around them

  3. and admire the changing leaves as autumn arrives

  4. where they are not allowed or should be kept on a lead

  5. who are tired of the noise, crowds and excitement of sightseeing

  6. who does not know the route to the place of destination

  7. that take cyclists away from traffic

Ответ

A

B

C

D

E

F

5

2

3

7

4

1

Task 43

The Survival of the Welsh Language

 Wales is a small country of just over 3 million people, on the north west seaboard of Europe. Despite many historical incursions of other peoples, particularly the English, it has preserved its ancient Celtic language, A_____. Welsh is habitually spoken by about 10% of the people, half understood by a further 10%, and not spoken at all by the majority in this ‘bilingual’ society.

Up to the First World War most people were Welsh speaking, especially in the mountains of North Wales. The English-speaking areas were along the more fertile coastal plains. On the whole there was an easy tolerance of the two languages, B______.

By 1919 there was a considerable drop in Welsh speakers. This was due to the large flows of capital investment from England into the South Wales coalfield, C_____.

Now, D_____, commerce and everyday business were carried out in English.

In the rural mountain areas 80% to 85% of the population were Welsh speakers, E ____. However, in the coalfield country of Glamorgan 70% spoke English only, and in its neighbour border county the figure was over 90%.

By 1931 the number of people able to speak Welsh in the whole of Wales had fallen to 37% of the population, F ____. It continued to drop and reached its lowest – 18.6% — in the 1990s. But by the start of the 21st century, numbers had begun to increase again and reached 21.7% in 2004!

  1. as well as education and the law

  2. the only one of a number of allied languages that remain

  3. with radio and the English press further speeding the decline

  4. many being able to speak Welsh only

  5. where Welsh was studied as language and literature in an academic manner

  6. apart from the fact that Welsh was not permitted to be used at all in the schools

  7. bringing a flood of immigrant labour from all over Britain

Ответ

A

B

C

D

E

F

2

6

7

1

4

3

Task 44

Secrets of Long Life

 There are places in the world where people live longer than anywhere else. The remote Japanese island of Okinawa is one of these places. While the lifespan in Britain is 77 years for men and 81 for women, Okinawa has a population of about one million, of which 900 are centenarians — A_____ in Britain or the USA. So what is their secret of long life?

«The calendar may say they’re 80, but their body says they’re 60,» says Bradley Willcox, a scientist researching the extraordinary phenomenon. The research has shown hormonal differences between Okinawans and B____ but their longevity has been linked to diet. They eat more tofu and soya than any other people in the world and also enjoy a range of different fruit and vegetables, all rich in anti-oxidants. But the most significant thing isn’t what they eat but how much. The Okinawans C_____ known as ‘hara hachi bu’, which translates as ‘eat until you are only 80 % full’.

Scientists refer to this way of eating as ‘caloric restrictions’. No-one knows exactly why it works, but scientists believe it D_____ that there is the danger of famine. This in turn E_____ and so may lead to better preservation and slower aging.

«It’s a stark contrast with the cultural habits that drive food consumption in F____ » says Mr. Willcox. If we look at high streets and supermarkets in most other countries, you will see that he is right. Restaurants offer all-you-can-eat menus and supersize portions. Supermarkets are full of special offers encouraging us to buy more food than we need.

  1. make it a healthy diet

  2. other parts of the world

  3. four times higher than the average

  4. have a cultural tradition

  5. sends a signal to the body

  6. the rest of the population

  7. makes the body protect itself

Ответ

A

B

C

D

E

F

3

6

4

5

7

2

Task 45

Beaches of Portugal

 Covering more than 850 km, the Portuguese coast boasts such a large number of fine, white sandy beaches that it is almost impossible to keep count. All bathed by the Atlantic Ocean and all different, their beauty is hard to describe, so there is nothing better A _____.

The most famous are in the Algarve. With three thousand hours of sun per year and warm waters, there are beaches to suit every taste and many dreamlike resorts. The choices are many, from sandy stretches extending as far as the eye can see B ______, the trade image of the region. They are always accompanied by a calm clear sea, C_____.

In Costa da Caparica, the beaches are particularly dear to Lisbonites D _____ for sun and sea bathing. There are deserted beaches here too, of a wild beauty, E ____ nature. In the centre, tourists will find very wide sandy stretches, to which traditional fishing adds a picturesque touch. And further north, the colder waters and the invigorating sea are tempered by the welcoming atmosphere and the clean air of the mountains and the forests.

Despite all their differences, all beaches share one thing – quality. They are safe and offer a wide range of support and recreational services, F ____. And a large number of Portuguese beaches are granted the European blue flag every year, a distinction that is a sign of their excellent conditions.

  1. where one can enjoy close contact with

  2. which meet every need of their users

  3. than to discover them once for oneself

  4. who has never been to this wonderful city

  5. which is ideal for various water sports

  6. to the smaller coves, sheltered by huge cliffs

  7. who have different options around the capital

Ответ

A

B

C

D

E

F

3

6

5

7

1

2

Task 46

The Joy of Reading

 Have you ever wondered why people read? Why reading is one of the few things A _____ for thousands of years? Even before reading became available to the general public, stories were told around campfires, passed down from generation to generation.

First of all, stories are a good way to escape from your ordinary life, to get immersed in another world, if only for a little time. While reading, you can imagine yourself in different situations B _____, but in the moment that doesn’t matter. Whether you’re suffering from depression or are just bored, reading is a great distraction.

Similarly, another reason people are attracted to stories, is because they are lonely, very often they feel as if they are the only ones in the world C _____. Identifying with a fictional character can make a big difference in helping a person understand D _____.

Other people read because it can be a good way to relax. It can be very nice to sit down and enjoy a good plot unfold, to watch the actions of fictional characters from the side, and to see the consequences of these actions, E ____.

Lastly, people read because it is the easiest way to gain knowledge in a certain area. Instead of finding a teacher, you can just find a book, sit down, and spend a few hours reading. This way you can study wherever you want, whenever you want F _____.

There are countless books in the world, and whoever you are, whatever you’re feeling, there is definitely a book out there, just waiting for you to discover it.

  1. try to avoid the boredom of life

  2. that has consistently remained part of society

  3. that they are not alone

  4. going through something difficult

  5. without having to bear any responsibility

  6. that range from unlikely to impossible

  7. at your own pace

Ответ

A

B

C

D

E

F

2

6

4

3

5

7

Task 47

Peter and Paul Fortress

 The Peter and Paul Fortress in St. Petersburg, located on small Hare Island, is the historic core of the city. The history of St. Petersburg begins with the history of the fortress.

Since 1700 Russia had been fighting the Northern War against Sweden. By 1703 the lands by the Neva River were conquered. To protect them from the attacks of the Swedes it was necessary to build a strong outpost here. The fortress was founded on Hare Island 16 (27) May, 1703 by joint plan of Peter I and French engineer Joseph-Gaspard Lambert de Guerin. This day is well known A____.

The fortress stretches from west to east with six bastions B____. The Peter’s Gate on the east side, C____, has remained since the time of Peter I. The Peter and Paul Cathedral, D____ emperors and the monument of Russian baroque, was completed after the death of the emperor, in 1733. The weathervane as a golden angel with a cross, E____, is one of the main symbols of the city. On the opposite side of the cathedral, there is the Mint building, constructed in the time of Paul I by architect A. Porto. Coinage was moved to the fortress F____ in the time of Peter I. The Peter and Paul Fortress has never directly participated in any fighting. From the very beginning of its existence it was used as a political prison. Since 1924 the Peter and Paul Fortress has been a part of the Museum of the History of St. Petersburg.

  1. as the day of the birth of St. Petersburg

  2. which was designed by D. Trezzini

  3. which was the burial place of Russian

  4. and reminding of the rich history of the city

  5. as the most protected part of the city

  6. which is located on the spire of the cathedral

  7. that are located at the corners

Ответ

A

B

C

D

E

F

1

7

2

3

6

5

Task 48

Surviving in a Desert

 A desert is defined as a place that gets less than 250 mm of rain each year. It differs sharply from the climate of a rain forest, A _____.

Arid desert lands cover about one third of the earth’s surface. Most deserts are covered with sand, B _____. There are also usually a lot of rocky areas. This combination of sand and rock means that the soil is not very fertile. C ____, some living things are able to do well in this setting. Many plants have changed and developed in ways D____. These changes have become apparent in a number of ways. Some plants are able to grow very quickly E____. They turn green and produce flowers within just a few days. Other desert plants simply stop growing in very dry weather. They appear to be dead, but when the rain returns, they come back to life and begin growing again.

Desert animals have also developed many characteristics that help them to survive in arid environment. Camels can go for a very long time without drinking. Other animals, such as snakes and rats, find cool places to sleep during the day and come out only at night. The extremely long ears of desert rabbits help them F_____. Changes like these have allowed some animals and plants to grow and develop successfully in a very challenging ecological system: the desert.

There are countless books in the world, and whoever you are, whatever you’re feeling, there is definitely a book out there, just waiting for you to discover it.

  1. which is often in the form of hills called sand dunes

  2. whenever it rains

  3. to find water as far as 25 metres away

  4. which can receive up to 10,000 mm of rain annually

  5. to better distribute their body heat and stay cool

  6. even though the desert environment is very dry and hot

  7. that help them to live in the desert

Ответ

A

B

C

D

E

F

4

1

6

7

2

5

Task 49

Nevsky Prospect

 Nevsky Prospect is the main and most famous street of St. Petersburg. The unique architectural ensemble of Nevsky Prospect was formed during the 18th – early 20th centuries. It starts from the bank of the Neva River, runs through the centre of the city and ends at the Neva River. The whole history of St. Petersburg can be seen in the history of the avenue. Nevsky Prospect is 4.5 km long and 25-60 m wide. The narrowest section is located from the Admiralty to the Moika River, A_____.

After the construction of the Admiralty in 1704 and the Alexander Nevsky Monastery in 1710, it was decided to build a road B_____ each other and with the Novgorod Path, which was used by Russian merchants. The construction began on both sides at the same time, the roads were laid through the wood, and in 1760s they were connected into one road, C_____, but with a turn at the Vosstaniya Square. Nevsky Prospect got its name only in 1783. The road was paved with cobble stones, D_____. It was the first street in St. Petersburg with gas lighting. By the early 20th century Nevsky Prospect had become the financial centre of Russia E____ had their offices there.

Nowadays, Nevsky Prospect is the centre of cultural and social life of St. Petersburg. There are museums, theatres, exhibition halls, cinemas, restaurants, cafés, shops F____.

  1. and hotels there or nearby the avenue

  2. showing the original width of the avenue

  3. which was not as straight as it was planned

  4. which were built by famous architects and

  5. connecting these two important structures with

  6. and a few rows of trees were planted along the street

  7. as the 40 largest banks of Russia, Europe and America

Ответ

A

B

C

D

E

F

2

5

3

6

7

1

Task 50

Whales in a Noisy Ocean

Whales use sound in very different ways. Some whales produce songs that travel over vast distances. They also use echolocation, like bats, A _____. But other noise in the ocean creates a problem for the whales.

Since 1987, the International Fund for Animal Welfare (IFAW) has sent their research vessel Song of the Whale around the world B _____. During the travels, the Song of the Whale scientists have developed expertise C ____ to listen to and record the sounds that the animals make. Thishelps them to track, identify, and survey different species.

One of the threats facing whales and other marine animals is noise pollution in the seas, such as noise from drilling, military activities, oil exploration, and coastal construction. This noise can cause great distress to whales and dolphins and can D _____.

It is feared this noise pollution may cause mass strandings, E _____. If the Song of the Whale team can F ____, then hopefully the nature and location of disturbing noise can be changed.

  1. in using underwater microphones

  2. to locate food and find their way

  3. result in injury and even death

  4. track and identify their habitats

  5. to filter out food from the water

  6. to provide a platform for marine research

  7. when large numbers come ashore

Ответ

A

B

C

D

E

F

2

6

1

3

7

4

Task 51

Unique nature of Kamchatka

Kamchatka is a peninsula located in the north-eastern part of Russia. It is surrounded with the Okhotskoye Sea, the Beringovo Sea and the Pacific Ocean. This region has a very unique environment A_____ one is looking for picturesque views, unforgettable travels and unity with nature.

Kamchatka is famous for its volcanoes, B_____. Volcanoes are represented on Petropavlovsk-Kamchatsky, the most eastern city in the northern hemisphere, coat of arms as well. There are more than 300 volcanoes

in Kamchatka, from 28 up to 36 of them are active, or potentially active. Kamchatka volcanoes are included in the list of the UNESCO World Heritage Sites.

The region is also known C____ — rivers and lakes. Many Kamchatka rivers spring from mountain tops and glaciers, that is why they are very clean, and it is wonderful for those D_____. In general, there are up to 14 thousand rivers and streams, 100 thousand lakes and 414 glaciers in Kamchatka.

Kamchatka is a home to the Valley of Geysers, E_____ geysers in the world, after Icelandic geyser fields. It is not easily accessible, as long as it is too unique to be opened for tourists all the time. The Valley of Geysers’ ecosystem is very vulnerable, F_____ and regulate the visiting. In fact, the larger part of Kamchatka is preserved. There are many nature reserves and nature parks in Kamchatka.

  1. which are depicted on most souvenirs there

  2. so it is necessary to monitor it all the time

  3. who love fishing, including Kamchatka bears

  4. which has the second largest concentration of

  5. to be a place of many water sources

  6. to be a popular nature reserve and health resort

  7. that makes it a place to visit when

Ответ

A

B

C

D

E

F

7

1

5

3

4

2

Task 52

The life of Pi

 «The Life of Pi» published in 2001 is the third book by the Canadian author Yann Martel. It has A_____, won several prizes and been translated into forty-one languages.

At the start of the book, we B____ in India. His father owns the city zoo and the family home is in the zoo. When they aren’t at school, Pi and his brother help their father at the zoo and he learns a lot about animals.

When Pi is sixteen, his parents decide to close the zoo and move to Canada. They travel by ship taking the animals with them. On the way, there is C_____. Sadly, Pi’s family and the sailors all die in the storm, but Pi lives and finds himself in a lifeboat with a hyena, zebra, orangutan and an enormous tiger. At first, Pi is scared of the animals and jumps into the ocean. Then he remembers there are sharks in the water and decides to climb back into the lifeboat. One by one, the animals in the lifeboat kill and eat each other, till only Pi and the tiger are left alive. Luckily for Pi, there is D_____, but he soon needs to start catching fish. He feeds the tiger to stop it killing and eating him. He also uses a whistle and E_____ and show it that he’s the boss.

Pi and the tiger spend 227 days in the lifeboat. They live through terrible storms and the burning heat of the Pacific sun. They are often hungry and ill. Finally, they arrive at the coast of Mexico, but you will have to F_____ in the end!

  1. read the book to find out what happens

  2. some food and water on the lifeboat

  3. his knowledge of animals to control the tiger

  4. received an award for being strong

  5. sold seven million copies worldwide

  6. learn about Pi’s childhood in Pondicherry

  7. a terrible storm and the ship sinks

Ответ

A

B

C

D

E

F

5

6

7

2

3

1

Task 53

Santa Claus

The man we know as Santa Claus has a history all to his own. Today, he is thought of mainly as the jolly man in red, but his story A_____ the 3rd century to a monk named St. Nicholas. It is believed that Nicholas was born sometime around 280 AD in modern-day Turkey. Much admired for his kindness, St. Nicholas B_____. It is said that he gave away all of his inherited wealth and traveled the countryside helping the poor and sick. Over the course of many years, Nicholas’s popularity spread and he became known as the protector of children and sailors. His feast day C_____ his death, December 6. This was traditionally considered a lucky day to make large purchases or to get married. By the Renaissance, St. Nicholas was the most popular saint in Europe.

St. Nicholas first D______ at the end of the 18th century. The name Santa Claus evolved from a Dutch shortened form of Sint Nikolaas. As his popularity grew, Sinter Klaas was described as everything from a jocker with a blue three-cornered hat, red waistcoat, and yellow stockings to a man wearing a broad-brimmed hat and a huge pair of Flemish trousers.

In the 19th centuries big stores E_____ using images of the newly-popular Santa Claus. In 1841, thousands of children visited a Philadelphia shop to see a life-size Santa Claus model. It F_____ before stores began to attract children, and their parents, with the lure of a peek at the “real-life” Santa Claus with his famous white beard and red gown.

  1. began to advertise Christmas shopping

  2. became the subject of many legends

  3. began dressing up unemployed men in

  4. is celebrated on the anniversary of

  5. was only a matter of time

  6. stretches all the way back to

  7. appeared in American popular culture

Ответ

A

B

C

D

E

F

6

2

4

7

1

5

Task 54

Welcome to the Smithsonian

When you visit any of the Smithsonian’s 19 museums and galleries or the National zoo, you are entering the largest museum complex in the world. This complex holds about 137 million unique objects in its trust for the American people.

The Smithsonian was established in 1846 with funds given to the United States by James Smithson, an English scientist. The main idea was to increase and spread knowledge for free. And now all Smithsonian institutions are still devoted to public education, A__________ history.

Ten Smithsonian museums and galleries are located in the centre of the U.S. capital. Six other museums and the National zoo are nearby in the Washington metropolitan area, B__________.

The 19th and the newest museum C__________ is the National Museum of African American history and culture. It is now operating in the form of a virtual museum. Its key feature is the memory book, D__________. These diverse memories are linked to each other and to the museum content, E__________.

The Smithsonian complex is home to the world’s foremost research centres in science, the arts and the humanities. Besides the basic research F__________, there are a number of special facilities. Conservation centre at the zoo studies rare and endangered species, environment centre carries out research in ecosystems in the coastal area.

  1. that is carried on regularly in each of the museums

  2. providing different materials in the arts, science and

  3. placing a spotlight on people and events in African American history

  4. that has been established within the Smithsonian complex

  5. which allows website visitors to upload their own stories or images

  6. and visitors can enjoy watching rare exhibits on

  7. and two museums are situated in New York City

Ответ

A

B

C

D

E

F

2

7

4

5

3

1

Task 55

National Gallery of Art

 The National Gallery of Art was created in Washington D.C. for the people of the United States in 1937. It started with the gift of the financier and art collector A__________. His gift also included a building to house the new museum, to be constructed on the National Mall. Opened to the public in 1941, this grand building, B__________, was at the time the largest marble structure in the world.

The newly created National Gallery soon attracted similar gifts from hundreds of other collectors. This tradition of generosity continues to this day with gifts from private donors and artists C__________.

The gallery’s East building contains the collection of modern and contemporary painting, sculpture, D__________. The East and West buildings are connected by an underground tunnel with a moving walkway.

The National Gallery enjoys federal support, E__________, to fulfill its mission to exhibit and interpret great works of European and American art in the nation’s collection. Since its founding, federal funds have fostered the protection and care of the art collection and have supported the gallery’s work, ensuring F__________. Private funding helped to create a renowned collection of works of art and to construct the two landmark buildings. Private support makes possible to arrange a changing programme of special exhibitions.

  1. which is now called the West building

  2. that the gallery brings daily profit to the country

  3. who are willing to share their possessions with the public

  4. who presented old master paintings and sculptures to the country

  5. as well as partnership with private organizations

  6. that the gallery is open daily and free of charge

  7. as well as an advanced research centre and an art library

Ответ

A

B

C

D

E

F

4

1

3

7

5

6

Task 56

Healthy school meals

Children at Southdown Infants School in Bath enjoy tasty homemade meals such as roast turkey with fresh vegetables, chicken, salad and fresh fruit for pudding. Vegetables are A ____________. Instead of crisps, chocolate and sweets, the school canteen serves organic carrots, dried fruit and fresh seasonal fruit in bags for 10p, B ______________.

Southdown’s healthy eating initiative began four years ago with the start of a breakfast club.

Now Ms Culley, the head teacher of the school, says that the teachers very clearly see the link between diet and concentration. “Children’s concentration and behaviour C ______________.” The teachers would also like to give the children the experience of eating together. It turned out that some children weren’t used to that.

Pupils are also encouraged to find out more about where their food comes from by  D ______________.

Parents are also involved and are invited in to try school dinners on special occasions, E _______________.

The efforts of staff, pupils and parents to create a healthy eating environment were recognized earlier this month F ______________ the Best School Dinner award.

Ms Culley said: “We are happy to win this award. Healthy eating is at the centre of everything we do. It’s really rewarding to see so many children enjoy real food.”

  1. such as Easter and Christmas

  2. visiting a local farm

  3. local, fresh and organic where possible

  4. provide good quality food

  5. definitely improve after a good meal

  6. and about 100 bags are sold each day

  7. when the school was awarded

Ответ

A

B

C

D

E

F

3

6

5

2

1

7

Task 57

Walking is not enough to keep fit

Walking may not be enough on its own to produce significant health benefits, research suggests. A team from Canada’s University of Alberta compared a 10,000-step exercise programme with a more traditional fitness regime of moderate intensity. Researchers found improvements A _______ were significantly higher in the second group. They told an American College of Sports Medicine meeting that gentle exercise was B __________. In total 128 people took C _________. The researchers assessed influence on fitness by measuring blood pressure and lung capacity. They found out the 10,000-step programme did help to get people motivated – and was an excellent way to start D _________. But to increase the effectiveness, some intensity must be added to their exercise. “Across your day, while you are achieving those 10,000 steps, take 200 to 400 of them at a faster pace. You’ve got to do more than light exercise and include regular moderate activity, and don’t be shy to have an occasional period of time at an energetic level.” The researchers were concerned there was too much focus E __________, rather than on its intensity.

Professor Stuart Biddle, an expert in exercise science at the University of Loughborough, said it was possible that the current guidelines on how much exercise to take were set too low. “However, you have got to find F ____. The harder you make it, the fewer people will actually do it.” Professor Biddle said there was no doubt that energetic exercise was the way to get fit, but volume rather than intensity might be more useful in tackling issues such as obesity.

  1. part in the project

  2. taking exercise

  3. gave marked health benefits

  4. in fitness levels

  5. on simply getting people to take exercise

  6. not enough to get fit

  7. a compromise between physiology and psychology

Ответ

A

B

C

D

E

F

4

6

1

2

5

7

Task 58

Double-decker Bus

A double-decker bus is a bus that has two levels. While double-decker long-distance buses are in widespread use around the world, A ____. Double-decker buses are popular in some European cities and in some parts of Asia, usually in former British colonies. Many towns around the world have a few that specialize in short sight-seeing tours for tourists because, as William Gladstone observed, «the way to see London is from the top of a ‘bus'».

Double-decker buses are taller than other buses. They are extensively used in the United Kingdom, B _____, removed from normal service in December 2005 — they still operate on heritage routes. Elsewhere in Europe, double-deckers are used throughout the Dublin Bus network in Ireland, where they are making a comeback on Dublin’s outer suburban routes and also the streets of Cork, Limerick, Galway and Waterford. They are a common sight in Berlin, where the BVG makes extensive use of them. Double-decker long-distance coaches are also in widespread use throughout Europe.

Most buses in Hong Kong and about half in Singapore are double-deckers as well. The only areas in North America that C _____ are the western Canadian province of British Columbia and the United States city of Las Vegas. They are currently being tested in Ottawa on the express routes. The city of Davis, California, in the United States uses vintage double-decker buses for public transport. Davis, California is also home to the first vintage double-decker bus converted from diesel gasoline to run on CNG. The city of Victoria, BC, the city of Vancouver, British Columbia, and a couple of others use Dennis Tridents. A few are also used as tour buses, especially in New York. Double-deckers are have also been used in Mumbai since 1937.

In Brazil, D _____, some companies use double-decker buses. Double-deckers are not a good option for use outside the towns (most roads in Brazil are in very poor condition), and E _____.

Double-decker buses are in widespread use in India in many of the major cities. Some double-decker buses F _____, with no roof and shallow sides. These are popular for sightseeing tours.

  1. double-deckers are adored by thousands of tourists

  2. use double-decker buses for public transport

  3. double-decker city buses are less common

  4. where perhaps the most famous was the London Routemaster

  5. their use is being discouraged by transportation authorities

  6. have an open upper deck

  7. where buses are sometimes the only interstate transport

Ответ

A

B

C

D

E

F

3

4

2

7

5

6

Task 59

Natural Links In a Long Chain of Being

I believe we are not alone.

Even if I am on the other side of the world from the farmhouse I live in, I still dream of the ancient vines out the window, and the shed out back that my grandfather’s father built in 1870 with eucalyptus trunks. As long as I can recreate these images, A ____

All of us need some grounding in our modern world of constant moving, buying, selling, meeting and leaving. Some find constancy in religion, others in friends or community. But we need some daily signposts that we are not different, not better, B ____

For me, this house, farm, these ancient vines are those roots. Although I came into this world alone and will leave alone, I am not alone. 

There are ghosts of dozens of conversations in the hallways, stories I remember about buying new plows that now rust in the barnyard and ruined crops from the same vines C ____

All of us are natural links in a long chain of being, and that I need to know what time of day it is, what season is coming, whether the wind is blowing north or from the east, and if the moon is still full tomorrow night, D _____

The physical world around us constantly changes, E _____. We must struggle in our brief existence to find some transcendent meaning and so find relief in the knowledge F _____.

You may find that too boring, living with the past as present. I find it refreshing. There is an old answer to every new problem, that wise whispers of the past are with us. If we just listen and remember, we are not alone; we have been here before.

  1. I never quite leave home

  2. but human nature does not

  3. that we are now harvesting

  4. but we as well as our heart did not

  5. not worse than those who came before us

  6. just as the farmers who came before me did

  7. that our ancestors have gone through this before

Ответ

A

B

C

D

E

F

1

5

3

6

2

7

Task 60

The Show Begins

My Uncle Jim took me to all the Broadway shows in New York City, and I was star struck! Actually he wasn’t my real uncle – that’s just what we called him. He was a close friend of my parents. He was a bit stocky with red hair, A _________.

I remember the theaters on Broadway, B __________. The curtains were made of this real heavy, dark red material. There were huge chandelier lights hanging from the ceiling. The walls were dark, paneled wood. The seats were red and cushy C __________.

The orchestra sat at the base of the stage in a pit. I usually went down to the front to see the musicians D __________. They were all crammed into such a tiny space. I played the flute myself and my dad kept encouraging me that if I kept it up, E ___________. But truly, I didn’t want to be tucked away down there. I wanted to be on top, front and center.

Most people dressed rather finely, and certain fragrances took center stage as various women passed by. The sounds of the audience F __________ at their seats were clearly heard while last minute patrons filled in. There was electricity in the air and then the lights would go down and up, and you knew it was time for the show to get started. The lights dimmed. The music began. And you were swept up into a whole new world. I loved it!

  1. I could be playing down there someday

  2. and set real close together

  3. which were so old and posh

  4. and he had a beard and moustache

  5. I wasn’t that good at music

  6. getting ready and warming up

  7. laughing and chattering away

Ответ

A

B

C

D

E

F

4

3

2

6

1

7

Task 61

Scouting moves ahead

The Scout Movement, which is also known as the Boy Scouts has changed massively in more than 100 years, though many people do not realise this.

For many people in Britain the word “scouting” evokes images of boys in short trousers A__________. Many people imagine that the Scout Association and its female counterpart the Guides Association are old-fashioned. They think these associations are for people B__________ than the future, people who just like camping in the rain and washing in cold water.

It’s quite easy to understand why Scouts and Guides have this sort of image. The “Boy Scouts” were founded over 100 years ago by Robert Baden-Powell, a retired English army general; the “Girl Guides” followed three years later. They were organised in an almost military manner. Young people had to learn discipline and how to do things as a group. They C__________ in difficult conditions, learnt to make campfires and, yes, they certainly had to get used to washing in cold water. In those days though, that D__________ many people washed in cold water.

Nevertheless, even at the start, there was much more to scouting than that. Scouts and Guides also learned the value of solidarity. Right from the start, they had to cope with difficult situations, E__________, and play a useful part in society. Baden-Powell’s organisations were inclusive, and never exclusive; any young person could become a Scout or a Guide, regardless of race, background or religion.

Though the Scout and Guide movements began in England, they soon spread to other countries, and within 50 years, scouting F__________ with young people all over the world.

  1. who are more interested in the past

  2. and girls in blue uniforms

  3. that were generally better

  4. was not particularly unusual as

  5. went on camping expeditions

  6. interact with other people

  7. had become a popular activity

Ответ

A

B

C

D

E

F

2

1

5

4

6

7

Task 62

Skip the sun, get a glow the healthy way

Everyone at some point has wanted a “healthy glow,” whether it’s a must-have for summer, or a vacation, the thought of tan skin has crossed the minds of millions. If you are pale, it A__________. There is wild excitement when after a day in the sun your skin is tan, not burnt. Surely everyone is familiar with the famous conversation upon the realization that you got fried at the beach. Your friends reassure you with “Don’t worry it B__________.” It may all seem like fun and games at the time, but alarming new research C__________.

Some tan-seekers do it the old-fashioned way, grab a towel and hit the pool or beach. Recently, millions of young girls D__________ instead. Regardless of how the tan is achieved, any change in skin coloring is evidence of skin cell damage. This can lead to cancer. According to the Skin Cancer Foundation, melanoma, or skin cancer, among people aged 18 to 39 has risen dramatically. In the United States the number of skin cancer cases due to tanning, is higher than the number of lung cancer cases due to smoking.

While it is true that being outside and active is great for your body and the sun does provide vitamin D, everyone’s health still needs protecting. However, it’s E__________, limit time spent in direct sunlight, between the hours of 10 a.m. and 4 p.m., and wear sunscreen at all times. A fashionable option is the sun hat: both elegant and fun. Big floppy hats may seem ridiculous at first, but F__________.

Another advice is to look into sunless tanners: They are cheap and in no way endanger the lives of users. So, fake it, don’t bake it!

  1. takes a lot of time and effort to tan

  2. have been turning to tanning beds

  3. they are actually quite classy accessories

  4. better to avoid indoor tanning

  5. have inspired people to get their skin checked

  6. will eventually turn into a tan

  7. has taken the healthy out of healthy glow

Ответ

A

B

C

D

E

F

1

6

7

2

4

3

Task 63

Grant-making agency

The National Endowment for the Humanities (NEH) is an independent grant-making agency of the United States government. Established in 1965, it is one of the largest sources of grant funds for humanities projects and programs in the U.S. NEH promotes knowledge of the history, thought, and culture, not only of the United States, A__________.

NEH grants facilitate research and original scholarship, strengthen teaching and learning in the humanities in American schools and colleges, give opportunities for citizens to engage in lifelong learning, B__________.

The Endowment is directed by a chairman, C__________ and confirmed by the U.S. Senate for a term of four years. Advising the chairman is the National Council on the Humanities, a board of 26 distinguished private citizens D__________ with the advice of the Senate. The National Council members serve six-year terms.

NEH grants are typically awarded to U.S. cultural institutions, such as museums, archives, libraries, colleges, universities, and public television and radio stations, E__________. Eligibility is limited to U.S. non-profit institutions and to U.S. citizens and foreigners F__________ prior to the time of application. Grants are awarded through a competitive process. The chairman takes into account the advice provided by the review process and, by law, makes all funding decisions.

  1. who is appointed by the president

  2. but of other countries of the world

  3. but in every aspect of social sciences

  4. who are also appointed by the president

  5. who have been living in the U.S. for three years

  6. as well as to individual scholars of the humanities

  7. as well as provide access to cultural and educational resources

Ответ

A

B

C

D

E

F

2

7

1

4

6

5

Task 64

The Bonfire Night

The 5th of November has always had a very special place in my heart. More important than New Year’s Eve, but probably less important than the Olympics ceremonies, the 5th of November every year is A __________ all over the country to go wild!

The night of the 5th is often cold and damp and parents wrap up their children in layers of jumpers, coats, hats, scarves and gloves. They fuss over the littlest B __________ aren’t scared. They comfort their pets and give them a safe place to curl up inside, away from the cacophony about to start outside.

Outside the bonfire is C __________ up your nose. If you’re lucky, there might be some pumpkin soup left over from Halloween to warm you up, because in spite of all the layers and the excitement, you’ll still need warming up until the bonfire gets going!

When it’s absolutely dark and the bonfire is blazing, the children and parents huddle together in groups, staring up at the sky. What are they waiting for? The screech of the first firework deafens them all and D __________. The “oohs” and “aaahs” of the crowd keep perfect time with the “kabooms” of the rockets. With every firework that lights up the sky, parents watch the delight grow on their children’s faces and sigh with relief.

After the grand finale, they make their way home with the noises still echoing in their ears. An extra special treat E __________! Waving them through the chilly air, spelling out names and drawing pictures, even the oldest members of the family remember how to be kids!

This is what the 5th of November means to me. Every year, it F __________ such bright and colourful fireworks and heard such loud bangs. I really hope I never grow out of it!

  1. differences in traditions

  2. children and hope that they

  3. the day for fireworks lovers

  4. the explosion lights up the sky

  5. feels like the first time I’ve seen

  6. waits at home though: sparklers

  7. lit and the smell of smoke creeps

Ответ

A

B

C

D

E

F

3

2

7

4

6

5

Task 65

Earth-sheltered homes

Earth-sheltered or simply underground homes are one of those creations by man, which brings him closer to nature. Unlike the normal traditional houses that A __________, these earth-sheltered homes are built using the shelter of the ground. Earth-sheltered homes can be easily made in hilly areas.

The basic idea behind the construction of such a house is that they are built with the idea of B ________ and each of these homes is built entirely different from each other.

The construction of these homes is usually done according to the shape of the area where the house is built. Their designs C ________ to the nature. The early earth houses which were initially built lacked windows. Modern day earth-sheltered homes though have windows as well as any other facility that the people living there might require.

Some of the major benefits of earth-sheltered homes are that they are naturally insulating. This makes them cool in the summer and cozy and warm in the winter. Another advantage D __________ and are well protected from earthquakes as well as wind-storms. Many earth-sheltered homes are also defended against intruders since there is usually only one entry.

As everything has its pros and cons, earth-sheltered homes also do. The interior decoration of these homes, like placing the furniture or huge paintings, E __________. These homes also have dark spaces inside and for this reason, lots of lighting is essential.

Earth-sheltered homes are one of the greenest housing designs that combines Mother Nature with eco-friendly F __________.

  1. are built on the ground

  2. are usually very organic

  3. is being built facing south

  4. being environmentally friendly

  5. building materials and lifestyle

  6. is that these homes are safe from fire

  7. can be difficult due to the construction

Ответ

A

B

C

D

E

F

1

4

2

6

7

5

Task 66

Australia

Australia is one of the world’s most ethnically diverse nations. Nearly a quarter of the people who live in Australia A __________. They come from the United Kingdom and other European countries, but also from China, Vietnam, North Africa, and the Middle East.

First people arrived in Australia about 50,000 years ago. They B __________ land bridges when sea levels were lower. The next to land in Australia were Dutch explorers. They came in 1606. In 1788 the British began to settle there. Many settlers C __________ as punishment. For a short time, the newcomers lived peacefully with the Aboriginal people.

In 1851, gold was discovered in Australia. A rush to find riches brought D __________ 1859, six separate colonies were created which later became part of the British Commonwealth.

Australian culture is founded on stories of battlers, bushrangers and brave soldiers. Today E __________ its Aboriginal heritage, vibrant mix of cultures, innovative ideas and a thriving ecosystem.

Australia’s ecosystem is an unusual one because of its remote location. As a result, there are F __________ and nowhere else in the world, such as kangaroo and koala.

One of Australia’s most amazing sites rises like an enormous whale’s back from a flat desert called the Red Center. It is a sacred natural formation at the heart of the country and the largest rock in the world!

  1. Australia is one of the most

  2. were born in other countries

  3. Australia also defines itself by

  4. many animal species that occur here

  5. may have travelled from Asia across

  6. thousands of new immigrants, and by

  7. were criminals sent to live in Australia

Ответ

A

B

C

D

E

F

2

5

7

6

3

4

Task 67

Living nature in Madeira

Right in the middle of the Atlantic, the islands of Madeira and Porto Santo are a haven of natural beauty. The exotic colours of the flowers stand out from among the blue sea and the emerald green vegetation. This is an archipelago where the big territory is a protected area and A __________ is located.

The Madeira Natural Park was created in 1982 to preserve this vast natural heritage, a worldwide rarity. The park is classified as a Biogenetic Reserve, B __________, with some rare species such as the mountain orchid, unique in the world, and also some exotic large trees. To visit this park is to discover Nature! The park covers about two-thirds of the island, making Madeira a truly ecological destination.

The springtime temperature, C __________, cries out for open air activities. Visitors can go for a walk in the park, visit the city of Funchal or roam freely around the island. Boat rides are an excellent way of D __________. In such a naturally welcoming environment, balance and well-being are taken for granted. Madeira offers various tourist complexes E __________.

Popular feasts, F __________, are opportunities to appreciate traditional gastronomic flavours and see Madeira partying, especially for the Carnival parades, the Flower festival, the Atlantic festival and, above all, the end-of-year fireworks display.

  1. which is felt all year round

  2. which take place in Madeira all year round

  3. where the largest laurel forest in the world

  4. admiring the coastline from a different perspective

  5. where one can find a unique range of flora and fauna

  6. choosing this holiday destination for its natural beauty

  7. that have prime conditions for boating and scuba diving

Ответ

A

B

C

D

E

F

3

5

1

4

7

2

Task 68

Wild animals in cities

Have you ever seen bears in Vancouver parks, leopards on the streets of Mumbai or wild pigs in gardens in Berlin? Recently, there A __________ on TV about big animals coming into towns and cities. What happens when wild animals come into our cities? Is it dangerous for us and the animals?

Wild animals usually come into cities to look for food. In Cape Town, South Africa, baboons sometimes come into the suburbs. They eat fruit from gardens and go into people’s kitchens and take food from cupboards and fridges! Baboons are B __________ children and fight with pet dogs. Many people do not like them, but the city can be dangerous for baboons too. Sometimes, baboons are C __________ human food can be very bad for their teeth. The city council in Cape Town has a team of Baboon Monitors whose job is to find baboons D ___________ to the countryside. This makes the city safer for people and is healthier for the baboons. However, the main problem is that a lot of baboons will come back to the city to find food again.

In Berlin, Germany, groups of wild pigs have come into the city for hundreds of years, but now the winters are warmer, there are even more pigs than in the past. Pigs eat flowers and plants and dig in gardens and parks in the city. They also E __________ accidents. Some city residents like the pigs and give them food. But the city council is worried about the traffic accidents, so they F __________ have put up fences to stop the pigs coming into the city.

  1. cause lots of problems

  2. in the city and take them back

  3. walk in the street and cause traffic

  4. hurt in car accidents and the sugar in

  5. strong animals and sometimes they scare

  6. have been many reports in newspapers and

  7. have told people to stop giving the pigs food and

Ответ

A

B

C

D

E

F

6

5

4

2

3

7

Task 69

Europe’s best hidden gems

There are incredible destinations in Europe known worldwide, such as Amsterdam and its canals, London and its museums, its shopping and atmosphere, or Paris, the City of Light. Europe also has thousands of hidden treasures. There is a wide selection of the finest unknown destinations in Europe, from Lugano in Switzerland  A __________.

Lugano is an international city, the crossroads and melting pot of European culture. It constitutes one of the most interesting regions to be discovered. Lugano is not only Switzerland’s third most important financial centre,  B __________ old buildings.

The area of Cinque Terre in Italy represents one of the best preserved natural sights of the Mediterranean. Human activity has contributed to creating a unique landscape in which the development of typical stone walls is so extensive C __________. All this, D __________, makes the Cinque Terre an increasingly popular location among Italian and foreign tourists.

Sintra is a jewel set between the mountains and the sea, waiting to be discovered by tourists E __________, luxuriant nature and cosmopolitan cultural offer. Sintra has a wonderful charm that left a deep impression on the soul and work of the writers F __________. Sintra is truly the capital of Romanticism. It is a place to be experienced by everyone!

  1. but showed evidence of an early human housing

  2. to Cinque Terre in Italy and Sintra in Portugal

  3. as to equal that of the famous Great Wall of China

  4. but also a town of parks and flowers, villas and

  5. who want to be lost in its majestic historical heritage

  6. combined with the beauty of a crystal clear sea

  7. who pioneered the Romantic spirit in the eighteenth century

Ответ

A

B

C

D

E

F

2

4

3

6

5

7

Task 70

Beautiful cities of Italy

The political and cultural centre of Italy is the ‘Eternal City’, Rome. Ever since the Roman Empire, as its capital, Rome has become famous as a centre of European culture. The most striking sights of Rome are, of course, the Colosseum and the Forum. Once the Colosseum was able to receive about 50 thousand spectators, A __________ and concert halls. The Pantheon, the old temple of all gods, B __________, is also located in Rome.

The second most important town in Italy is Milan. Milan is the capital of fashion and C __________, exhibitions and conferences. The main attraction of Milan is its Cathedral Square, where the monument to the King Victor Emmanuel II is installed. Theatre fans will not be left disappointed by visiting the Theatre of La Scala.

The most popular city among tourists is Venice. The city is unique because it has more than 120 islands, D __________ and 400 bridges. Venice has been known for more than fifteen hundred years, and for E __________. The main area of the city is Saint Mark’s Square with the Cathedral of San Marco. One of the most beautiful buildings in Venice is the Palace of Doges. The other famous attraction is the Grand Canal F______.

In addition to this, there are such beautiful cities in Italy as Naples, Turin, Florence, Genoa, Pisa and the islands of Sardinia and Sicily. All of them are outstanding places to visit.

  1. that is the largest in Venice

  2. which was built in the early I century

  3. that everyone is dreaming about this trip

  4. which is comparable with modern stadiums

  5. which are сonnected by more than 150 canals

  6. the venue for major international festivals

  7. that time it produced a lot of attractions

Ответ

A

B

C

D

E

F

4

2

6

5

7

1

Task 71

City of fountains

 Peterhof is a monument of world architecture and palace and park art. Peterhof includes a palace and park ensemble of the 18-19th centuries, A__________. Peterhof is a city of fountains as it contains 173 fountains and 4 cascades B__________. Each year up to 3 million people come here to enjoy the splendour of numerous fountains and the unique parks of Peterhof.

The name Peterhof was first mentioned in 1705. It was a coastal manor, close to which the construction of a grand country residence began. The original plan belonged to Peter the Great. After the brilliant victory of Russian troops over the Swedes, security of St. Petersburg both from the land C__________. Since that time, the construction of the Peterhof residence grew immensely in scope.

According to the plan of Peter the Great, on the one hand, Peterhof was meant to be equal in splendour with the most famous royal residences in Europe, D__________ to access the Baltic Sea. Both were successfully done. The Great Palace was built on a natural hill here, E__________. Following the plan of Peter the Great, F__________, the Grand Cascade with the famous Samson fountain was constructed. This is still one of the most spectacular fountains in the world. In 1990 the palaces and parks of Peterhof were included in the list of the world heritage of UNESCO.

  1. and from the sea has been firmly ensured

  2. which is a former royal countryside residence

  3. who designs many royal residences in Europe

  4. and then rebuilt in the baroque style in the 18th century

  5. who wanted to decorate the main entrance with waterfalls

  6. that are located in the park on the coast of the Gulf of Finland

  7. and on the other hand, to become a monument of Russia’s struggle

Ответ

A

B

C

D

E

F

2

6

1

7

4

5

Task 72

Sights of Sochi

 Sochi is unique among other Russian cities because it has many aspects of a subtropical resort. Apart from the scenic Caucasus Mountains, pebble and sand beaches, the city attracts tourists with its vegetation, numerous parks, monuments, and extravagant architecture. About two million people visit Greater Sochi each summer, A__________. The famous Caucasian Biosphere Reserve, B__________, lies just north from the city.

Popularity of Sochi among tourists is largely explained by the beauty of its surroundings. Walking along the river Agura, everyone will admire the nature around, C__________, and amazing waterfalls. From the bridge over the Agura opens a magnificent view to the lowest Agura waterfall. In the shady Agura gorge tourists will feel the gentle coolness, D__________.

Akhun Mountain the biggest in the region has a beautiful tower on the top. The height of the tower is more than 30 metres, E__________ are stunning. The observation platform on the top of the tower gives a chance to take superb pictures of the city. Every year thousands of people visit this stone tower, F__________ the perfect view of the Black Sea coast and the Caucasus Mountains. It is a truly unforgettable experience. Tourists will enjoy visiting all the sights and the resort itself, full of exotic green and the boundless blue of the Black Sea. 

  1. and the views that open from it

  2. which is built on the top to give visitors

  3. when the subtropical resort is almost empty

  4. which is a UNESCO World Heritage Site

  5. enjoying the sound of birds singing and waterfalls gurgling

  6. when the city is home to the annual film festival “Kinotavr”

  7. including high cliffs, exotic vegetation, breathtaking canyons

Ответ

A

B

C

D

E

F

6

4

7

5

1

2

Task 73

Saint Petersburg

 A city of palaces and museums, broad avenues and narrow canals, St. Petersburg’s short history is rich in architectural and artistic treasures. Alongside world-famous attractions such as the Hermitage and St. Isaac’s Cathedral, the city has a lot of equally interesting buildings A__________. St. Petersburg is considered to be Russia’s cultural capital. It reflects the country’s extraordinary fate like no other city.

St. Petersburg is a relatively young city, by both Russian and European standards, as B__________. Despite its short life, the city has a rich history. From the early days of Peter the Great to modern times, the city has always bustled with life and intrigue.

Lying across the delta of the Neva River, St. Petersburg, the Venice of the North, is a city C__________, some of which are well-known for their unique history. Bridges are an essential part of the city’s architectural make-up. Among the city’s over 500 bridges, there are numerous technological masterpieces. The centre of the city offers vast areas of green space, D__________.

St. Petersburg is a beautiful and fascinating holiday destination and one E__________. Whether to visit the city in a romantic and snowy Russian winter F__________, visitors will be spellbound by 
St. Petersburg’s culture and beauty. 

  1. that is built on hundreds of islands

  2. or during the dazzling white nights in summer

  3. it was only founded in 1703 by Tsar Peter the Great

  4. or considering a variety of the trip accommodation offers

  5. that reveal the mysterious and tragic genius of St. Petersburg

  6. of the most intriguing and historically significant cities in Europe

  7. including beautiful historic gardens and extensive leisure parks

Ответ

A

B

C

D

E

F

5

3

1

7

6

2

Task 74

State Hermitage Museum

 The Hermitage is St. Petersburg’s most popular visitor attraction, and one of the world’s largest and most prestigious museums. It is a must-see for all first-time travellers to the city. With over 3 million items in its collection, it also rewards repeat visits, A__________ of the riches on offer here, from Impressionist masterpieces to fascinating Oriental treasures. It was estimated B__________ on display for just one minute. So many visitors prefer a guided tour to ensure C__________ highlights. Art-lovers, however, may find it more rewarding to seek out for themselves the works D__________.

The bulk of the Hermitage collection is housed in the Winter Palace, E__________. However, there are a number of other sites that constitute part of the Hermitage, including the recently opened Storage Facility in the north of St. Petersburg. It offers guided tours through some of the museum’s vast stocks. The magnificent General Staff Building opposite the Winter Palace is most famous for its central triumphal arch, F__________ Nevsky Prospekt. The General Staff Building contains a number of unique exhibitions. It includes the Modern European Art, probably the most visited section of the Hermitage with well-known collections of Picasso and Matisse, as well as a wealth of popular Impressionist paintings. 

  1. that they are particularly interested in

  2. that they have time to catch all the collection’s

  3. and new-comers can only hope to get a brief taste

  4. which brings pedestrians out on to Palace Square from

  5. that one would need eleven years to view each exhibit

  6. which was the official residence of the Russian emperors

  7. and the exhibition was often visited by military historians

Ответ

A

B

C

D

E

F

3

5

2

1

6

4

Task 75

Letniy Sad

 Letniy Sad (The Summer Garden) is a park ensemble, a monument of landscape art of the 18th century. Letniy Sad is the oldest park in St. Petersburg. The park was founded by Peter I in 1704. The Tsar dreamed of his own Versailles and drew its original plan himself. He planned to create a regular, architectural park with accurate layout and straight paths. Prominent architects and gardeners took part in its creation. The park was supposed to become a place of relaxation, A__________.

Letniy Sad is surrounded by water. Natural boundary of the park from the north and east are the Neva and Fontanka Rivers, B__________.

Peter I brought sculptures from Italy for the park and was very proud of them. In the 18th century there were more than two hundred sculptures, C__________, or moved to suburban royal residences and the Hermitage. Now Letniy Sad is decorated with 90 sculptures – copies made of artificial marble.

In May, 2012 Letniy Sad was opened after reconstruction. The reconstruction work had been going on for about three years, D__________ Letniy Sad as it was in the 18th century. Among the new items in Letniy Sad, there is the Archaeological Museum, E__________ during the restoration of the park. Visitors can take a tour of the park F__________ on Sundays. 

  1. and restorers have done everything possible to keep

  2. combining the features of urban and suburban estates

  3. which are planned to be the centre of scientific research

  4. which contains interesting objects found by archaeologists

  5. but later many of them were either destroyed in the flood

  6. and enjoy the exhibitions and performances of a brass band

  7. and from the south and west – the Moika River and the Lebyazhiy Canal

Ответ

A

B

C

D

E

F

2

7

5

1

4

6

55

Опубликовано 3 года назад по предмету
Английский язык
от Кybako

  1. Ответ

    Ответ дан
    КотДилан

    прочитайте текст еще раз и определите правильное или неправильное предложение

  2. Ответ

    Ответ дан
    _gatha_145

    T
    f
    t
    f
    t

    ребят, делайте так. мне 5 поставили

Самые новые вопросы

Julia2101

Математика — 3 года назад

Решите уравнения:
а) 15 4 ∕19 + x + 3 17∕19 = 21 2∕19;
б) 6,7x — 5,21 = 9,54

na2005stud

Информатика — 3 года назад

Помогите решить задачи на паскаль.1)
дан массив случайных чисел (количество элементов
вводите с клавиатуры). найти произведение всех элементов массива.2)
дан массив случайных чисел (количество элементов
вводите с клавиатуры). найти сумму четных элементов массива.3)
дан массив случайных чисел (количество элементов
вводите с клавиатуры). найти максимальный элемент массива.4)
дан массив случайных чисел (количество элементов
вводите с клавиатуры). найти максимальный элемент массива среди элементов,
кратных 3.

Оксаночка1233

География — 3 года назад

Почему япония — лидер по выплавке стали?

Анимешка2

Математика — 3 года назад

Чему равно: 1*(умножить)х?     0*х?

laraizotova

Русский язык — 3 года назад

В каком из предложений пропущена одна (только одна!) запятая?1.она снова умолкла, точно некий внутренний голос приказал ей замолчать и посмотрела в зал. 2.и он понял: вот что неожиданно пришло к нему, и теперь останется с ним, и уже никогда его не покинет. 3.и оба мы немножко удовлетворим свое любопытство.4.впрочем, он и сам только еле передвигал ноги, а тело его совсем застыло и было холодное, как камень. 5.по небу потянулись облака, и луна померкла. 

Информация

Посетители, находящиеся в группе Гости, не могут оставлять комментарии к данной публикации.

К содержанию

Анатолий
Анатольевич
Eфремов

Раздел 3 — Языковой материал (задания по Грамматике и Лексике)

Прочитайте приведённый ниже текст. Преобразуйте слова, напечатанные заглавными буквами в конце строк, обозначенных номерами 18-26, так, чтобы они грамматически соответствовали содержанию текста. Заполните пропуски полученными словами. Каждый пропуск соответствует отдельному заданию 18-26.

18 I was happy to take part in the exchange programme and to go to college in Britain. An opportunity to spend two __________________ in London sounded fantastic. MONTH
Ответ: months
19 Our teacher told us that we __________ in a student hostel. LIVE
Ответ: would live
20 It was going to be __________ than staying with a host family as the hostel was very close to the college. CONVENIENT
Ответ: more convenient
21 When we arrived in London, the __________ thing we had was a bus excursion round the British capital. ONE
Ответ: first
22 Unfortunately, when we __________ Westminster Abbey, it started to rain hard. PASS
Ответ: were passing
23 We had to go down to the lower level of our double-decker and we __________ to enjoy the wonderful sights. NOT/MANAGE
Ответ: did not manage / didn’t manage
24 Anyway, London is very impressive. People from all over the world live there. Some of __________ come as tourists, others choose this place to study or to work in. THEY
Ответ: them
25 It is a dynamic multicultural city, though there __________ so many historical buildings in it. BE
Ответ: are
26 Amazingly, many of the houses __________ centuries ago! BUILD
Ответ: were built

8A8342

Прочитайте приведённый ниже текст. Преобразуйте слова, напечатанные заглавными буквами в конце строк, обозначенных номерами 27-32, так, чтобы они грамматически и лексически соответствовали содержанию текста. Заполните пропуски полученными словами. Каждый пропуск соответствует отдельному заданию 27-32.

27 My friend, Mary, is an extraordinary girl. She has always tried new things – new sports and new hobbies. She says life should be __________________ and it should change all the time. INTEREST
Ответ: interesting
28 Now she has a new idea – she wants to be a firefighter. You may say that it is not a job for girls, but Mary will __________ with this. AGREE
Ответ: disagree
29 In her opinion, putting out fires is a __________ job but women are able to do it too. DANGER
Ответ: dangerous
30 She has enrolled in a special training school where they learn skills and study firefighting __________. EQUIP
Ответ: equipment
31 The teenagers in the camp also learn how to work as a team, which is a very __________ skill wherever you work. USE
Ответ: useful
32 And Mary is sure her team will be effective in any __________. SITUATE
Ответ: situation

4B9B0D

За это задание ты можешь получить 4 балла. На решение дается около 8 минут. Уровень сложности: повышенный.
Средний процент выполнения: 79.4%
Ответом к заданию 2 по английскому языку может быть последовательность цифр, чисел или слов. Порядок записи имеет значение.

Разбор сложных заданий в тг-канале

Задачи для практики

Задача 1

Вы услышите диалог. Определите, какие из приведённых утверждений А–G соответствуют содержанию текста (1 – True), какие не соответствуют (2 – False) и о чём в тексте не сказано, то есть на основании текста нельзя дать ни положительного, ни отрицательного ответа (3 – Not stated). Запишите номер выбранного Вами варианта ответа. Вы можете прослушать запись дважды.

A. The test helps to assess students equally.

B. The scores will be sent by e-mail.

C. Michael asked three teachers to give him recommendation letters.

D. This is their last semester.

E. Michael wants to go to another state to study.

F. Charlotte wants to major in English.

G. Michael has decided to become a Biology teacher.

Решение

Michael: That test was so long! Four hours! I really do not understand why we have to take this test anyway. Are our grade point averages not good enough for college?

Charlotte: That test did seem long, didn’t it? But since the curriculum of one high school is slightly different from another high school, this test allows the colleges to consider each student equally. Although one student might have taken different classes from another student, all high school students take the same test.

Michael: I suppose that makes sense. Either way, I really hope I do not have to take that test again!

Charlotte: Same here. Well, we will just have to wait and see. The scores will be out in a few weeks. Anyways, have you gotten your recommendation letters yet?

Michael: I have asked Mrs. Smith for one already, and I am thinking about asking Mr. Johnson for another one. I liked both their classes and I think they got to know me better than my other teachers, so hopefully the letters will be well-written. What about you?

Charlotte: I have also asked Mrs. Smith for a letter. Do you know Mr. Lawson? I would like to get a recommendation letter from him. I hope he remembers me as I have not had a class with him since last year. I am not sure who else I could have asked.

Michael: Well, at least we have one letter each. Mrs. Smith is the best, isn’t she? I wish I could have another class with her, but this is our last semester, and after that it is graduation!

Charlotte: I agree. Mrs. Smith really enjoys teaching, and that definitely helps her students to enjoy learning.

Michael: What colleges are you thinking of applying to? Have you visited any campuses yet?

Charlotte: I am still considering whether I should stay close to home or go to an out-of-state school. I have toured the Stanford campus and the people there were very friendly. I have also visited Notre Dame, the university in Indiana. What about you? Have you toured any campuses yet?

Michael: I want to stay close to home so I am hoping to go to either a California State University or the University of California.

Charlotte: Which one? There is practically one or the other in each of the big cities.

Michael: Perhaps to the University of California. But I am not too worried about getting into a college just yet. I am more worried about this test at the moment!

Charlotte: I understand how you feel. However, we should still consider what should be done next. It will be bad if we fall behind and don’t make it into college by next fall. Have you decided what to study yet? I’m thinking about majoring in English myself.

Michael: English is definitely a possibility. I was thinking about becoming an English teacher like Mrs. Smith.

Charlotte: I do not think you have to worry about getting into college. Still, let’s just keep trying our best!

Ответ: 1321212

Задача 2

Вы услышите диалог. Определите, какие из приведённых утверждений А–G соответствуют содержанию текста (1 – True), какие не соответствуют (2 – False) и о чём в тексте не сказано, то есть на основании текста нельзя дать ни положительного, ни отрицательного ответа (3 – Not stated). Запишите номер выбранного Вами варианта ответа. Вы можете прослушать запись дважды.

A. The traffic was heavy this morning.

B. Mike is the Finance Department Manager.

C. The new employee should go to the bank every day.

D. The position requires a four-year college degree in Finance.

E. The new person will be trained on special courses.

F. Jane has just graduated with a Master degree in Finance.

G. Jane hopes that she will handle pressure.

Решение

Mike: Good morning, Jane. I am Mike.

Jane: Good morning.

Mike: How are you doing?

Jane: I am doing fine. Thank you.

Mike: How was the traffic coming over here?

Jane: I am so glad that the traffic was light this morning. No traffic jam and no accidents.

Mike: That is good. Jane, let’s start the interview. Are you ready?

Jane: Yes, I am.

Mike: First of all, let me properly introduce myself. I am the Finance Department Manager. As you know there is an open position in my department, and I need to fill this position as soon as possible.

Jane: Please, tell me a little bit about the position.

Mike: It is an entry-level position. The new employee will have to work closely with the Accounting department. He will also have to deal with the bank on a daily basis.

Jane: What type of qualifications do you require?

Mike: I require a four-year college degree in Finance. Some working experience would be helpful.

Jane: What kind of experience are you looking for?

Mike: Doing office work is good. However, since this is an entry-level position, I do not require a lot of experience. I am willing to train the new person.

Jane: That is great!

Mike: Jane, tell me a little bit about yourself.

Jane: I was a student at West Coast University, and I just graduated with a Bachelor degree in Finance. I have been working part-time as a payroll clerk for the last two years.

Mike: What are you looking for in a job?

Jane: The job should help me see what Finance is all about. I have learned a lot of Finance theories at school, and now it is time for me to put them into practice.

Mike: Anything else?

Jane: I also hope that it will help me grow in my field.

Mike: What are your strengths? Why should I hire you?

Jane: I am a hard-working person and a fast learner. I am very eager to learn, and I get along fine with people.

Mike: OK. Now, let me ask you a few quick questions. You do not mind working long hours, do you?

Jane: No, I do not.

Mike: Can you handle pressure?

Jane: Yes, I can. When I was going to school, I took quite a few courses each semester while working at least twenty hours every week. And, I handled that situation very well.

Mike: Do you still have any questions for me?

Jane: No, I think I have a pretty good understanding of the job. I believe that I can handle it with ease, and I hope to have the opportunity to work for you.

Mike: Jane, nice meeting you. Thank you for coming.

Jane: Nice meeting you too. Thank you for seeing me.

Ответ: 2131321

Задача 3

Вы услышите диалог. Определите, какие из приведённых утверждений А–G соответствуют содержанию текста (1 – True), какие не соответствуют (2 – False) и о чём в тексте не сказано, то есть на основании текста нельзя дать ни положительного, ни отрицательного ответа (3 – Not stated). Запишите номер выбранного Вами варианта ответа. Вы можете прослушать запись дважды.

A. The new down town fitness club will open next week.

B. The fitness club is well equipped.

C. All the machines help to lose weight.

D. Ted is afraid because he can break the machines.

E. The guidance on the equipment usage is exciting.

F. There is only one class in the gym.

G. There are too many people in the fitness club.

Решение

Helen: Hey, Ted, I’m looking for a gym. Actually, I hear you’ve joined a gym. Which one?

Ted: Yeah, actually, there’s a nice gym down town. It just opened about a month ago. Maybe two months ago. It’s really nice.

Helen: OK, what’s the name of it?

Ted: The name is Star Fitness Club.

Helen: Good name.

Ted: Yeah, so, and it’s pretty much has everything. I mean it has, you know, free weights, of course, and it has all the latest machines.

Helen: Ah, good.

Ted: Actually some of the machines are complicated. I really don’t know how they work yet. I’ll have to ask the staff. I’m actually afraid to get on the thing. I don’t want to break it, you know.

Helen: Are there people that can help you?

Ted: Yeah, actually, the part of the problem is that you can’t use any of the equipment unless you get trained for it. They’re really specific, so you have to have guidance. It’s kind of annoying actually.

Helen: OK. But what happens if I’ve been to a gym before? Do I still need to get the training before I start?

Ted: Yeah. That’s how I was. I told them that I, you know, had been lifting weights for a while and they didn’t care.

Helen: OK, so, do they have classes though?

Ted: They do. Actually, they pretty much have everything. They have yoga, kick boxing, spin classes, dancing, so the schedule looks pretty diverse.

Helen: Oh, that’s good. I do prefer classes than the weights.

Ted: Only one thing that’s bad about the classes though is that because it’s new, and it’s a new gym and there are lots of people and it looks like it’s pretty crowded in there, I think you have to reserve what classes you’re going to join.

Helen: Oh, really.

Ted: Yeah, isn’t that terrible.

Helen: Yeah, when you feel like going to the gym, you go.

Ted: Right, right. One thing that is nice though is they have an actual gym floor, like a basketball gym, so you can play basketball. They have volleyball tournaments, so I mean, if you like more traditional sport, like tennis, they also have tennis courts. You can do that as well.

Helen: Lovely. OK. So, yeah, I would love to come and have a look at it.

Ted: OK, well next week, anytime, just give me a call. I’ll take you there.

Helen: Alright. Great. Thanks.

Ответ: 2131221

Задача 4

Вы услышите диалог. Определите, какие из приведённых утверждений А–G соответствуют содержанию текста (1 – True), какие не соответствуют (2 – False) и о чём в тексте не сказано, то есть на основании текста нельзя дать ни положительного, ни отрицательного ответа (3 – Not stated). Запишите номер выбранного Вами варианта ответа. Вы можете прослушать запись дважды.

A. English is Ben’s first language.

B. Ben admired his teacher’s fluency in English.

C. Susan learnt a lot of words when she read in English.

D. The books, Ben had at home, were bought by his grandparents.

E. Susan enjoyed her study of English.

F. Ben didn’t help his classmates with English tasks because of difficulties.

G. Susan’s interest in English got bigger when she made new friends from Great Britain.

Решение

Susan: So Ben, you also learned English as a foreign language, right?

Ben: That’s right, yes.

Susan: It’s not your first language? So how did you do it?

Ben: That’s a good question. When I think back, I think, I try to think about the things that helped me most and I think it was a funny combination of things. I had one really interesting English teacher, she was really strict, but she was really interesting and I think she inspired me because her English was so good and I thought ‘wow’ that’s really great. The other thing that really helped me was reading like I really read so much in English. I think you’ve mentioned it too.

Susan: Yeah, I think so too. I read a lot, not because I wanted to learn but because I like reading and so I read in English and later on I realized I must have learned a lot of vocabulary and things that I didn’t even realize I learned.

Ben: I agree with you.

Susan: Just through reading.

Ben: Yeah, the same thing happened to me. My home had so many books in it, I was lucky I guess in that way and at least half of them were in English and when I discovered that there were books in the house that I couldn’t get into and I was really into stories and reading, I felt so angry then I started reading so much and it was really good for me in the end.

Susan: That’s interesting. You see people ask me how I learnt English. I didn’t really have a plan, I just enjoyed English that’s why I studied in my free time but I didn’t think of it as something I have to do, I just did it for fun.

Ben: Also in high school I remember my classmates complaining about all the homework in English and they couldn’t do it and they used to come to me for help. It didn’t feel like work to me. It just felt like something kind of fun.

Susan: That’s right.

Ben: But I can’t tell you where that fun started. I guess it has something to do with being curious about things that are written or said. Do you think so?

Susan: Yeah, I also remember when I first became interested in English but later on when I met some people from different countries, I became more interested in communicating with them and, like you said, my teacher also inspired me. I enjoyed just chatting to her in English.

Ответ: 2113123

Задача 5

Вы услышите диалог. Определите, какие из приведённых утверждений А–G соответствуют содержанию текста (1 – True), какие не соответствуют (2 – False) и о чём в тексте не сказано, то есть на основании текста нельзя дать ни положительного, ни отрицательного ответа (3 – Not stated). Запишите номер выбранного Вами варианта ответа. Вы можете прослушать запись дважды.

A. Mary’s parents live far away.

B. James is looking for the most low-cost place.

C. James is looking for a rather big and comfortable place in the center.

D. Mary can work full-time during the term.

E. Mary has been looking for a flat for a week.

F. The price depends on the distance from the school.

G. James has a car so he can find something far from school.

Решение

Mary: Hey, James. What are you doing here?

James: I am looking for an apartment to rent. It is a real problem for me because I cannot make a decision where to live. All my friends have decided to live at the school dormitory but I want to try and find an apartment. What about you? What are you doing here? Looking for an apartment also?

Mary: Yes. Since my parents’ house is so far away, I need to find an apartment closer to school. I do not like to use public transport and as you know it is rather expensive to go by bus every day. But I thought you were going to stay at the school dormitory.

James: I still have not decided whether to stay at the dormitory or not. I am looking at different options to find the cheapest lodging. I hope to find the most convenient one but now I understand that it is rather hard.

Mary: So, what are you looking for?

James: All I need is a place big enough for my bed, my desk and my television. Of course, the place should have a kitchen so that I can cook my meals. I will be living on a very tight budget and will have to watch every dime.

Mary: Me too. I cannot work full-time like I did during the summer. I will cut down on my workload in order to spend most of the time on my studies. So, a safe and decent apartment is all I need.

James: How long have you been looking for it?

Mary: I’ve just started this week. Since school is going to start next month, I figured I better start the process as soon as possible. I hope that I could find something convenient and rather cheap.

James: It is not easy to find an apartment to your liking that does not cost a lot. I have been looking at the ads in the newspaper for two weeks, and I still have not found anything yet. I also have tried to search on the Internet but there are few ads and they sometimes offer fake apartments.

Mary: Really? Is it that difficult to find an apartment?

James: No, it is just that everything I like so far is too expensive and way beyond my reach.

Mary: Is it because they are very close to school? I heard that the closer they are to school, the higher the rental cost.

James: Maybe that is the problem. Since I do not have a car, I need to find something close to school.

Mary: Have you thought about sharing an apartment? That could solve our problem.

James: It seems that sharing an apartment may work. Do you want to try it?

Mary: Yes. It’s a wonderful idea.

Ответ: 1132112

Задача 6

Вы услышите диалог. Определите, какие из приведённых утверждений А–G соответствуют содержанию текста (1 – True), какие не соответствуют (2 – False) и о чём в тексте не сказано, то есть на основании текста нельзя дать ни положительного, ни отрицательного ответа (3 – Not stated). Запишите номер выбранного Вами варианта ответа. Вы можете прослушать запись дважды.

A. Going to the cinema is a good idea.

B. Summer Pizza House sold a new kind of pizza with pepper and tomatoes.

C. Sam and Laurie will meet at 2 o’clock at the cinema.

D. Laurie’s cousin is in town now.

E. Laurie’s cousin will start her teaching career in the English school.

F. Laurie’s cousin knows how to teach young children.

G. Karen will show them some of the things for Christmas.

Решение

Laurie: So, what are your plans for this weekend?

Sam: I don’t know. Do you want to get together or something?

Laurie: How about going to see a movie? Cinemax 26 on Carson Boulevard is showing Enchanted.

Sam: That sounds like a good idea. Maybe we should go out to eat beforehand.

Laurie: It is fine with me. Where do you want to meet?

Sam: Let’s meet at Summer Pizza House. I have not gone there for a long time.

Laurie: Good idea again. I heard they have just come up with a new pizza. It should be good because Summer Pizza House always has the best pizza in town.

Sam: When should we meet?

Laurie: Well, the movie is shown at 2:00PM, 4:00PM, 6:00PM and 8:00PM.

Sam: Why don’t we go to the 2:00PM show? We can meet at Summer Pizza House at noon. That will give us plenty of time to enjoy our pizza.

Laurie: My cousin Karen is in town. Can I bring her along? I hate to leave her home alone.

Sam: Karen is in town? Yes, bring her along. We met her at Sara’s high school graduation party two years ago. She is such a nice person, and funny too.

Laurie: She will be happy to meet you again.

Sam: What is she doing these days?

Laurie: She graduated last June, and she will start her teaching career next week when the new school term begins.

Sam: What grade is she going to teach?

Laurie: She will teach kindergarten. She loves working with kids, and she always has such a good rapport with them.

Sam: Kindergarten? She must be a very patient person. I always think kindergarten is the most difficult class to teach. Most of the kids have never been to school, and they have never been away from mommy for long.

Laurie: I think Karen will do fine. She knows how to handle young children. I think the first few weeks will be tough. However, once the routine is set, it should not be too difficult to teach kindergarten.

Sam: You are right. The kids might even look forward to going to school since they have so many friends to play with.

Laurie: There are so many new things for them to do at school too. They do a lot of crafts in kindergarten. I am always amazed by the things kindergarten teachers do.

Sam: Yes, I have seen my niece come home with so many neat stuff.

Laurie: Maybe we can ask Karen to show us some of the things that we can do for this Halloween.

Sam: Maybe we can stop by the craft store after the movie. What do you think, Laurie?

Laurie: I will talk to her. I think she will like that. It will help her with school projects when Halloween comes.

Ответ: 1321213

Задача 7

Вы услышите диалог. Определите, какие из приведённых утверждений А–G соответствуют содержанию текста (1 – True), какие не соответствуют (2 – False) и о чём в тексте не сказано, то есть на основании текста нельзя дать ни положительного, ни отрицательного ответа (3 – Not stated). Запишите номер выбранного Вами варианта ответа. Вы можете прослушать запись дважды.

A. The car is very expensive.

B. Alex wants to buy a car this month.

C. Alex attended musical concert in the park last year.

D. The tradition started 10 years ago.

E. Jane listens to music every morning.

F. Jane wants to sit in the shade.

G. Jane will give Alex a piece of cream cake tomorrow.

Решение

Alex: Hey Jane, John, there is a musical concert in the park. You want to go and see the band play?

Jane: I am done with my homework; I can go. Hey, look at that sport car. Isn’t it neat?

Alex: That is exactly the kind of car that I want once I get a good job. I bet it is very fast. I want mine to be red though.

Jane: Keep on dreaming, Alex. That car costs a fortune.

Alex: It does not hurt to set high expectations. Maybe one day I will make a lot of money, and I might surprise you.

Jane: Talking about cars, why is the traffic so heavy today?

Alex: People are probably heading toward the park for the concert. The band does play pretty good music. For the last three years, I have never missed the concert. I have always arranged my schedule so that I could attend the event once the band was in town.

Jane: How long ago did the band start playing at our park?

Alex: I think it started this tradition five years ago before you moved into our neighborhood. Every year it always arrives the first week of June to play for the whole week. You will enjoy this evening, Jane. There will be good Country music.

Jane: It sounds like fun. My favorite is Rock and Roll music; however, I have to say that country melodies can be quite enticing. I can listen to them all day long. Alex, what kind of music do you like?

Alex: Oh, I like all kinds of music as long as it is not Hard Rock.

Jane: Wow, look at the number of people who have already shown up for the concert. Good thing that we are here already.

Alex: Jane, where do you want to sit? In the shade or in the sun?

Jane: In the shade, please. I have been in the sun too much lately.

Alex: There is a food stand over there. Do you two want anything?

Jane: Nothing for me, thanks. I already have my bottle of water.

Alex: I want a bag of chips and a soda. Are you sure you do not want any chips, Jane?

Jane: I am quite sure. Besides, my mother is cooking a good steak dinner, and I want to save my appetite.

Alex: Jane, you are so lucky to have such a good cook for a mother.

Jane: We will have cream cake for dessert this evening. I will save you a piece if you want, Alex.

Alex: I always love cake, so please save me a piece. Thanks, Jane.

Jane: OK, I will save a piece of cream cake for you tonight. We can meet at the cafeteria at lunch time tomorrow and I will give it to you.

Ответ: 1212311

Задача 8

Вы услышите диалог. Определите, какие из приведённых утверждений А–G соответствуют содержанию текста (1 – True), какие не соответствуют (2 – False) и о чём в тексте не сказано, то есть на основании текста нельзя дать ни положительного, ни отрицательного ответа (3 – Not stated). Запишите номер выбранного Вами варианта ответа. Вы можете прослушать запись дважды.

A. One of speakers is going to the zoo on Friday.

B. John’s child wants to become a doctor.

C. John is seek because of sweets.

D. Ann loves hot chocolate.

E. Ann thinks that children find junk food themselves.

F. John appreciates people to give sweets to his children.

G. John thinks it’s better to ask parents about treats for their kids.

Задача 9

Вы услышите диалог. Определите, какие из приведённых утверждений А–G соответствуют содержанию текста (1 – True), какие не соответствуют (2 – False) и о чём в тексте не сказано, то есть на основании текста нельзя дать ни положительного, ни отрицательного ответа (3 – Not stated). Запишите номер выбранного Вами варианта ответа. Вы можете прослушать запись дважды.

A. The action takes place in the early winter.

B. A man likes the cold weather.

C. It is going to rain the rest of the week.

D. Girl’s locker is right in the classroom.

E. Heroes liked the weather in autumn.

F. California has extremely bad weather conditions.

G. There is great amount of snow every winter in Caliornia.

Задача 10

Вы услышите диалог. Определите, какие из приведённых утверждений А–G соответствуют содержанию текста (1 – True), какие не соответствуют (2 – False) и о чём в тексте не сказано, то есть на основании текста нельзя дать ни положительного, ни отрицательного ответа (3 – Not stated). Запишите номер выбранного Вами варианта ответа. Вы можете прослушать запись дважды.

A. Kate is not going on holiday this year because she’s trying to save money.

B. Rick’s idea is to teach Kate to save money before and while travelling.

C. Looking for extras is a good idea.

D. Сheck at a restaurant doesn’t usually include a tip.

E. Rick advises to stay in hotels right outside the city center.

F. Kate is going to book in a business minded hotel.

G. Kate is a rewards member.

Решение

Kate: Hey, Rick! I know you are keen on travelling! Would you mind giving me some tips how I can increase my hotel savings? Does it seem like hotel costs just go up and up? It’s true that they rise each year, sometimes significantly.

Rick: Hey, Kate, sure, with pleasure. I’ll tell you some factors to consider to help you save money, both before you book and during your stay.

Kate: So, what shall I pay attention to?

Rick: The more services your rate includes, the better You’ll want to make sure they are features you will actually use though, otherwise you’re spending more than you need to. Common extras that can save you lots are free breakfast, free Wi-Fiand kids eat free. All those bowls of fruit and free coffee and tea in the lobby are also included in your room rate, so take advantage of them during your stay.

Kate: I don’t like the breakfast to be included. I’d like to eat out, but on the other hand I should be trying to save money.

Rick: I don’t agree with you. Though room service sounds really appealing, prices for food on the menu can be twice as much as you would pay for the same food at a restaurant. Gratuity is always included in the tab and usually masked as a delivery charge. Most guests don’t know this and add a tip to the bill when the waiter brings their food.

Kate: I see. To say the truth, I haven’t chosen the hotel yet.

Rick: What about the Rozy Hotel? If the costs for transportation won’t make up the difference you’d be paying on a closer location, you can frequently get a fantastic deal this way. You could also look into staying in the university district of a city. Hotels are more affordable, cheap eateries are plentiful and public transportation is convenient.

Kate: Actually, it’s not a business trip. I’m going to have some rest and devote time to myself in a calm and peaceful atmosphere.

Rick: Room rates for hotels that get the majority of their business from professionals drop their prices when business is slow, like weekends and holidays.

Kate: Moreover, business hotels provide mini-bars in the apartments. They also have nice lounge-bars.

Rick: As for me, I usually bring my own snacks or hit the grocery store. I think, anything is better than the rates for items in the mini bar. The prices are outrageous and I never think about trying to replace it with the same item later.

Kate: Do you normally stay at the same hotel or use the same chain when you travel?

Rick: Sure! I sign up for their rewards programme. Not only can you rack up points for each stay that you can then use on upgrades and free nights in the future, but rewards members also get notified of exclusive deals that can save you a ton. You can sign up online!

Kate: Nice idea! Thank you!

Rick: My pleasure, Kate.

Ответ: 3112132

Задача 11

Вы услышите диалог. Определите, какие из приведённых утверждений А–G соответствуют содержанию текста (1 – True), какие не соответствуют (2 – False) и о чём в тексте не сказано, то есть на основании текста нельзя дать ни положительного, ни отрицательного ответа (3 – Not stated). Запишите номер выбранного Вами варианта ответа. Вы можете прослушать запись дважды.

A. To feel more confident John should understand the subject matter in details.

B. John has done his research in the library.

C. John has not prepared an outline yet.

D. John doesn’t have pictures to persuade people about the dangers of global warming.

E. John doesn’t know how to be at ease during his presentation.

F. Considering the presentation of the point of view to the friends adds stress.

G. John will give his first presentation in school conference.

Решение

John: I will have to give a presentation on global warming on Friday, and I am so nervous.

Mary: There are a lot of things you can do to make you feel more confident and less nervous.

John: What should I do, Mary?

Mary: First of all, you need to understand the subject matter thoroughly. You need to know what is global warming, what causes global warming, and what people should do to abate the effects of global warming.

John: I have done a lot of research on the subject, and I know I can answer any questions I will receive from the audience.

Mary: The next thing that you need is an outline of your presentation. You should think about how to effectively present the subject matter.

John: You mean what I should talk about, or more precisely the sequence of my presentation?

Mary: Yes, what you should present first, second, third…

John: If that is the case, then I already have an outline. To make it easy for my audience to follow the presentation, I intend to post the outline on the board at all time during my speech.

Mary: Good idea! By the way, do you have any facts to back you up? For example, change of climate, yearly disasters…

John: No, I have not thought about that. I better get some statistics from the Internet. I should not have any problems since the Internet has all kinds of data.

Mary: Good. It is easier to convince people and to hold their attention with actual data. It would be even better if you show some pictures along the way. Do you have any?

John: No, it is another thing to add to my To Do list. I guess I will need at least two or three pictures to persuade people about the dangers of global warming.

Mary: Pictures will keep your audience from being bored. In order for you to succeed, you need to keep them interested and involved.

John: What else do I need? Is there anything else I can do to help me relax and be at ease during my presentation?

Mary: You need to practice your presentation. Just pretend that you are standing in front of your audience and start to give your speech.

John: Pretending is one thing; actually giving a speech is another thing.

Mary: Think positive. Tell yourself that you can do it without any problems.

John: I guess I can look at this as a presentation of my point of view to my friends. I mean a lot of my friends. I mean all the friends that I have made since kindergarten, plus all the relatives that I have on both my mother and father’s sides.

Mary: If you are really prepared, it will be a piece of cake. You will be able to speak with ease and confidence, and you will be amazed by how well you can express yourself.

John: I need to do this presentation really well. This is my first presentation in front of a big audience, and it is so important to me.

Mary: This is only the beginning, John. Being able to express your ideas with confidence and clarity is one of the best skills to possess.

John: You are absolutely right. I will take time to practice and to learn to relax and express myself really well. Wish me luck, Mary!

Mary: I know you. You can do it. Good luck, John!

Ответ: 1321123

Задача 12

Вы услышите диалог. Определите, какие из приведённых утверждений А–G соответствуют содержанию текста (1 – True), какие не соответствуют (2 – False) и о чём в тексте не сказано, то есть на основании текста нельзя дать ни положительного, ни отрицательного ответа (3 – Not stated). Запишите номер выбранного Вами варианта ответа. Вы можете прослушать запись дважды.

A. Helen is solving a Maths problem.

B. Tim advises Helen to revise some material before the session.

C. Tim needs to build a dollhouse for his sister.

D. Helen’s got a lot of experience in building.

E. Tim thinks it’s not necessary to put in windows and doors.

F. Tim knows that Nancy likes all kinds of doll houses.

G. Helen wants Tim to find a good shop and buy a new doll house.

Решение

Tim: Helen, what are you doing?

Helen: I have been trying to solve this physics problem for the last half hour, and I still have no idea how to do it.

Tim: When do you have to turn it in?

Helen: It is due at the end of this week.

Tim: Well, it is only Monday. Why don’t you get some after-schooltutoring tomorrow?

Helen: I have to sign up for it first. I guess I will go sign up for the Wednesday session tomorrow. You should reread the chapter before you show up for the session. It will help you understand the subject matter better.

Helen: OK, I will do that.

Tim: Now that your problem is solved, I need you to lend me a hand with my problem.

Helen: What is up?

Tim: I need to build a new doll house for Nancy. I was putting her doll house away, and somehow I accidentally dropped it. It was broken into pieces.

Helen: How clumsy of you! Does she know?

Tim: I told her about it. I could not lie to her.

Helen: How did she take it?

Tim: My sister really liked that doll house, and obviously she was not very happy. But, I told her that I would build a better one for her. So, I need your help.

Helen: Me? Help you? I have never built anything in my life.

Tim: Me neither. But, don’t worry. Putting together pieces of wood is not going to be difficult.

Helen: It is more than just putting pieces of wood together. You need to put in windows and doors. Have you thought about how you would do that?

Tim: Who says houses need to have windows and doors?

Helen: So, you are going to build a doll house with no windows and no doors?

Tim: Yes, I do not see anything wrong with that. Do you?

Helen: No. But, are you sure that Nancy will not mind having a doll house with no doors and no windows?

Tim: No, I am not sure. What should I do then?

Helen: Get help from somebody who knows how to build a doll house. I would not be of any help to you in this project.

Tim: Who do you suggest I ask?

Helen: Mr. Brown used to be a carpenter. He will be glad to help you out.

Tim: You are right. Thanks for your advice.

Ответ: 2112132

Задача 13

Вы услышите диалог. Определите, какие из приведённых утверждений А–G соответствуют содержанию текста (1 – True), какие не соответствуют (2 – False) и о чём в тексте не сказано, то есть на основании текста нельзя дать ни положительного, ни отрицательного ответа (3 – Not stated). Запишите номер выбранного Вами варианта ответа. Вы можете прослушать запись дважды.

A. Mark doesn’t mind going shopping.

B. Laura wants to have some cereal for breakfast.

C. Mark needs to buy some beef.

D. Laura asks Mark to buy some tomatoes.

E. Laura likes apples very much.

F. The girls’ mother prefers cherry vanilla ice-cream.

G. Mak is going to use public transport to go shopping.

Решение

Laura: Can you go shopping, Mike?

Mike: I guess I can. I am done with my homework. What do you want me to buy?

Laura: Well, buy some groceries, meat and vegetables, you can buy something for snacks and breakfast.

Mike: What do you want for breakfast?

Laura: I guess some cereal as usual.

Mike: I do not want cereal every day. I will buy some pancakes and syrup then.

Laura: Get the new FineFood pancakes in the frozen food section please. I want to see how it tastes.

Mike: Do we still have enough coffee and cream for mom and dad?

Laura: Yes, we do.

Mike: Next, what do you want for snacks?

Laura: Some chips would be fine with me. You probably want your chocolate cookies.

Mike: I better write down all these things; otherwise, I will forget them.

Laura: Right! As far as meat, mom wants some pork and some chicken.

Mike: Just any kind of pork?

Laura: I forgot to ask mom about that. Anyway, you can ask the butcher for his opinion. He knows what is best.

Mike: Now, how about vegetables? Buy carrots, potatoes, cabbages and some lettuce.

Laura: That would be fine. We need some fruits also. What do you want?

Mike: Dad always packs an apple for lunch every day. I want the same thing.

Laura: We have already run out of eggs. So, get a carton of eggs also.

Mike: How about some soft drinks?

Laura: No, buy some bottled water instead. It is healthier for us. We need to cut down on our intake of sugar, as too much sugar is not good for our bodies.

Mike: I need to put bread on the list before I forget. Should I get one loaf or two?

Laura: One loaf will be enough.

Mike: Do we need anything for dessert?

Laura: Get a box of ice cream.

Mike: Ice cream? Isn’t ice cream packed with sugar?

Laura: Yes, but since I already drink water instead of soft drinks, one or perhaps two scoops of ice cream after dinner is not too bad. Besides, mom loves ice cream, especially cherry vanilla.

Mike: OK, I will put down one box of ice cream if you say so.

Laura: Drive carefully please!

Mike: I will.

Ответ: 1122312

Задача 14

Вы услышите диалог. Определите, какие из приведённых утверждений А–G соответствуют содержанию текста (1 – True), какие не соответствуют (2 – False) и о чём в тексте не сказано, то есть на основании текста нельзя дать ни положительного, ни отрицательного ответа (3 – Not stated). Запишите номер выбранного Вами варианта ответа. Вы можете прослушать запись дважды.

A. Mimi’s Café is famous for its burgers.

B. Susan is planning to work during the vacation.

C. Susan changed her job because she didn’t like the place and the people.

D. Brendan likes the idea of working during holidays.

E. Brendan and his friends are planning to go snowboarding.

F. They will rent out a cabin.

G. Susan will try to go with Brendan.

Решение

Brendan: Hey, Susan, throw your stuff in the back and ride up front with me.

Susan: OK, Brendan. Thank you for giving me a ride home. I did not want to call my parents because they were at Mimi’s Café for my aunt’s birthday.

Brendan: Mimi’s Café? That place has the best burgers in town! The atmosphere is also nice. By the way, do you have any plans set up for the upcoming vacation?

Susan: Well, besides going to basketball practices with you, I will most likely be working.

Brendan: Oh, working? Did you get a new job or are you still working at Starbucks?

Susan: Well, Starbucks was a good first job for me and I loved the people I worked with. However, my work schedule was neither convenient nor easy.

Brendan: Well, what are you doing now at your new job?

Susan: Well, I am an Administrative Assistant for a company called Multi-Max. I’m busy inputting data into the computer and following up on customers’ orders.

Brendan: Oh, that sounds pretty easy.

Susan: Well what about you? Do you have any plans for break?

Brendan: Not really. I think I am just going to hang out at home and read some books. I am planning a snowboarding trip though.

Susan: Oh, that sounds like fun! Who is going and where are you guys going to go snowboarding?

Brendan: As of right now, my roommates, Jay and Kristian, are planning to go. And, there are Josh, Alexis, and Ann. We are planning to go to Big Bear. I hear that there are pretty good conditions for skiing and snowboarding this season up there.

Susan: Yes, that is what I heard as well. The whole trip sounds like fun. Where are you guys staying at? Did you rent out a cabin?

Brendan: No, we did not. My cousin actually owns a cabin up there, and he said it is OK for us to stay at his place for the weekend when I spoke with him last Friday.

Susan: Would it be OK with your cousin if I come along as well?

Brendan: I am sure that will be fine. He did not tell me there is a limit on the number of people I could bring, so you are more than welcome to join us.

Susan: Awesome! I am so excited! Well, I will check my schedule, and I will give you an update tomorrow. Hey, that is my house on the corner. Thanks again for the ride, Brendan!

Brendan: You are welcome, Susan. Have a good night and I will talk to you later about the trip.

Ответ: 1123121

Задача 15

Вы услышите диалог. Определите, какие из приведённых утверждений А–G соответствуют содержанию текста (1 – True), какие не соответствуют (2 – False) и о чём в тексте не сказано, то есть на основании текста нельзя дать ни положительного, ни отрицательного ответа (3 – Not stated). Запишите номер выбранного Вами варианта ответа. Вы можете прослушать запись дважды.

A. Dan was a success at school.

B. Mrs. Anderson is fond of baking.

C. Mrs. Anderson doesn’t allow Dan to have a pie before dinner.

D. Not all members of their family like pies.

E. Dan is going to buy cherries for a cherry pie.

F. Dan usually comes home at 3.30 after school.

G. Mrs. Anderson is going to cook meat for dinner.

Решение

Dan: Mom, I am home.

Mrs. Anderson: How was school, Dan? How did you do on the test?

Dan: School was OK, and I did great on the test. Mom, I was so worried about that test, but now I feel great. What a relief!

Mrs. Anderson: I am glad to hear that. You have been studying so hard the past few weeks. Now, you can relax and enjoy life.

Dan: What are you cooking? It smells so good.

Mrs. Anderson: I am baking cakes. This is your favorite carrot cake.

Dan: It looks really yummy. And I see muffins over there too. You were busy, weren’t you?

Mrs. Anderson: Yes. Jeff has to take something to school tomorrow. So, those muffins are for him. Don’t touch them.

Dan: Can I have a piece of carrot cake? I want to enjoy life right now.

Mrs. Anderson: You don’t want to wait until after dinner?

Dan: It looks inviting, and I bet it is delicious. No, I don’t want to wait. Can I, mom?

Mrs. Anderson: OK, go ahead.

Dan: Did you see the new recipe that was posted on Today Cooking’s website? I believe it was called Scrumptious Pie.

Mrs. Anderson: No, I did not. But I want to try that recipe. Your dad loves pie.

Dan: So do I.

Mrs. Anderson: So does Jeff. Our whole family is crazy about pie.

Dan: When do you want to try the new recipe? I want to learn too. Should we bake a cherry pie or an apple pie?

Mrs. Anderson: Since this is the cherry season, let’s make a cherry pie. Tomorrow, I will get some cherries at the supermarket, and we can start baking in the afternoon when you get home from school.

Dan: I need to finish a science project, and I will not get home until 3:30. Will it be too late to start baking, mom? If it is, you can start without me.

Mrs. Anderson: 3:30 PM is fine. I will prepare dinner early, and then I will have everything ready for our baking session before you get home.

Dan: It is a deal.

Mrs. Anderson: Enough about baking pies. I need to start working on today’s dinner. It is three o’clock already.

Dan: What do we have for dinner tonight?

Mrs. Anderson: I will make roast beef and cream of mushroom soup.

Dan: It has been a long time since you made cream of mushroom soup. Do you need any help, mom?

Mrs. Anderson: No, go do your homework and leave the cooking to me.

Dan: Thanks, mom.

Ответ: 1322231

Задача 16

Вы услышите диалог. Определите, какие из приведённых утверждений А–G соответствуют содержанию текста (1 – True), какие не соответствуют (2 – False) и о чём в тексте не сказано, то есть на основании текста нельзя дать ни положительного, ни отрицательного ответа (3 – Not stated). Запишите номер выбранного Вами варианта ответа. Вы можете прослушать запись дважды.

A. The interior and setting of a café is important for Paul.

B. Both Paul and Lizzy go to cafes for their atmosphere.

C. Paul usually falls asleep in libraries.

D. It’s difficult for Paul to concentrate on a book in a café.

E. Both Paul and Lizzy meet new people in cafes and make friends with them.

F. Paul likes studying in a library;.

G. Paul’s favourite bar is at his native place.

Решение

Lizzy: So speaking of places like Starbucks, Paul, what do you think really makes a good café or a good coffee shop?

Paul: Well, Lizzy, number one are the employees themselves. People who run a café who are generally amicable people, and also the setting of cafés. Also I like cafes that have music or shows. That is really cool to me. What do you think?

Lizzy: Actually, I agree with that quite a bit. I think the atmosphere is a very important part of it. And actually I have another question for you. When you go to a café, do you actually go there for the coffee or do you go there for something else and you happen to buy a coffee there?

Paul: Sometimes both. I am a bit of a coffee addict. So I definitely go for the caffeine boost.

Lizzy: Okay. I think for me so often it’s that I want to sit down somewhere and read a book but I want to get out of my house. I want to get out and go somewhere where there’s energy and people. And so I think the atmosphere plays a big role for me because I usually go there to do some work or read a book or study language or things like that.

Paul: That’s so true, I could never study in a library because it’s so quiet that I can fall asleep but if I were to go to a coffee shop, there’s the ability to kind of zone out because there’s so much going on around you, people meeting to chat, plenty of table space to where you can go and you know just kind of sprawl out with all of your different belongings and study or even get lost in a book.

Lizzy: Right. And I think when I’m reading I want a really comfortable cushy chair but I find that actually when I’m studying especially if I’m writing, I’d rather have maybe a wooden chair and a table that I can work with.

Paul: But studying is really not that much fun so if you’re in a café you always have the ability of possibly running into somebody who’s going to distract you. What about your favorite bars, what about the atmosphere for them?

Lizzy: My favorite bars, I really like pub style settings more than places that are strictly a bar. When I go to a bar usually I try not to go alone as much as go with friends. How about you?

Paul: I love bars that are true to like a theme. Like my favorite bar probably in my home town, it’s only beer and wine but it has a massive lawn. And it has like just kind of karaoke that they do on like Sunday nights and it’s all wood. And you can just sit outside at a picnic table and just drink beer and wine. It’s not like you have to be classy, not like you have to dress up. We’re just going to hang out and have a summer time like beer and maybe listen to music if they have a band playing on the lawn.

Lizzy: That sounds like a pretty nice place to me.

Paul: Yeah, it’s really nice.

Ответ: 1132321

Задача 17

Вы услышите диалог. Определите, какие из приведённых утверждений А–G соответствуют содержанию текста (1 – True), какие не соответствуют (2 – False) и о чём в тексте не сказано, то есть на основании текста нельзя дать ни положительного, ни отрицательного ответа (3 – Not stated). Запишите номер выбранного Вами варианта ответа. Вы можете прослушать запись дважды.

A. George thinks online editions can earn more money because there are no cost for paper and no delivery cost.

B. All newspapers have online advertising.

C. Anna heard newspapers don’t get more ad revenew online.

D. Anna thinks books are different because you flick through them.

E. George likes to get background info by clicking links when reading online.

F. Anna and George have similar views about books.

G. Anna and George check the same sites.

Решение

George: Taking one step back to the newspapers, my impression was that newspapers got all their money from the advertisements, and that people buying newspapers basically paid for the paper, the actual printing and maybe the delivery. That the actual cost, that any money they make was through advertisements so I don’t see how that’s any different than if they just had it online where they would get rid of all those costs of delivery and the paper and the printing and then just still have the same advertisement so I don’t understand why they can’t make that transition more smoothly.

Anna: And actually I heard that they get more money from online advertising than from the print advertising so, yeah, I don’t really know why it matters come to think of it.

George: I’m sure that the newspaper will go the way of the Dodo fairly soon.

Anna: Yeah, yeah.

George: But again, why is it that people will accept that but not books?

Anna: ‘Cause I think …I can handle a small article, like one page on screen, but I’m not for a long time. I can’t read the same like. And also with news you just want to flick through it. You don’t really … you don’t use it to relax.

George: That’s true. That’s true. It’s usually more active.

Anna: Yeah, whereas with a book you can read in the bath, or you could read it on the bed, or just … it’s a pleasurable sort of thing rather than a «OK, I’ve got to do this to be …»

George: Up with current events.

Anna: Yeah.

George: Yeah, actually, and one nice feature about reading news online and that I’ve noticed lately when I went to the, I shouldn’t say names, but the BBC web site and so I was reading an article.

Anna: What do you mean?

George: Recently in the news, they were talking about Iran and the elections and things like that, and they mentioned a lot of names of different positions in the government, and on the side was all of these links to background information about those people and about those positions and about the government and the structure of the government and it was really fun to be able to go through it and learn quite a bit quite quickly about the government system in Iran and that kind of thing.

Anna: I see.

George: That’s much better than a newspaper where you read it and go, «Ooh, where’s my encyclopedia?»

Anna: Yeah. Exactly. Yeah, I did a similar thing with a report about Iran, and then looking at people who are campaigning against the government and I was like, «I don’t know these people. But who are they?» and there was a link and I could find out about more which was cool.

George: Yeah, it’s nice. It’s nice. Yeah. So …

Anna: The death of the newspaper.

George: The death of the newspaper. I think it’s inevitable.

Anna: The book still survives.

George: Let’s keep the books. I like that.

Ответ: 1322113

Задача 18

Вы услышите диалог. Определите, какие из приведённых утверждений А–G соответствуют содержанию текста (1 – True), какие не соответствуют (2 – False) и о чём в тексте не сказано, то есть на основании текста нельзя дать ни положительного, ни отрицательного ответа (3 – Not stated). Запишите номер выбранного Вами варианта ответа. Вы можете прослушать запись дважды.

A. You may get welfare from the government for many years.

B.Welfare is not positively looked upon in the USA.

C.If you are on welfare some people feel that you are idle.

D.William thinks people in poor areas are not happy about being on welfare.

E. Almost all people in impoverished areas don’t have proper education.

F. Usually single mothers are not supported by the police.

G. Both Rebecca and William think that people on welfare are stuck in a cycle.

Решение

Rebecca: So what about America? If you lose your job, what happens?

William: Well, we have what you have. It’s not called the dole though, it’s called welfare, and basically what happens is if you lose your job, you can get money from the government, but it’s really short-term. I think it’s only six months or a year, but the biggest difference I think between Europe and America is the word. We call it welfare. And welfare is really negative. If you tell somebody that you’re on welfare, or even on unemployment, then people really look at you negatively.

Rebecca: Why do you think it is negative?

William: Well, actually let me clarify. There’re two: There’s welfare which is kind of more long-term like if you have children, and then there’s unemployment which is money you get from the government if you don’t have a job. If you’re on unemployment, it’s not as bad but people still kind of look at you like you’re a loser. That you should go out and find a job, and if you’re on welfare, it has a really negative connotation. People look at you like you’re just lazy basically.

Rebecca: Is that true if you’re in a community where, like, the majority of people are on welfare? Are there any places like that in America?

William: Yeah, there are some really poor areas that have a lot of people on welfare and I imagine that they probably look at it differently but I think that maybe deep down they probably wouldn’t want to be on welfare. I mean the main problem is that those people probably weren’t given the same quality of education. They don’t have the same job opportunities, so deep down they probably wish they had a better job, but they don’t, so they’re kept in this vicious cycle that keeps them down, so I think that there might even be indignation.

Rebecca: Yeah, that sounds similar to areas of Britain with like extreme deprivation. There’re places like that where three generations have been on the dole. Another problem is for single mothers. Sometimes, the cost of childcare means that it’s not actually worth them getting a job. Is it the same in America?

William: Yeah, definitely. We don’t have any government supported childcare. And actually there’s been instances where law enforcement people like sheriffs and police officers have said that they actually prefer that some single mothers are on welfare.

Rebecca: Really? But why?

William: Because if they’re not then it makes their children be more likely to be infl uenced by crime or by life in crime, so if they don’t have a strong parent figure at home, it could really have affects, you know, later on in life, so it really is sort of this vicious cycle for single mothers in poor areas that they don’t have child care. They don’t have education. They don’t have job opportunities, and so it’s really kind of hard for them to get out, and they feel almost hopeless.

Rebecca: Yes, it sounds like we have some things in common even though our welfare systems are so different, but I hope these people, their children especially can get out of this cycle.

Ответ: 2111321

Задача 19

Вы услышите диалог. Определите, какие из приведённых утверждений А–G соответствуют содержанию текста (1 – True), какие не соответствуют (2 – False) и о чём в тексте не сказано, то есть на основании текста нельзя дать ни положительного, ни отрицательного ответа (3 – Not stated). Запишите номер выбранного Вами варианта ответа. Вы можете прослушать запись дважды.

A. During his year as a nomad, Ted made a lot of friends.

B.During his year of freelancing, Ted did web work.

C.Ted couldn’t plan his journey.

D.The idea of complete freedom was frightening for Ted.

E. Ted realized that he wanted regular working hours.

F. Jessica thinks that being free gives meaning to everybody’s life.

G. Jessica had spent a year freelancing, too.

Решение

Jessica: OK, so we’ve talked about freelancing your skills and living and working abroad. Have you ever done that? Have you ever lived and worked in a…

Ted: Like digitally, just like a kind of nomad?

Jessica: Yeah.

Ted: Yeah, I did. I did it for a year. I was actually a nomad. I lived in five different countries and I did all my work online. I mainly was just doing web stuff but it was an interesting experience. I mean I had never, I thought it would be the perfect life and when I told people what I was going to do, everybody said that’s amazing. You know, I mean basically I said that I was just travelling the world and I would just work from whatever city I was in. I would choose my own itinerary and my own destination and I would work when I wanted to.

Jessica: And then why did you quit?

Ted: Well, I think you referenced it earlier. It actually became incredibly lonely. I was surprised how lonely I became. I would meet people but you didn’t have family and you didn’t have your social network. You didn’t have people that you would see every day at work.

Jessica: No workmates?

Ted: Right. You got no workmates, no-one to go out and have a drink with and after a while you really crave like social interaction and not always being the new person, like people that know your name. I remember the big thing that changed is one day I realised I had nowhere to go, I had noone to see and I can go anywhere in the world. I had complete freedom. I could go to the airport and buy a ticket to any place. I could have gone anywhere in the world. There was nothing to stop me. No schedule, no appointments, no-one to report to and it was actually kind of frightening. Like it was almost paralyzing.

Jessica: It’s like standing on the edge of an abyss looking down.

Ted: It was. It was like, you know, I realized I wanted normalcy again. I wanted to have a schedule. I wanted to, you know, have the norm that I had before so I went back to teaching at university and now in my university job I have a lot of time to travel in between but just living around the world going from place to place, yeah, it wasn’t so great.

Jessica: Well, I think it’s great that you had the chance to explore this option. I’m sure everyone would benefit maybe from trying it and then working out what it is they really value because I’m sure we all think wow freedom, absolute freedom, isn’t that like the ultimate goal but then when you have it, you realize well actually no, some of those constraints that I had were quite good.

Ted: Right.

Jessica: They gave me a sense of being or they give structure to my life or meaning to my life.

Ted: Totally, totally and actually now that I’ve done it, I would do it again. I would know how to do it right. I would do it different than I did it before.

Jessica: But it sounds cool though. I can see the attraction definitely.

Ответ: 3121133

Задача 20

Вы услышите диалог. Определите, какие из приведённых утверждений А–G соответствуют содержанию текста (1 – True), какие не соответствуют (2 – False) и о чём в тексте не сказано, то есть на основании текста нельзя дать ни положительного, ни отрицательного ответа (3 – Not stated). Запишите номер выбранного Вами варианта ответа. Вы можете прослушать запись дважды.

A. Tom is fond of buying expensive clothes.

B. Sometimes Samantha shows off her fashionable clothes.

C. Samantha has much more footwear than she needs.

D. Samantha is a good marathon runner.

E. Samantha gets a lot of T-shirts as presents.

F. Samantha prefers buying clothes of a certain colour.

G. Tom usually catches a bus to go to school.

Решение

Now we are ready to start.

Samantha: So talking about clothes, Tom, do you spent a lot of money on clothes?

Tom: I don’t really spend a lot of money on clothes, Samantha. I think the clothes that I buy last for a long time. And I have a lot of them but I only add to them very slowly over the years.

Samantha: The same for me, I don’t like to spend a lot of money on clothes, but every now and then it is nice to splurge on something that looks really good. There’s some great shops like TJ Maxx and Target and, they’re, you know, half price and not as expensive. But you can still find some great looking things.

Tom: Right. And I think by adding to the collection that I already have, I can wear different styles on different days.

Samantha: That’s true.

Tom: Do you think you have too many clothes by chance, do you feel like you need to get rid of any of your clothes?

Samantha: Right now I have way too many shoes actually. I kind of feel like I have a pair of sneakers that I use for sports and tennis, I have a pair of nice cute shoes that I wear when go out and dress up and then I have a pair of kind of everyday flipflops that I walk around in all the time. But there’s three other only things I need but in my closet right now I have about like probably 12 pairs of shoes and it’s ridiculous like you know I don’t need exactly three pairs of black shoes, one that’s heels, one that’s not, one that’s sandals. So I have way too many shoes for one. Shirts, I have maybe too many t-shirts, they tend to accumulate, if you go to certain events a lot of times you’ll get free t-shirts. I run in a lot of marathons and one of the things they give you is a t-shirt. So they’ve kind of accumulated and I have a lot, a lot of t-shirts. What about you?

Tom: Yeah, I think I have some shirts that I need to get rid of, a few t-shirts that are from events that don’t really look so nice. So I think I need to get rid of some of those.

Samantha: I have a terrible habit of when I find something that fits me well and I think looks good, I’ll buy several of the same thing, just in different colors, so I’m wearing the same thing all the time just with different colors. Yeah, I love hats, I tend to wear a hat whenever I go outside in the summer, when I play tennis, when I go to the beach, all the time. And they get so worn, I need more of them.

Tom: Yeah, I have a few hats. I don’t wear them very often, for me a hat is like a last ditch effort to save a bad hair day, like I woke up late, I have to go or I’ll miss the bus, okay here’s a hat.

Samantha: I do that all the time. Oh, I don’t have enough time to shower this morning, just throw on a hat.

Tom: Yeah.

Samantha: Nobody will ever know, it’s good.

Ответ: 2113123

Рекомендуемые курсы подготовки

Понравилась статья? Поделить с друзьями:
  • The gift economy егэ ответы
  • The geographical position of the russian federation сочинение
  • The generation gap can never be bridged сочинение
  • The garden party егэ ответы
  • The gaming grandma текст егэ ответы